You are on page 1of 183

MATEMATİK

OLİMPİYATI ÇALIŞMA
KİTAPÇIĞI
www.sbelian.wordpress.com
Matematı̇k Olı̇mpı̇yatları
Çalışma Kı̇tapçığı

www.sbelian.wordpress.com

6 Temmuz 2010
İçindekiler

1 Giriş 5
1.1 İlksöz . . . . . . . . . . . . . . . . . . . . . . . . . . . . . . . . 5

2 KONULAR 6
2.1 Denklem Sı̇stemlerı̇ . . . . . . . . . . . . . . . . . . . . . . . . . 6
2.1.1 Çalışma Soruları . . . . . . . . . . . . . . . . . . . . . . 9
2.1.2 Çözümler . . . . . . . . . . . . . . . . . . . . . . . . . . 11
2.2 Repunı̇tler . . . . . . . . . . . . . . . . . . . . . . . . . . . . . . 16
2.2.1 Çalışma Soruları . . . . . . . . . . . . . . . . . . . . . . 19
2.2.2 Çözümler . . . . . . . . . . . . . . . . . . . . . . . . . . 20
2.3 Sophı̇e Germaı̇n Özdeşlı̇ğı̇ . . . . . . . . . . . . . . . . . . . . . 23
2.3.1 Çalışma Soruları . . . . . . . . . . . . . . . . . . . . . . 24
2.3.2 Çözümler . . . . . . . . . . . . . . . . . . . . . . . . . . 26
2.4 Tamkareler Pozı̇tı̇ftı̇r . . . . . . . . . . . . . . . . . . . . . . . . 30
2.4.1 Çalışma Soruları . . . . . . . . . . . . . . . . . . . . . . 31
2.4.2 Çözümler . . . . . . . . . . . . . . . . . . . . . . . . . . 34
2.5 Eşı̇tsı̇zlı̇kler I . . . . . . . . . . . . . . . . . . . . . . . . . . . . 40
2.5.1 Çalışma Soruları . . . . . . . . . . . . . . . . . . . . . . 49
2.6 Eşı̇tsı̇zlı̇kler II . . . . . . . . . . . . . . . . . . . . . . . . . . . . 50
2.6.1 Çalışma Soruları . . . . . . . . . . . . . . . . . . . . . . 58
2.7 İndı̇rgemelı̇ Dı̇zı̇ler . . . . . . . . . . . . . . . . . . . . . . . . . 59
2.7.1 Birinci Dereceden İndirgemeler . . . . . . . . . . . . . . 59

2
2.7.2 İkinci Dereceden İndirgemeler . . . . . . . . . . . . . . . 63
2.7.3 Alıştırmalar . . . . . . . . . . . . . . . . . . . . . . . . . 65
2.8 Yenı̇den Düzenleme [Rearrangement] Eşı̇tsı̇zlı̇ğı̇ . . . . . . . . . 66
2.9 Trı̇gonometrı̇k Değı̇şken Değı̇ştı̇rme . . . . . . . . . . . . . . . . 75
2.9.1 Çalışma Soruları . . . . . . . . . . . . . . . . . . . . . . 78
2.9.2 Çözümler . . . . . . . . . . . . . . . . . . . . . . . . . . 80
2.10 Cebı̇rde Teleskopı̇k Toplamlar ve Çarpımlar . . . . . . . . . . . 90
2.10.1 Çalışma Soruları . . . . . . . . . . . . . . . . . . . . . . 93
2.10.2 Çözümler . . . . . . . . . . . . . . . . . . . . . . . . . . 96
2.11 Tamdeğer Fonksı̇yon Problemlerı̇ . . . . . . . . . . . . . . . . . 102
2.11.1 Çalışma Soruları . . . . . . . . . . . . . . . . . . . . . . 107
2.11.2 Çözümler . . . . . . . . . . . . . . . . . . . . . . . . . . 108
2.12 Bölünebı̇lme ve Asal Sayılar . . . . . . . . . . . . . . . . . . . . 110
2.12.1 Çalışma Soruları . . . . . . . . . . . . . . . . . . . . . . 114
2.12.2 Çözümler . . . . . . . . . . . . . . . . . . . . . . . . . . 116
2.13 Trı̇gonometrı̇de Sonsuz Toplam ve Farklar . . . . . . . . . . . . 122
2.13.1 Çalışma Soruları . . . . . . . . . . . . . . . . . . . . . . 124
2.13.2 Çözümler . . . . . . . . . . . . . . . . . . . . . . . . . . 126
2.14 Kuş, Güvercı̇n, Yuva... . . . . . . . . . . . . . . . . . . . . . . . 133
2.14.1 Çalışma Soruları . . . . . . . . . . . . . . . . . . . . . . 135
2.14.2 Çözümler . . . . . . . . . . . . . . . . . . . . . . . . . . 136
2.15 Üstel Dı̇yofant Denklemlerı̇ . . . . . . . . . . . . . . . . . . . . 137
2.15.1 Çalışma Soruları . . . . . . . . . . . . . . . . . . . . . . 139
2.15.2 Çözümler . . . . . . . . . . . . . . . . . . . . . . . . . . 140
2.16 Kalan Sınıfları [Resı̇dues] . . . . . . . . . . . . . . . . . . . . . 143
2.16.1 Çalışma Soruları . . . . . . . . . . . . . . . . . . . . . . 146
2.16.2 Çözümler . . . . . . . . . . . . . . . . . . . . . . . . . . 147
2.17 Vı̇ete Teoremi . . . . . . . . . . . . . . . . . . . . . . . . . . . . 151
2.17.1 Newton-Girard Formulæ . . . . . . . . . . . . . . . . . . 152
2.17.2 Lagrange İnterpolasyon Teknı̇ğı̇ . . . . . . . . . . . . . . 155
2.17.3 Çalışma Soruları . . . . . . . . . . . . . . . . . . . . . . 157

3
2.17.4 Çözümler . . . . . . . . . . . . . . . . . . . . . . . . . . 159
2.18 Bağıntı Sayıları . . . . . . . . . . . . . . . . . . . . . . . . . . . 163
2.18.1 Çalışma Soruları . . . . . . . . . . . . . . . . . . . . . . 167
2.18.2 Çözümler . . . . . . . . . . . . . . . . . . . . . . . . . . 167
2.19 Lı̇neer Denklemlerı̇n Tamsayı Çözümlerı̇ . . . . . . . . . . . . . 168
2.19.1 Çalışma Soruları . . . . . . . . . . . . . . . . . . . . . . 172
2.19.2 Çözümler . . . . . . . . . . . . . . . . . . . . . . . . . . 173
2.20 Fonksı̇yonel Denklemler . . . . . . . . . . . . . . . . . . . . . . 173
2.20.1 Tek Değişkenliler - Temel Teknikler . . . . . . . . . . . 173
2.20.2 Çok Değişkenliler . . . . . . . . . . . . . . . . . . . . . . 176
2.20.3 Çalışma Soruları . . . . . . . . . . . . . . . . . . . . . . 180
2.20.4 Çözümler . . . . . . . . . . . . . . . . . . . . . . . . . . 180

4
Bölüm 1

Giriş

1.1 İlksöz
Belkide internetin hayatımıza kattığı en büyük artı değer legal paylaşımlar
yapmak ve bilgiyi birbirimizle paylaşmaya ortam hazırlamak olmuştur. Bizde
bu paylaşımın bir parçası olarak bu zamana kadar bir çok ders notunu ve
çözüm paketini sizlerle paylaşmıştık. Şimdi de, tüm bu ders notlarının birle-
şimi olan bu mini kitabı sizlerin paylaşımına açıyoruz. Sizlerde çekinmeden
bu kitapçığı gerek fotokopi ile gerekse elektronik yollarla birbirinizle paylaşın.
Yararlanabildiğiniz kadar yararlanın.
Bu çalışmanın hazırlanmasında öncelikle www.sbelian.wordpress. com sayfa-
mıza teveccüh gösteren tüm dostlarımıza ve faydalı olabilmek için çalışmala-
rımıza yardım eden LATEX 2ε dostlarına teşekkür ederiz.

Sbelian Σ

Haziran 2010

5
Bölüm 2

KONULAR

2.1 Denklem Sı̇stemlerı̇


Bu bölümde bazı standart olmayan denklem sistemlerinin çözümlerini ya-
pacağız. Ancak ilerleyen örneklerde sizinde farkedeceğiniz üzere, kullanacağı-
mız yöntemler genelde bazı cebirsel manipülasyonlardan oluşuyor. Bu yön-
temleri kullanarak hem çözümlere daha kolay ulaşacağız hemde her bir soruda
farklı bir tekniği öğrenmiş olacağız.

Örnek.
x + y2 + z2 = 3
y + z 2 + x3 = 3
z + x2 + y 3 = 3
denklem sisteminin pozitif çözüm üçlüsünün yanlızca (1, 1, 1) olduğunu kanıt-
layınız.

Çözüm. Eğer ilk iki denklemin farkını alırsak


x(1 − x2 ) + y(y − 1) + z 2 (z − 1) = 0
denklemini elde ederiz. Benzer şekilde ikinci ve üçüncü denklemlerin farkını
alırsak
y(1 − y 2 ) + z(z − 1) + x2 (x − 1) = 0

6
2.1. DENKLEM SİSTEMLERİ 7

denklemini elde ederiz. Eğer bu denklemi z ile çarpıp bir önceki denklemden
çıkarırsak
x(x − 1)(1 + x + xz) = y(y − 1)(1 + z + yz)
eşitliğini elde ederiz. Benzer işlemleri yaparak

y(y − 1)(1 + y + yx) = z(z − 1)(1 + x + xz)

eşitliğinede ulaşırız. Son yazdığımız iki eşitlikte eğer x, y, z pozitifse x = y =


z = 1, x, y, z < 1 veya x, y, z > 1 olacaktır. Son iki yazdığımız olasılığın

x + y2 + z3 = 3

eşitliğini sağlayamayacağı açıktır. Demek ki tek çözüm

(x, y, z) = (1, 1, 1)

olacaktır.

Örnek.

a + b + c + d = 12
abcd = 27 + ab + ac + ad + bc + bd + cd

denklem sistemini sağlayan tüm (a, b, c, d) pozitif sayı dörtlülerini bulunuz.

Çözüm. Eğer soruda verilen sistemdeki ikinci eşitliğe Aritmetik Orta -


Geometrik Orta eşitsizliğini uygularsak

abcd ≥ 27 + 6 abcd

olacaktır. Eğer bu eşitsizliği düzenlersek yeni denklemimiz abcd değişkenine
bağlı √ √
( abcd + 3)( abcd − 9) ≥ 0
eşitsizliğini elde ederiz. Burada

abcd ≥ 9
8 BÖLÜM 2. KONULAR

olduğu açıktır. Eğer bu durumu sistemdeki ilk denklemle beraber kullanırsak

a+b+c+d √
4
≤ abcd ⇒ 3 ≤ 3
4
olduğundan eşitlik durumu söz konusudur. Buna göre tek çözüm

a=b=c=d=3

olacaktır.

Örnek.
µ ¶
√ 1
3x 1 + = 2
x+y
µ ¶
p 1 √
7y 1 − = 4 2
x+y

denklem sistemini pozitif reel sayılarda çözünüz.

Çözüm. Bu soruda çözüme daha kolay işlemlerle ulaşmak için bazı değiş-
√ √
ken eğiştirmeler yapmak yerinde olacaktır. Buna göre eğer x = u ve y = v
alırsak sistemimiz
µ ¶
1 2
u 1+ 2 2
= √
u +v 3
µ ¶ √
1 4 2
v 1− 2 = √
u + v2 7

şeklini alır. Burada u2 + v 2 aslında z = u + iv karmaşık sayısının normunun ka-


residir. Buna göre ikinci denklemi i karmaşık sayısı ile çarpıp birinci denkleme
eklersek  
u − iv 2
u + iv + 2 = √ √ 
u + v2 3 + i 4√ 2
7

eşitliğini elde ederiz. Burada


u − iv
u2 + v 2
2.1. DENKLEM SİSTEMLERİ 9

ifadesi
z z 1
2
= =
|z| (zz) z
olacaktır. Yani à √ !
1 2 4 2
z+ = √ +i √
z 3 7
eşitliği elde edilir. Eğer bu denklemi düzenlersek
à √ !
2 2 4 2
z − √ +i √ z+1=0
3 7

denklemini ve bu denklemin çözümü olan


µ ¶ Ã √ !
1 2 2 2 √
√ ±√ +i √ ± 2
3 21 7

sayısını elde ederiz. Buna göre soruda verilen sistemin çözümü


µ ¶2 Ã √ !2
1 2 2 2 √
x= √ ±√ ve y = √ ± 2
3 21 7

olacaktır.

2.1.1 Çalışma Soruları


1.
2
x+ = 2y
x
2
y+ = 2z
y
2
z+ = 2x
z
denklem sistemini çözünüz.
10 BÖLÜM 2. KONULAR

2. [x], x sayısının tam kısmını, {x} ise ondalık kısmını temsil etmek üzere
x = [x] + {x} olarak veriliyor. Buna göre

x + [y] + {z} = 1, 1
z + [x] + {y} = 2, 2
y + {x} + [z] = 3, 3

denklem sistemini sağlayan x, y, z değişkenlerini bulunuz.

3. Aşağıdaki denklem sistemini pozitif reel sayılarda çözünüz.

xy + yz + zx = 12
xyz = 2 + x + y + z

4.

4x2
= y
4x2 + 1
4y 2
= z
4y 2 + 1
4z 2
= x
4z 2 + 1
Denklem sisteminin tüm reel çözümlerini bulunuz.

5.
3 = x + y + z = x3 + y 3 + z 3
eşitliğini sağlayan tüm tamsayı üçlülerini bulunuz.

6.
6(x − y −1 ) = 3(y − z −1 ) = 2(z − x−1 ) = xyz − (xyz)−1
eşitliğini sağlayan tüm sıfıran farklı x, y, z reel sayılarını bulunuz.
2.1. DENKLEM SİSTEMLERİ 11

7. a, b, c birbirinden ve sıfırdan farklı reel sayılar olmak üzere verilen


x2 − yz = a
y 2 − zx = b
z 2 − xy = c
denklem sisteminin tüm (x, y, z) reel üçlülerini bulunuz.
8.
(x + y)3 = z
(y + z)3 = x
(z + x)3 = y
denklem sistemini sağlayan tüm (x, y, z) reel sayı üçlüleini bulunuz.
9.
x3 − 9(y 2 − 3y + 3) = 0
y 3 − 9(z 2 − 3z + 3) = 0
z 3 − 9(x2 − 3x + 3) = 0
denklem sisteminin tüm çözümlerini bulunuz.
10. a, b, x, y reel sayılar olmak üzere
ax + by = 3
ax2 + by 2 = 7
ax3 + by 3 = 16
ax4 + by 4 = 42
olarak veriliyor. Buna göre, ax5 + by 5 toplamının eşitini bulunuz.

2.1.2 Çözümler
1. Eğer çözümde aritmetik orta - geometrik orta eşitsizliğini kullanırsak
r
1 2 √
x+ ≥2 x· =2 2
x x
12 BÖLÜM 2. KONULAR

√ √ √ √
ise 2y ≥ 2 2 ve y ≥ 2 olacaktır. Benzer şekilde y ≥ 2 ve z ≥ 2
olacaktır. Eğer sistemdeki 3 denklemi −1 ile çarpıp toplarsak
µ ¶
1 1 1
x+y+z =2 + +
x y z

eşitliği elde edilir. Bu denklemin sol kısmı



x+y+x≥3 2

iken sağ kısım µ ¶


1 1 1 √
2 + + ≤3 2
x y z
olacaktır. Demek ki
µ ¶
1 1 1 √
x+y+z =2 + + =3 2
x y z

olmalıdır. Buradan istenen çözümler



x=y=z=± 2

olarak bulunur.

2. Eğer sistemdeki 3 eşitsizliği taraf tarafa toplarsak

2x + 2y + 2z = 6.6

olduğundan
x + y + z = 3.3
denklemini elde ederiz. Sistemdeki her bir denklemi son bulduğumuz
eşitlikten çıkarırsak yeni sistemimiz

{y} + [z] = 2.2


{x} + [y] = 1.1
{z} + [x] = 0
2.1. DENKLEM SİSTEMLERİ 13

olacaktır. Bu sistemde ilk denklemden {y} = 0.2 ve [z] = 2 bulunur.


İkinci denklemden [y] = 1, {0.1} ve üçüncü denklemden {z} = 0, [x] = 0
bulunur. Buna göre, denklem sisteminin çözümü
x = 0.1, y = 1.2, z = 2
olacaktır.

3. Varsayalım 3 xyz = a olsun. Aritmetik orta - geometrik orta eşitsizliğin-

den
12 = xy + yz + zx ≥ 3a2
ve
a3 = 2 + x + y + z ≥ 2 + 3a
olacaktır. ilk eşitsizlikten 12 ≥ 3a2 ise 4 ≥ a2 bulunur. İkinci eşitsizlik-
tense a ≥ 2 bulunur. Dolayısıyla a = 2 ve x = y = z olacaktır. Demek
ki tek çözüm
(x, y, z) = (2, 2, 2)
olur.
4. Çözüme, denklem sistemimizdeki her bir denklemi karşılayan bir fonk-
siyon bularak başlayalım. Varsayalım f fonksiyonu f : [0, ∞) → [0, ∞)
olmak üzere
4t2
f (t) = 2
4t + 1
olarak verilsin. f fonksiyonun artan olduğu açıktır. Dolayısıyla eğer x < y
ise f (x) < f (y) dolayısıyla y < z olacaktır. Eğer bu argümanı tekrar
edersek z < x elde ederiz. Öyleyse x < y < z < x olacaktır ki bu durum
imkansızdır. Benzer biçimde x > y ile başlarsak yine çelişki elde ederiz.
Demek ki x = y = z olmalıdır. Buna göre,
4t2
=t
4t2 + 1
denklemini çözersek t = 0 veya t = 1/2 olacaktır. Öyleyse sistemin
çözümleri sadece
1 1 1
(0, 0, 0) ve ( , , )
2 2 2
olarak bulunur.
14 BÖLÜM 2. KONULAR

5. Eğer soruda verilen eşitlik üzerinde biraz oynarsak


X
24 = (x + y + z)3 − (x3 + y 3 + z 3 ) = 3 x2 y + 6xyz

eşitliğine ulaşırız. Eğer sadeleştirme yapıp çarpanlarına ayırırsak

8 = (x + y)(x + z)(y + z)

olacağından
8 = (3 − x)(3 − y)(x + y)
eşitliği elde edilir. 8 sayısının çarpanlarını kontrol edersek çözümleri

(1, 1, 1), (4, 4, −5), (4, −5, 4), (−5, 4, 4)

olarak bulabiliriz.

6. Soruda verilen eşitliği kullanarak

(x − y −1 ) + (y − z −1 ) + (z − x−1 ) = xyz − (xyz)−1

yazabiliriz. Eğer bu eşitliği yeniden düzenlersek de

(x − y −1 )(y − z −1 )(z − x−1 ) = 0

denklemini elde ederiz. Demek ki çarpanlardan biri sıfır olmalıdır. Ancak


sorudaki eşitlikten hepsinin sıfır olduğu ortaya çıkar. Dolayısıyla

xy = yz = zx = 1 ve x = y = z = ±1

bulunur.

7. Soruda verilen sistemdeki herbir eşitliğin karesini alıp diğer iki eşitliğin
çarpımından çıkarırsak yeni sistemimimiz

a2 − bc = x(x3 + y 3 + z 3 − 3xyz)
b2 − ac = y(x3 + y 3 + z 3 − 3xyz)
c2 − ab = z(x3 + y 3 + z 3 − 3xyz)
2.1. DENKLEM SİSTEMLERİ 15

olacaktır. Burada
x3 + y 3 + z 3 − 3xyz = k
olarak alırsak

(a2 − bc)2 − (b2 − ac)2 (c2 − ab) = k 2 (x2 − yz) = k 2 a

olur. Burada eşitliğin sol tarafını açıp düzenlersek


p
k = ± a3 + b3 + c3 − 3abc

olacaktır. Buna göre sistemin çözümleri

a2 − bc b2 − ac c2 − ab
x= , y= ,
k k k
olacaktır.

8. Eğer ikinci denklemi birinciden çıkarırsak


¡ ¢
(x − z) (x + y)2 + (x + y)(y + z) + (y + z)2 = z − x

eşitliğini elde ederiz. Burada

(x + y)2 + (x + y)(y + z) + (y + z)2 > 0

olduğuna göre, x = z olacaktır. Simetriden dolayı y = z ve elimizde


2
8x3 = x denklemi oluşacaktır. Bu denklemin kökleri x = 0 ve x = ± 2√ 2
olacaktır. Buna göre sistemin çözümleri
1
x = y = z = 0, x = y = z = ± √
2 2
olur.

9. Soruda verilen sistemi düzenlersek, yeni sistemimiz

(y − 3)3 = y 3 − x3
(z − 3)3 = z 3 − x3
(x − 3)3 = x3 − z 3
16 BÖLÜM 2. KONULAR

olacaktır. Bu eşitliği toplarsak

(x − 3)3 + (y − 3)3 + (z − 3)3 = 0

eşitliği elde edilir. Genelliği bozmadan varsayalım x ≥ 3 olsun. Sistem-


deki üçüncü denklemden

z 3 − 27 = 9x(x − 3)

olacaktır, dolayısıyla z ≥ 3 olur. Benzer biçimde y ≥ 3 olacaktır. Ancak


üçü birden 3’ten büyük olamaz. Buna göre tek çözüm x = y = z = 3
olacaktır.

10. n = 2 ve n = 3 için

(axn + by n )(x + y) − (axn−1 + by n−1 )xy = axn+1 + by n+1

eşitliğinin sağlandığı görülebilir. Buna göre,

7(x + y) − 3xy = 16

ve
16(x + y) − 7xy = 42
olacaktır. Bu iki denklemi çözersek x + y = −14 ve xy = −38 olur. n = 4
için başta belirlediğimiz reküransı yeniden uygularsak

ax5 + by 5 = (42)(−14) − (16)(−38) = −588 + 608 = 20

olacaktır.

2.2 Repunı̇tler
Basamaklarını oluşturan rakamların hepsi 1 olan doğal sayılara repunit
diyeceğiz. Öyleki
111
| ·{z
· · 111}
n basamaklı
2.2. REPUNİTLER 17

sayısı bir repunit sayı olarak sorularımızda yerini alacaktır. Şimdi ilk örne-
ğimizle başlayalım. Sorumuz 2005 Bulgaristan Matematik Olimpiyatları’nda
sorulmuştur.

Örnek. x, y, z ∈ Z olmak üzere verilen

x2 + 2y 2 + 98z 2 = 111
| ·{z
· · 111}
666 tane 1

eşitliğini sağlayan tamsayı üçlülerinin bulunmadığını kanıtlayınız.

Çözüm I. Çözüme çelişki ile ulaşmaya çalışalım. varsayalım soruda ve-


rilen denklemi sağlayan (x, y, z) tamsayı üçlüsü varolsun. Buna göre eşitliğin
sol tarafındaki repuniti çarpanlarına ayırırsak

106 − 1
111111 · (1 + 106 + · · · + 106·110 ) = (1 + 106 + · · · + 106·110 )
9

olacaktır. Küçük Fermat teoremine göre, 106 − 1 sayısı 7 ile bölünebilir. Buna
göre eşitliğimize (mod7) altında bakabiliriz. Tamkare bir sayının (mod7) al-
tındaki kalan sınıfı {0, 1, 2, 3, 4} ve 98 sayısı 7 ile kalansız bölünebildiğine göre,
eşitliğin sağ tarafındaki ifademizde x ve y sayıları 7 ile kalansız bölünmelidir.
Bu durumda eşitliğin sol tarafı 72 ile tam bölünecektir. Ancak eşitliğin sağ
kısmındaki ifademiz

7 · 15873 · (1 + 106 + · · · + 106·110 )

olduğundan bu ifadenin (mod7) altındaki ikinci çarpanı 4 olacaktır. Üçüncü


çarpanı (mod7) altında 0 olmadığına göre, eşitliğin sağ tarafı 49 ile bölüne-
mez. Çelişki vardır. Demekki (x, y, z) tamsayı üçlüleri yoktur.

Çözüm II. Eğer soruda verilen eşitliğin sağ tarafına (mod) altında ba-
karsak
x2 + 2y 2 + 2x2 ≡ 111 (mod 8)
veya
x2 + 2y 2 + 2z 2 ≡ 7 (mod 8)
18 BÖLÜM 2. KONULAR

olduğunu görebiliriz. Bir tamsayının karesinin ( mod 8) altındaki kalanları {0, 1, 4}


olabilir. Eğer iki katlarını alırsak kalan sınıfı {0, 2} olacaktır. Ancak elde edilen
{0, 1, 4} kümesinin elemanlarını {0, 2} ve {0, 2} kümesinin elemanlarına ekleye-
rek (mod8) altında 7 sayısına ulaşamayız. Demek ki, soruda verilen denklemi
sağlayan (x, y, z) tamsayıları yoktur.
Sıradaki sorumuzda Rusyada yayınlanan Potansia Magazine isimli dergi-
den alıntıdır.

Örnek. Repunitleri yine repunitlere götüren tüm ikinci dereceden tam-


sayı katsayılı polinomları bulunuz.

Çözüm. varsayalım ikinci dereceden polinomumuz f (x) = ax2 + bx + c


olsun. Soruda verilen şartlara göre,

f (111
| {z· · · 1}) = |111{z
· · · 1}
m tane n tane

yani µ ¶
10m − 1 10n − 1
f =
9 9
olacaktır. Eğer µ ¶
x−1
g(x) = 9f +1
9
olarak alırsak,
à µ ¶ µ ¶ ! µ ¶
x−1 2 x−1 a 2 b
9 a +b +1 = 9 (x − 2x + 1) + (x − 1) + c + 1
9 9 92 9
a 2
= (x − 2x + 1) + b(x − 1) + c + 1
9 µ ¶
a 2 2a a
= x + b− x + (9c + 1 − b + )
9 9 9

olacaktır. Demek ki
µ ¶
a 2 2a a
g(x) = x + b − x + (9c + 1 − b + )
9 9 9
2.2. REPUNİTLER 19

olacaktır. Buna göre, aslında g(10m ) = 10n olacaktır. yani g polinomu 10’un
kuvvetlerini yine 10’un kuvvetlerine götürecektir. Buna göre,
µ ¶ ³
a 2a a ´ −2m
10−2m · g(10m ) = 10n−2m = + b − 10−m + 9c + 1 − b + 10
9 9 9
olacaktır. İlk eşitlikten görüldüğü üzere 10−2m · g(10m ) sayısı 10’un kuvvetidir.
Eğer m değerini yeterince büyük alırsak 10−2m g(10m ) ifadesi a/9 değerine
yakınsak. Buradan da, a/9 = 10k ve 10−2m g(10m ) = 10k olacaktır. Dolayısıyla
µ ¶ ³
2a a ´ −2m
b− 10−m + 9c + 1 − b + 10 =0
9 9
olacağından
2a a
b− = 9c + 1 − b + = 0
9 9
10k −1
olur. Buradan b = 2 · 10k ve c = 9 olacağından istenen polinomlar
10k − 1
f (x) = 9 · 10k · x2 + 2 · 10k · x +
9
olur.

2.2.1 Çalışma Soruları


1. (a.) Çift sayıda basamağı olan, beş tabanındaki repunitlerin ardışık po-
zitif iki tamsayının çarpımı olduğunu gösteriniz.
(b.) Dokuz tabanındaki tüm repunitlerin Üçgensel Sayı 1 olduğunu ka-
nıtlayınız.
2.
|111{z
· · · 1} = 222
| {z· · · 2} +(333
| {z· · · 3})2
2n tane n tane n tane
eşitliğini kanıtlayınız.
1
Bir ak üçgensel sayısı, 1’den k’ya kadar olan tamsayıların toplamıdır. Üçgensel sayı
denilmesinin temel sebebi, eşkenar bir üçgenin üzerine eşit aralıklarla yerleştirilibilecek eşit
yarıçaplı kürelerin sayısını vermesidir. Öyleki, ilk bir kaç üçgensel sayı 1, 3, 6, 10, 15, 21, 28
olarak kolaylıkla bulunabilir.
2 ¡ ¢
an = 1 + 2 + 3 + · · · + (n − 1) + n = n(n+1)
2
= n 2+n = n+1 2
genel formuda n. üçgensel sayıyı
vermektedir.
20 BÖLÜM 2. KONULAR

3. 19 ile bölünebilen ve tüm basamakları 1 olan en küçük pozitif tamsayıyı


bulunuz.

4. Asal bir repunit sayının basamak sayısınında asal olduğunu kanıtlayınız.


Ayrıca, bu önermenin tersi doğrumudur?

5. Tüm basamakları 1 olan, 81 basamaklı bir sayı 81 ile kalansız bölünebilir


mi?

6. (a.) 111
| {z· · · 1} sayının 41 ile ancak ve ancak 5|n olduğunda, bölünebile-
n basamaklı
ceğini kanıtlayınız.
(b.) 111
| {z· · · 1} sayının 91 ile ancak ve ancak 6|n olduğunda, bölünebile-
n basamaklı
ceğini kanıtlayınız.

7. 1’den büyük hiç bir repunitin tam kare olamayacağını kanıtlayınız.

8. Son basamağı 1, 3, 7 veya 9 olan her n tamsayısı için, n ile bölünebilen


bir repunit bulunduğunu kanıtlayınız.

9. Herhangi iki elemanı aralarında asal olan sonsuz büyüklükte bir repu-
nitler dizisi olduğunu kanıtlayınız.

10. Sonsuz çoklukta n değeri için, basamaklarında sıfır bulunmayan n basa-


maklı bir sayının basamakları toplamına bölünebileceğini kanıtlayınız.

2.2.2 Çözümler
1. (a.)

(111
| {z· · · 1})5 = 1 + 5 + · · · + 52n−1
2n tane
52n − 1 5n − 1 5n + 1
= = ·
4 2 2
5n −1 5n +1
eşitliğine göre, 2 ve 2 ardışık tamsayılardır.
2.2. REPUNİTLER 21

(b.)

(111
| {z· · · 1})9 = 1 + 9 + · · · + 9n−1
n tane
9n − 1 1 3n − 1 3n + 1
= = · ·
8 2 2 2
3n −1 3n +1
olduğuna göre bu sayı bir üçgensel sayıdır. Çünkü, 2 ve 2
ardışık tamsayılardır.

2. Soruda verilen eşitliği kullanalım. Buna göre,

|111{z
· · · 1} − 222
| {z· · · 2} = |111{z
· · · 1} 000
| {z· · · 0} − 111
| {z· · · 1}
2n tane n tane n tane n tane n tane
n
= |111{z
· · · 1} ·(10 − 1) = 111
| {z· · · 1} · 999
| {z· 9}
n tane n tane n tane
2
= |333{z
· · · 3} · 333
| {z· · · 3} = (333
| {z· 3})
n tane n tane n tane

olur.

3. Varsayalım A sorudaki şartları sağlayan sayımız olsun. Eğer basamak


n
sayısını n olarak alırsak A = 10 9−1 olacaktır. 9 ve 19 sayıları aralarında
asal olduklarına göre, en küçük n değeri için, 10n − 1 sayısının 19 ile
bölünmesi gerekmektedir. Küçük Fermat teoremine göre, 1018 − 1 sayısı
19 ile bölünebilir. Eğer daha küçk bir n değeri varsa, 10n ≡ 1 (mod19) ve
n|18 olmalıdır. Bu durumda 2, 3, 6 ve 9 değerlerini kontrol etmemiz gere-
kir. Ancak bu sayılardan hiçbirisinin denkliği sağlamadığı görülecektir.
Demek ki en küçük sayımız 19 basamaklıdır.

4. m, n > 1 olmak üzere repunit sayımız m × n basamaklıysa, bu sayıyı

111
| {z· · · 1} ×1 000
| {z· · · 0} 1 000
| {z· · · 0} 1 · · · 1 000
| {z· · · 0} 1
n tane n−1 tane n−1 tane n−1 tane

şeklinde çarpanlara ayırabiliriz. Burada m − 1 tane sıfırlardan oluşmuş


grup vardır. Dolayısıyla bu şekildekii bir repunit asal olamaz. Demek ki,
22 BÖLÜM 2. KONULAR

asal bir repunit için, basamak sayısıda asal olmalıdır. Ancak tersi doğru
değildir. Mesela 111 = 3 × 37 ve

111
| {z· · · 1} = 21649 × 513239
11 tane

sayıları asal değildirler.

5. Soruda verilen sayıyı çarpanlarına ayırmaya çalışalım. Buna göre,

111
| {z· · · 1} = 111
| {z· · · 1} × 100
| · · · 0100{z
· · · 010 · · · 01}
81 tane 9 tane 9 tane 1 64 tane 0

olur. Çarpanların her ikiside 9 ile bölünebilir. Dolayısıyla sayımız 81 ile


bölünebilir.

6. (a.) Varsayalım n = 5k + r, r ∈ {0, 1, 2, 3, 4} olsun. Buna göre,

· · · 1} = 111
|111{z | {z· · · 1} 000
| {z· · · 0} + 111
| {z· · · 1} = 11111×100001 · · · 00001+111
| {z· · · 1}
n tane 5k tane r tane r tane r tane

olacaktır. 11111 = 41 × 271 olduğuna göre, ifade mod41 altında


|111{z
· · · 1}’e eşittir. Fakat 1, 11, 111, 1111 sayıları 41 ile bölünemez.
r tane
Dolayısıyla 111
| {z· · · 1} sayısının 41 ile bölünebilmesi için ancak ve an-
n tane
cak r = 0 yani 5|n olmalıdır.
(b.) Önceki şıkta kullanılan aynı yöntemle çözüme gidilir. Sadece 111111 =
91 × 1221 olacaktır.

7. Varsayalım sayımız
10n − 1
A = 111
| {z· · · 1} =
9
n tane

bir tamkare olsun. Buna göre, A ≡ 111 · · · 1 ≡ 11 ≡ 3 (mod4) olur.


Ancak bu imkansızdır çünkü bir tamkarenin (mod4) altıdaki denkleri
sadece {0, 1} olabilir.
2.3. SOPHİE GERMAİN ÖZDEŞLİĞİ 23

8. Eğer dikkat edilirse 1, 3, 7 ve 9 sayılarının 10 ile aralarında asal oldukları


kolayca görülebilir. Varsayalım elimizde 1, 11, 111, · · · , 111
| {z· · · 1} sayıları-
n+1 tane
mız olsun. Güvercin Yuvası İlkesine göre, bu (n + 1) sayı arasından iki
tanesi n ile bölündüğünde aynı kalanları verir. Bu sayıların farkı ise n ile
bölünebilir ve fark a · b formundadır. Burada a bir repunit ve b ise 10’un
kuvvetidir. (n, 10) = 1 olduğuna göre, repunit a sayısı 10 ile bölünmeli-
dir. ispat tamamlanır.

9. Euclide’in sosuz çoklukta asalın varlığını kanıtladığı ispatını sorumuza


adapte edelim. Varsayalım a1 = 1 olsun ve dizinin elemanlarını an ’e ka-
dar seçelim. Bir önceki problemden a1 · a2 · · · · · an çarpımını bölen bir
m repunit sayısı vardır. 10m + 1 sayısıda bir repunittir ve m ile arala-
rında asaldırlar. Dolayısıyla 1 ≤ k ≤ n olmak üzere ak ’larda aralarında
asaldırlar. Eğer an+1 = 10m + 1 seçersek kanıt tamamlanır.

10. Tümevarımla ispatlamaya çalışalım. Buna göre 3n basamaklı bir repu-


nitin 3n ile bölünebileceğini göstermemiz yeterli olacaktır. n = 1 için
111 = 3 × 37 olur. Varsayalım n basamak içinde doğru olsun. Buna göre,

111
| {z· · · 1} = 111
| {z· · · 1} ×1 000
| {z· · · 0} 1 000
| {z· · · 0} 1
3n+1 tane 3n tane 3n −1 tane 3n −1 tane

olacaktır. İlk çarpanın 3n ile bölünebildiği açıktır. İkinci çarpanda 3 ile


kalansız bölünebileceğine göre, (n + 1) için ispatı tamamlarız.

2.3 Sophı̇e Germaı̇n Özdeşlı̇ğı̇


x2 + 1 polinomunun R üzerinde indirgenemeyen yada çarpanlarına ayrıla-
mayan bir polinom olduğunu biliyoruz. Ancak, her nekadar benzer gibi görün-
sede, x4 + 1 polinomu için durum aynı değildir. Öyleki,
¡ ¢
x4 + 1 = x4 + 2x2 + 1 − 2x2
¡ ¢2 ³√ ´2
= x2 + 1 − 2x
³ √ ´ ³ √ ´
= x2 + 2x + 1 x2 − 2x + 1
24 BÖLÜM 2. KONULAR

olacaktır. Benzer biçimde


¡ ¢¡ ¢
x4 + 4y 4 = x2 + 2xy + 2y 2 x2 − 2xy + 2y 2

özdeşliğide elde edilebilir. Eğer katsayıyı değitirirsek,


µ ¶µ ¶
4 1 4 2 1 2 2 1 2
x + y = x + xy + y x − xy + y
4 2 2
olacaktır. Bu son yazdığımız iki özdeşlik sorularn çözümünde oldukça kulla-
nışlıdır ve literatürdeki adıda Sophie Germain Identity olarak geçer.

Örnek.
n
X 4k
4k 4
+1
k=1
olarak verilen toplamı bulunuz.

Çözüm. Eğer bildiğimiz özdeşlikleri kullanırsak,


n
X n
X
4k (2k 2 + 2k + 1) − (2k 2 − 2k + 1)
4
=
4k + 1 (2k 2 + 2k + 1)(2k 2 − 2k + 1)
k=1 k=1
Xn µ ¶
1 1
= 2
− 2
(2k + 2k + 1) (2(k + 1) + 2(k + 1) + 1)
k=1
1
= 1−
2n2 + 2n + 1
olacaktır. Yukarıda çözümlerini verdiğimiz örnekler konunun anlaşılması için
yeterlidir. Şimdi bu bilgiler ışığında çalışma sorularını çözmeye çalışınız.

2.3.1 Çalışma Soruları


n −2
1. n > 2, n ∈ Z için 22 + 1 sayısının asal olmadığını kanıtlayınız.

2. √
xn+1 + xn−1 = 2xn
dizisinin periyodik olduğunu kanıtlayınız.
2.3. SOPHİE GERMAİN ÖZDEŞLİĞİ 25

3.
n
X k 2 − 1/2
k 4 + 1/4
k=1

toplamını hesaplayınız.

4. ³ ´
¡ 4 1¢ ¡ 4 1¢
1 + 4 3 + 4 · · · · · · (2n − 1)4 + 14
¡ ¢¡ ¢ ³ ´
24 + 14 44 + 41 · · · · · · (2n)4 + 41

ifadesinin eşitini bulunuz.

5. n4 + a sayısının asal olmamasını sağlayan sonsuz sayıda pozitif a değeri


olduğunu kanıtlayınız.

6. n4 + 4n sayısının ancak ve ancak n = 1 durumunda asal olduğunu kanıt-


layınız.

7. P (x) = x4 +6x2 −4x+1 polinomu veriliyor. Buna göre, P (x4 ) polinomu-


nun katsayıları tamsayı olan ve derecesi 1’den büyük olan iki polinomun
çarpımı şeklinde yazılabileceğini kanıtlayınız.

8. n12 + 64 polinomunun her biri 1’den büyük 4 farklı çarpanın çarpımı


olarak yazılabileceğini kanıtlayınız.

9. m, n pozitif tamsayılardır, m bir çift sayı ise

m
X
(−4)k n4(m−k)
k=0

toplamının asal olmadığını kanıtlayınız.

10. P (x) = xn−4 + 4n polinomunun, sabit polinomdan farklı 4 polinomun


çarpımı şeklinde yazılmasını sağlayan en küçük n değeri kaçtır?
26 BÖLÜM 2. KONULAR

2.3.2 Çözümler
1. Soruda verilen ifadeyi

n −2 m4 n−2
22 +1=1+ , m = 22
4
olarak yazabiliriz . Buna göre,
µ ¶µ ¶
m4 1 1
1+ = 1 + m + m2 1 − m + m2
4 2 2

olacağından sayımız asal değildir.

2. x4 + 1 polinomunu çarpanlarına ayırırsak,


³ √ ´³ √ ´
x4 + 1 = x2 − 2x + 1 x2 + 2x + 1

olacaktır. Buna α ve β sayıları x2 + 2x + 1 karakteristiğinin kökleri
ise bunlar
√ aynı zamanda x4 + 1 polinomunun kökleridir. Benzer biçimde
x2 + 2x + 1 içinde aynı durum söz konusu ise, xn = a · αn + b · β n
eşitliğinden, dizinin periyodunun 8 olduğu bulunur.
¡ ¢¡ ¢
3. k 4 + 14 = k 2 − k + 12 k 2 + k + 12 olduğuna göre,
n n µ ¶ n
à !
X k 2 − 1/2 X k − 1/2 k + 1/2 X k − 1/2 (k + 1) − 1/2
= − 2 = −
k 4 + 1/4 2
k − k + 1/2 k + k + 1/2 k − k + 1/2 (k + 1)2 − (k + 1) + 1/2
2
k=1 k=1 k=1

olacağından toplam
(2n + 1)
1−
(2n2 + 2n + 1)
olarak bulunacaktır.

4. Soruda verilen ifadeyi genelleştirirsek,

Yn
(2k − 1)4 + 1/4
k=1
(2k)4 + 1/4
2.3. SOPHİE GERMAİN ÖZDEŞLİĞİ 27

ifadesini elde ederiz. Eğer bu ifadenin pay ve paydasını çarpanlına ayı-


rırsak,
³ ´³ ´
Yn (2k − 1)2 + (2k − 1) + 1/2 (2k − 1)2 − (2k − 1) + 1/2
³ ´³ ´
2 2
k=1 (2k) + 2k + 1/2 (2k) − 2k + 1/2

durumunu elde ederiz. Ancak

m2 − m + 1/2 = (m − 1)2 + (m − 1) + 1/2

olduğundan pay ve paydadaki çarpanlardan birer tanesi sadeleşir. Sadece


12 − 1 + 1/2 pay kısmında ve (2n)2 + 2n + 1/2 paydada kalır. Dolayısıyla
istenilen cevap
1
2
8n + 4n + 1
olacaktır.

5. Eğer a = 4k 4 , k > 1 olarak alırsak,


¡ ¢¡ ¢
n4 + 4k 4 = n2 + 2nk + 2k 2 n2 − 2nk + 2k 2

olacaktır. Buna göre, n2 + 2nk + 2k 2 > k > 1 ve n2 − 2nk + 2k 2 =


(n − k)2 + k 2 > k 2 > 1 olduğundan n4 + 4k 4 asal olamaz ve sonsuz
sayıda a değeri seçilebilir.

6. Eğer n çift bir sayı ise n4 +4n çift olacaktır. Eğer n tek sayı ve n = 2k +1
ise,

(2k + 1)4 + 4(2k+1) = (2k + 1)4 + 4 · 42k = (2k + 1)4 + 4 (2)4k

olacaktır. Eğer (2k + 1) = a ve 2k = b olarak alırsak, ifademiz


¡ ¢¡ ¢
a4 + 4b4 = a2 + 2ab + 2b2 a2 − 2ab + 2b2

olacaktır. Buna göre ifademiz,


³ ´³ ´
(2k + 1)2 + 2 (2k + 1) 2k + 2 · 22k (2k + 1)2 − 2 (2k + 1) 2k + 2 · 22k
28 BÖLÜM 2. KONULAR

olacaktır. Buradan da,


³ ´³ ´
n2 + 2k+1 n + 22k+1 n2 − 2k+1 n + 22k+1

olacaktır. Eğer n > 1 ise her iki çarpanda 1’den büyük olacaktır. Dola-
yısıyla n = 1 durumu tek asal olduğu durumdur.
¡ ¢ ¡ ¢4 ¡ ¢4
7. P x4 = x16 + 6x8 − 4x4 + 1 = x4 − 1 + 4 x3 olacaktır. Buna göre,
A = x4 − 1 ve B = x3 olarak alırsak P (x4 ) polinomu
h¡ ¢2 ¡ ¢ ¡ ¢2 i h¡ 4 ¢2 ¡ ¢ ¡ ¢2 i
x4 − 1 + 2 x4 − 1 x3 + 2 x3 x − 1 − 2 x4 − 1 x3 + 2 x3

olur. İspat tamamlanır.

8. Sophie Germain özdeşliğini kullanırsak


¡ ¢¡ ¢
n12 + 64 = n6 − 4n3 + 8 n6 + 4n3 + 8
¡ 4 ¢¡ ¢
olarak yazabiliriz. Diğer yandan, ¡n12 +64¢ifadesi
¡ n + 4 ¢n¡8 − 4n4 + 16¢
biçiminde de yazılabilir. Burada
¡ n4 + 4 =¢ ¡ n2 − 2n + 2¢ ¡ n2 + 2n + 2 ¢
olduğuna göre ifademiz n12 = n2 − 2n + 2 n2 + 2n + 2 n8 − 4n4 + 16
olacaktır. Şimdi de (n6 − 4n3 + 8) ve (n6 + 4n3 + 8) çarpanlarını incele-
yelim. Buna göre,
¡ ¢¡ ¢
n6 + 4n3 + 8 = n2 − 2n + 2 n4 + 2n3 + 2n2 + 4n + 4

ve ¡ ¢¡ ¢
n6 − 4n3 + 8 = n2 + 2n + 2 n4 − 2n3 + 2n2 − 4n + 4
olacağına göre
¡ ¢¡ ¢¡ ¢¡ ¢
n12 +64 = n2 − 2n + 2 n2 + 2n + 2 n4 + 2n3 + 2n2 + 4n + 4 n4 − 2n3 + 2n2 − 4n + 4

olacaktır. Çarpanlardan herbiri artan fonksiyonlar olduğuna göre, hepsi


birbirinden farklıdır.

9. Geometrik dizi toplamını kullanırsak,


m n µ ¶ ¡ 4 ¢m+1 ¡ m+1 ¢4 ¡ ¢4
X k
X 4 k n + 4m+1 n + 4 2m/2
4(m−k) 4m
(−4) n =n − = =
n n4 + 4 n4 + 4
k=0 k=0
2.3. SOPHİE GERMAİN ÖZDEŞLİĞİ 29

eşitliğini elde edebiliriz. Sophie Germain özdeşliğini kullanarak kesrin


pay kısmı çarpanlarına ayrılabilir. Buna göre ifade,
³ m
´³ m
´
n2(m+1) + 2 2 +1 · nm+1 + 2m+1 n2(m+1) − 2 2 +1 · nm+1 + 2m+1

olacaktır. m ≥ 2 olduğundan, payda kısmı paydaki iki çarpandan da kü-


çük olacaktır. Sadeleştirmelerden sonra bile, kalan sayı hala 1’den büyük
iki sayının çarpımı olacaktır. Buna göre, ispat tamamlanır.

10. Çözüme başlamadan önce Einstein Kriteri’ni bir okuyalım.


Einstein Kriteri. P (x) = an xn + an−1 xn−1 + · · · + a0 , tamsayı kat-
sayılı bir polinom veriliyor. Varsayalım p asal sayı olmak üzere, p -
an , p|ak , k = 1, 2, · · · , n − 1 ve p2 - a0 olsun. Buna göre, P (x) poli-
nomu Z[x] üzerinde indirgenebilir.
Biz en küçük değerin 16 olduğunu göstereceğiz, ancak ilk 15 değerin ol-
madığını kanıtlamamız gerekiyor. Bunun içinde Einstein Kriteri’ni kul-
lanacağız.
n = 10, 11, 12, 13, 14, 15 için kriterimiz 5, 11, 3, 13, 7 ve 5 asallarını uygu-
layarak kontrol edebiliriz.
n = 8 veya 9 durumunda, eğer polinomumuz istenilen gibi çarpanlarına
ayrılabiliyorsa, çarpanlar lineer olmalıdır. Ancak kontrol edilebilirki bu
polinomların tamsayı kökleri olmayacaktır. Demek ki, n ≥ 16 olmalıdır.
Bu durumda da, zaten sorumuz (8.) örmekte olduğu gibi çözülebilir.

Sophie Germain2 (1776-1831)


Babası zengin bir ipek tüccarıdır. Toplumda liberal reformların konuşulup
planlandığı bir ailenin kızı olarak 1776 yılında doğar. Daha 13 yaşındayken
Arşimet’in ölüm hikayesini okuduktan sonra matematikçi olmaya karar ve-
rir.Kendi kendine Latince ve Yunanca öğrenir. Ailesinin muhalefetine rağmen,
anne ve babası uyduktan sonra Newton ve Euler’i okur. Felsefeye merak sarar.
Bu kadar inatçı bir çocukla baş edemeyen babası sonunda Sophie’yi hayatı
boyunca desteklemeye karar verir. Matematikteki zekasını ilk kez meşhur ma-
tematikçi Lagrange keşfeder. Lagrange için hazırladığı bir ödevi kadın oldu-
ğundan önem verilmeyeceği kaygısıyla “M. LeBlanc” diye sahte bir isimle verir.
2
Hürriyet gazetesi yazarı, Erkan Kumcu’nun 9 Şubat 2003 tarihli yazısından alınmıştır.
30 BÖLÜM 2. KONULAR

Lagrange bu dehanın Sophie Germain olduğunu daha sonra öğrenir. Sophie’nin


matematik alanında en büyük destekçilerinden biri Lagrange olur. Sophie Ger-
main’i en çok etkileyen matematikçilerden biri de çoğu kesimlerin fikir birliği
içinde matematiğin prensi diye adlandırılan Gauss oldu. Ona da çeşitli mate-
matik konularında bir çok mektup yazdı. Aynı kaygıyla, mektuplarına uzun
süre M. LeBlanc olarak imza attı.
Gauss, M. LeBlanc’ın Sophie Germain olduğunu
Fransızlar Gauss’un oturduğu şehri işgal edip Sop-
hie’nin aile dostu olan bir Fransız generalden Ga-
uss için ayrıcalık istediğinde öğrenir. Sophie Ger-
main’in matematikteki meşhur Fermat Teoremi’nin
çözümüne yaptığı katkılar bilinen en iyi yönüdür.
Yaptığı katkıların önemi kendinden ancak 100 yıl
sonra Kummer tarafından bir adım ileri götürüle-
bildiği düşünülürse daha iyi anlaşılır. Zamanın çok
prestijli yarışmalarına katılmıştır. Poisson gibi mate-
matik ve istatistiğin önde gelen isimleriyle yarışmış-
tır. Başarılı olamamıştır. Hak ettiği dereceler hiçbir
zaman kendine verilmemiştir. Geçmişte M. LeBlanc
ismini kullanmakla ne kadar haklı olduğunu tüm matematik dünyası adete
Sophie’ye ispat etmiştir.Poisson, Gaspard de Prony ve Laplace’dan oluşan bir
jürinin seçiciliğinde katıldığı bir yarışmada sunduğu makale bazı teknik hatalar
nedeniyle kabul dahi edilmemiş ve kendisine çalışmasının neden kabul edilme-
diği söylenmemiştir bile. Olaydan 55 yıl sonra Gaspard de Prony’nin yazdığı
makalelerinden birinin Sophie Germain’in yazdığı makalenin düzeltilmiş şekli
olduğu anlaşılmıştır.Bir çok deha gibi, Sophie Germain de çok genç yaşta öldü.

2.4 Tamkareler Pozı̇tı̇ftı̇r


Bu bölümde, cebirde oldukça basit bir eşitsizlik olan

x2 ≥ 0, ∀x ∈ R

eşitsizliğinin bazı uygulamalarını yapacağız. Eşitlik durumu yanlız ve yanlız


x = 0 durumunda vardır. Şimdi örneklerle konuyu kavramaya çalışalım.
2.4. TAMKARELER POZİTİFTİR 31

Örnek. 4x − x4 ≤ 3, x ∈ R eşitsizliğini kanıtlayınız.

Çözüm. Bu örnek eşitsizlikleri öğrenmeye başlayan genç bir öğrenci


tarafından ortaya atılmıştır. Henüz herhangi bir analiz kitabından eşitsizlikler
konusunu çalışmadığını düşünürsek fena bir örnek değildir. Buna göre,

x4 − 4x + 3 > 0 ⇒ x4 − 2x2 + 1 + 2x2 − 4x + 2 > 0

olacaktır. Buradan,
¡ 2 ¢2
x − 1 + 2 (x − 1)2 > 0
olacaktır ki, ispat tamamlanır.
¡ ¢
Örnek. f x2 −(f (x))2 > 1/4 eşitsizliğini sağlayan f : R → R ve birebir
olan tüm f fonksiyonlarını bulunuz.

Çözüm. x = 0 alırsak, f (0) − (f (0))2 ≥ 1/4 ve (f (0) − 1/2)2 6 0


olacaktır ki, bu durum imkansızdır ancak f (0) = 1/2 olabilir. Ancak x = 1
alırsak bu seferde f (1) = 1/2 olacaktır. Ancak bu durum f fonksiyonun birebir
olması durumu ile çelişir.
Şimdi alıştırmaları çözerek konuyu daha iyi kavramaya çalışınız. Alıştır-
malardaki yöntemlere benzer yöntemleri uygulayarak çözüme gitmeye gayret
ediniz.

2.4.1 Çalışma Soruları


1. n tane reel sayının ikişerli çarpımları toplamı ve toplamları sıfırdır. Buna
göre, bu sayıların küpleri toplamınında sıfır olduğunu kanıtlayınız.

2. a, b, c, d birer reel sayı olmak üzere, a−b2 , b−c2 , c−d2 ve d−a2 sayılarının
tümünün birden 1/4’ten büyük olamayacağını kanıtlayınız.

3. x, y, z pozitif reel sayılarının hepsi 4 ten küçüktür. Buna göre,


1 1 1 1 1 1
+ , + , +
x 4−y y 4−x z 4−x
sayılarından en az bir tanesinin 1’den büyük veya eşit olduğunu kanıtla-
yınız.
32 BÖLÜM 2. KONULAR

4. Aşağıdaki denklem sisteminin tüm reel çözümlerini bulunuz.



x+y = 4z − 1

y+z = 4x − 1
p
z+x = 4y − 1

5. x, t ∈ (0, 1) olmak üzere, öyle bir a 6= 1 pozitif sayısı vardır ki,

logx a + logy a = 4 logxy a

eşitliği sağlanmaktadır. Buna göre, x = y olduğunu kanıtlayınız.

6.
x4 + y 4 + z 4 − 4xyz = −1
eşitliğini sağlayan tüm (x, y, z) reel üçlülerini bulunuz.

7.

2xy − z 2 ≥ 1
z − |x + y| ≥ −1

denklem sistemini sağlayan tüm (x, y, z) reel üçlülerini bulunuz.

8. x4 + ax3 + 2x2 + bx + 1 denkleminin reel çözümü yoksa a2 + b2 ≥ 8


olacağını gösteriniz.

9. a, b ve c reel sayılar olmak üzere a2 + c2 > 4b olarak veriliyor. Buna göre


x ∈ R olmak üzere

x4 + ax3 + bx2 + cx + 1 > 0

eşitsizliğini kanıtlayınız.

10.
3
x2 + y 2 + z 2 − xy − yz − xz > (x − y)2
4
eşitsizliğinin tüm (x, y, z) reel değerleri içi sağlandığını kanıtlayınız.
2.4. TAMKARELER POZİTİFTİR 33

11.
p p p 1
x1 − 12 + 2 x2 − 22 + · · · · · · + n xn − n2 = (x1 + x2 + · · · + xn )
2

eşitliğini sağlayan tüm (x1 , x2 , · · · , xn ) reel sayılarını bulunuz.

12. a. a, b, c negatif olmayan reel sayılardır. Buna göre,


p
ab + bc + ca > 3abc (a + b + c)

eşitsizliğini kanıtlayınız.
b. a, b, c negatif olmayan reel sayılardır, öyleki a + b + c = 1 veriliyor.
Buna göre,

a2 + b2 + c2 + 12abc 6 1

eşitsizliğini kanıtlayınız.

13. k, m, n ∈ R olmak üzere verilen

f (km) + f (kn) − f (k) · f (mn) > 1

eşitsizliğini sağlayan tüm f : N → R fonksiyonlarını bulunuz.

14. Bir dik paralelyüzün kenar uzunlukları a, b, c ve köşegen uzunluğu d ise



a2 b2 + b2 c2 + c2 a2 > abcd 3

eşitsizliğini kanıtlayınız.

15. a1 , a2 , · · · , an reel sayılar olmak üzere


n X
X n
i · j · cos (ai − aj ) > 0
i=1 j=1

eşitsizliğini kanıtlayınız.
34 BÖLÜM 2. KONULAR

2.4.2 Çözümler
1. Varsayalım sayılarımız a1 , a2 , · · · , an olsun. Soruda verilen şartlara göre,

a21 +a22 +· · ·+a2n = (a1 + a2 + · · · + an )2 −2 (a1 a2 + a1 a3 + · · · + an−1 an ) = 0

olduğuna göre, a1 = a2 = · · · = an = 0 olacağına göre,a31 +a32 +· · ·+a3n =


0 olacaktır.

2. Varsayalım tümü birden,


1 1 1 1
a − b2 > , b − c2 > , c − d2 > , d − a2 >
4 4 4 4
olsun. Eğer bu dört eşitsizliği toplarsak,
¡ ¢
a + b + c + d − a2 + b2 + c2 + d2 > 1

olacaktır. Buradan,
µ ¶2 µ ¶2 µ ¶2 µ ¶2
1 1 1 1
−a + −b + −c + −d <0
2 2 2 2
olacaktır ki, bu durum imkansızdır.

3. Soruya göre, varsayalım hepsi birden 1’den küçük olsun. Buna göre,
µ ¶ µ ¶ µ ¶
1 1 1 1 1 1
+ + + + + <3
x 4−x y 4−y z 4−z
olur. Diğer taraftan
µ ¶
1 1
+ > 1, a < 4
a 4−a

olacağından (a − 2)2 ≥ 2 olacaktır. İspat tamamlanır.

4. Çözüm I. Bir önceki soruda olduğu gibi, üç denklemide taraf tarafa
toplayıp, kareler tolamını elde etmeye çalışalım. Buna göre,
√ p √
2x + 2y + 2z − 4x − 1 − 4y − 1 − 4z − 1 = 0
2.4. TAMKARELER POZİTİFTİR 35

olacaktır. Bu eşitliği öyle kareler toplamı şeklinde yazalım ki, bunlardan


biri x, biri y ve biride z değişkenine bağlı olsun. Eğer tüm denklemi 2 ile
bölersek elimizde r
1
x− x−
4
oluşacaktır. Eğer bu denkleme 1/4 ekler çıkarırsak
r Ãr !2
1 1 1 1 1
x− − x− + = x− −
4 4 4 4 2

oluşacaktır. Şimdi bu yöntemi diğerleri içi uygularsak,


Ãr !2 Ãr !2 Ãr !2
1 1 1 1 1 1
x− − + y− − + z− − =0
4 2 4 2 4 2

olacaktır. Buna göre, x = y = z = 1/2 denklem sisteminin tek çözümü


olarak bulunur.

Çözüm II. Eğer denklemlerin karelerini alıp toplarsak,


³ ´ ³ ´ ³ ´
2 2 2
0 = (x + y) − 4z + 1 + (y + z) − 4x + 1 + (z + x) − 4y + 1
= (x + y − 1)2 + (y + z − 1)2 + (z + x − 1)2

olacağından x + y = y + z = z + x = 1 veya x = y = z = 1/2 bulunur.

5. a > 0 ve a 6= 1 olduğuna göre eşitlik


1 1 4
+ =
loga x loga y loga xy
veya
loga x + loga y 4
=
loga x loga y loga x + loga y
olarak yazılabilir. Eğer içler dışlar çarpımı yapılırsa,

(loga x + loga y)2 = 4 loga x loga y


36 BÖLÜM 2. KONULAR

eşitliği elde edilir. Buradan da,

(loga x − loga y)2 = 0

olacağından x = y olduğu açıktır.

6. Eğer eşitliğin sol tarafını ilk iki terimi içeren bir tamkare olarak yazmaya
çalışırsak ¡ ¢2
−1 = x2 − y 2 + 2x2 y 2 + z 2 − 4xy
olacaktır. Bu noktadan sonra denkleme 2z 2 ekler ve çıkarırsak bir tam-
kare daha elde ederiz. Zaten bu basamaktan sonra yeni denklemimiz üç
karenin toplamı
¡ 2 ¢2 ¡ ¢2
x − y 2 + z 2 − 1 + 2 (xy − z)2 = 0

olacaktır. Eşitlik ancak hepsinin sıfır olması durumunda geçerlidir. Buna


göre, z = ±1 olacağından istenen üçlüler

(1, 1, 1) , (−1, −1, 1) , (−1, 1, −1) , (1, −1, −1)

olacaktır.

7. Çözüm I. ikinci denklemden z ≥ |x + y| − 1 olacağından, bunu ilk


denklemde yerine koyarsak

2xy − (1 − |x + y|)2 > 1

olacaktır. Buna göre,

2xy − (1 − |x + y|)2 = 2xy − |x + y|2 + 2 |x + y| − 1


= 2xy − x2 − y 2 − 2xy + 2 (±x ± y) − 1
= −x2 − y 2 + (±x ± y) − 1

eğer işaret seçimimizi uygularsak, yeni denklem aşağıdaki gibi olacaktır.


Buna göre,

0 > x2 + y 2 − 2 (±x ± y) + 1 + 1 = (1 ± x)2 + (1 ± y)2


2.4. TAMKARELER POZİTİFTİR 37

olacağından her iki karede sıfır olmalıdır. Yani x = y = ±1 değerlerini


alacaklardır. Birinci denklemden xy > 0 olacağına göre x ve y aynı
işaretli olmalıdır. Yani x = y = 1 veya x = y = −1 olmalıdır. Eğer
bu değerleri yerine koyarsak x = y = z = 1 ve x = y = 1, z = 1 çözümler
olacaktır.

Çözüm II. Eğer ikinci denklemi z + 1 ≥ |x + y| olarak yazıp karesini


alırsak
(x + y)2 ≥ (x + y)2
olacaktır. Birinci eşitsizliği 2 ile çarpıp son yazdığımız eşitsizliği toplarsak

0 ≥ (x − y)2 + (z − 1)2

olacaktır. Demek ki x = y ve z = 1 olmalıdır. Eğer bu eşitlikleri yerine


koyarsak 2x2 ≥ 2 ve 2 ≥ 2|x| ise |x| = 1 olmalıdır.

8.
³ µ ¶
a ´2 b 2 1¡ ¢
x4 + ax3 + 2x2 + bx + 1 = x2 + x + 1 + x + 8 − a2 − b2 x2
2 2 4

eşitliğinden bu ifadenin pozitif olduğunu söyleyebiliriz. Ancak, 8 − a2 −


b2 ≤ 0 ise yani a2 + b2 ≥ 8 olursa durum bozulur.

9.
³ µ ¶
4 3 2 2 a ´2 a2 + c2 ³c ´2
x + ax + bx + cx + 1 = x + x + b − x2 + x+1
2 4 2

olduğuna ve b ≥ (a2 + c2 )/4 ise ifade daima sıfırdan büyük eşittir.

10. Eğer eşitsizliği düzenlersek


1h i 3
(x − y)2 + (y − z)2 + (z − x)2 > (x − y)2
2 4
olacağından h i
2 (y − z)2 + (z − x)2 > (x − y)2
38 BÖLÜM 2. KONULAR

olacaktır. Eğer a = y − z ve b = z − x alırsak


¡ ¢
2 a2 + b2 > (a + b)2

olur ki, doğrudur, ispat tamamlanır.

11. Yine bu sorudada amacımız, verilen denklemi kareler toplamı biçiminde


yazmak olacaktır.Buna göre denklemi 2 ile genişletip, hepsini sol tarafa
toplarsak
√ p p
(x1 + x2 + · · · + xn ) − 2 x1 − 1 − 4 x2 − 22 − · · · − 2n xn − n2

ifadesini elde ederiz. Buna göre,


¡ √ ¢ ³ p ´ ³ p ´
x1 − 1 − 2 x1 − 1 + 1 + x2 − 22 − 4 x2 − 22 + 22 +· · ·+ xn − n2 − 2n xn − n2 + n2

ifadesi elde edilir. Eğer bu ifade de kareler toplamı olarak yazılırsa


¡√ ¢2 ³p ´2 ³p ´2
x1 − 1 − 1 + x2 − 22 − 2 + · · · + xn − n2 − n

ifadesi elde edilir. Eğer dikkat edilirse tüm toplamın sıfır olabilmesi için
tamkare ifadelerin de sıfır olması gerektiğini görmek zor değildir. Buna
göre,
x1 = 2, x2 = 8, · · · , xn = 2n2
olacaktır.

12. a. Eşitsizliğin iki tarafınında karesini alırsak

a2 b2 + b2 c2 + c2 a2 + 2abc (a + b + c) > 3abc (a + b + c)

olduğuna göre,

1h i
(ab − bc)2 + (bc − ca)2 + (ca − ab)2 > 0
2

olacaktır. İspat tamamlanır.


2.4. TAMKARELER POZİTİFTİR 39

b. a + b + c = 1 ise soruda verilen eşitsizliği,


p ¡ ¢
12abc (a + b + c) 6 2 (a + b + c)2 − a2 + b2 + c2

olarak yazabiliriz. Buna göre eşitsizliğimiz


p
12abc (a + b + c) 6 2 (ab + bc + ac)

olacaktır. Zaten benzer eşitsizliği (a.) şıkkında çözmüştük. Aynı


yöntemle siz tamamlayabilirsiniz.

13. Bu bölümün konu anlatım bölümünde çözdüğümğz örnekte olduğu gibi


buradada m, n ve k için bazı değerleri kontrol edeceğiz. Eğer m = n =
k = 0 alırsak
2f (0) − f 2 (0) > 1

olacağından 0 > (f (0) − 1)2 olacaktırki, buradan f (0) = 1 olarak bulu-


nur. Eğer m = n = k = 1 alırsak, bu seferde f (1) = 1 olacaktır. Eğer
m = n = 0 alınırsa 2f (k) ≥ 1 olacağından f (k) ≤ 1 olarak bulunur.
Benzer biçimde k = 1 ve m = 0 alırsak 1 + f (n) − 1 ≥ 1 ise f (n) ≥ 1
olacaktır. Demek ki fonksiyonumuz f (x) = 1 fonksiyonu olmalıdır.

14. Elimizde dik bir paralelyüz olduğuna göre


p
d= a2 + b2 + c2

olacağından eşitsizlik
¡ 2 2 ¢2 ¡ ¢
a b + b2 c2 + c2 a2 > 3a2 b2 c2 a2 + b2 + c2

olacaktır. Eğer parantezleri açar ve gruplama yaparsak,

c2 ¡ 2 ¢2 a4 ¡ 2 ¢2 b4 ¡ 2 ¢2
a − b2 + b − c2 + c − a2 > 0
2 2 2

olacağından soruda verilen eşitsizlik kanıtlanmış olur.


40 BÖLÜM 2. KONULAR

15. Kosinüs için fark formülünü kullanırsak,


n X
X n n X
X n
ij cos (ai − aj ) = ij (cos ai cos aj + ij sin ai sin aj )
i=1 j=1 i=1 j=1
Xn n
X n
X n
X
= i cos ai j cos aj + i sin ai j sin aj
i=1 j=1 i=1 j=1
à n
!2 Ã n
!2
X X
= i cos ai + j sin aj >0
i=1 i=1

olduğundan ispat tamamlanmış olur.

2.5 Eşı̇tsı̇zlı̇kler I
Eşitsizlikleri çözerken sıklıkla sayıları ve matematiksel ifadeleri karşılaştı-
rırız. Yada bize verilen bir matematiksel ifadenin en büyük yada en küçük
değerini bulmaya çalışırız. Bu ders notumuzda eşitsizlik sorularında sıklıkla
karşımıza çıkan temel tip eşitsizlikleri ve uygulamalarını göreceğiz.
Aritmetik Orta-Geometrik Orta-Harmonik Orta Eşitsizliği. a1 , a2 , · · · , an >
0 ve ai ∈ R için,
a1 + · · · + an √ n
AO = ≥ GO = n a1 · a2 · · · an ≥ HO = 1 1 1
n a1 + a2 + ··· + an

olacaktır. Eşitlik olması durumunda ise a1 = a2 = · · · = an olacaktır.


Örnek.(1)A0 − GO eşitsiliği ile, x > 0 olmak üzere,
r
1 1
x+ ≥2 x· =2
x x
olacaktır. Eşitlik ise x = 1 durumunda sağlanacaktır.
(2) AO − HO eşitsizliğinden eğer a1 , a2 , · · · , an > 0 ise
1 1 1
(a1 + a2 + · · · + an )( + + · · · + ) ≥ n2
a1 a2 an
2.5. EŞİTSİZLİKLER I 41

olacaktır.
(3) a, b, c > 0 ve abc = 1 ise (a + b + c)(ab + cb + ac) çarpımının en küçük
değerini bulunuz.
Çözüm.AO − GO eşitsizliğinden,

a+b+c √
3 ab + bc + ac √
3
≥ abc ve ≥ ab · bc · ac
3 3
ise
(a + b + c)(ab + ac + bc) ≥ 9
olacaktır. Buna göre istenen en küçük değer 9 olacaktır.
(4) n ∈ Z+ ise
1 n 1 n+1
(1 + ) ≤ (1 + )
n n+1
eşitsizliğini kanıtlayınız.
1
Çözüm. Varsayalım a1 = a2 = · · · = an = 1 + n ve an+1 = 1 olarak alalım.
Buna göre, AO − GO eşitsizliğinden
r
n(1 + n1 ) + 1 1 n+1 1 n
AO = =1+ ≥ GO = (1 + ) ·1
n+1 n+1 n
eşitsizliğinden soruda istenen durum elde edilir.
(5)(1964, IMO) a, b, c bir üçgenin kenar uzunluklarıdır. Buna göre,

a2 (b + c − a) + b2 (c + a − b) + c2 (a + b − c) ≤ 3abc

eşitizliğini kanıtlayınız.
Çözüm. Varsayalım,

a+b−c b+c−a c+a−b


x= , y= , z=
3 2 2
olarak alalım. Buradan a = x+z, b = x+y, c = y +z olacağından eşitsizliğimiz

(x + z)2 2y + (x + y)2 2z + (y + z)2 2x ≤ 3(z + x)(x + y)(y + z)


42 BÖLÜM 2. KONULAR

olacaktır. Eğer son bulduğumuz son eşitsizliği düzenlersek,

x2 y + y 2 z + z 2 x + xy 2 + yz 2 + zx2 ≥ 6xyz

eşitsizliğini elde ederiz. Buradan eşitsizliğin sol tarafına AO − GO eşitsizliği


uygulanırsa soruda istenen eşitsizlik kanıtlanmış olur.
(6)(2008, AUMO3 ) n doğal sayısının kaç tane değeri için,

x1 + x2 + · · · + xn = 9
1 1 1
+ + ··· + = 1
x1 x2 xn
denklem siteminin pozitif reel sayılarda çözümü vardır?
Çözüm. AO − HO eşitsizliğini kullanırsak,
n x1 + x2 + · · · + xn n 9
1 1 1 ≤ ⇒ ≤ ⇒ n2 ≤ 9
x1 + x2 + ··· + xn
n 1 n

olduğuna göre, n = 1, 2, 3 değerlerini alabilir. Ancak, n = 1 değeri için doğru-


lanmadığı açıktır. Öyleyse sadece 2 ve 3 için çözümlüdür.

Cauchy-Schwartz Eşitsizliği. a1 , a2 , · · · , an ve b1 , b2 , · · · , bn reel sayıları


için

(a1 b1 + a2 b2 + · · · + an bn )2 ≤ (a21 + a22 + · · · + a2n )(b21 + b22 + · · · + b2n )

eşitsizliği vardır. Eşitlik durumu ai bj = ai bj , i, j = 1, · · · , n için vardır.


(7) 0 ≤ θ < 2π için
a · cos θ + b · sin θ
ifadesinin alabileceği en büyük ve en küçük değerleri bulunuz.
Çözüm. C.S. eşitsizliğinden

(a cos θ + b sin θ)2 ≤ (a2 + b2 )(cos2 θ + sin2 θ)


3
Antalya Üniversitesi Matematik Olimpiyatları, 2008
2.5. EŞİTSİZLİKLER I 43

eşitsizliğinden,
p p
− a2 + b2 ≤ a cos θ + b sin θ ≤ a2 + b2

aralığı bulunur.
(8)(1978, USAMO4 ) a, b, c, d, e reel sayıları için a + b + c + d + e = 8 ve
a2 + b2 + c2 + d2 + e2 = 16 ise, e’nin alabileceği en büyük değeri bulunuz.
Çözüm. C.S. eşitsizliğinden,

(a + b + c + d)2 ≤ (1 + 1 + 1 + 1)(a2 + b2 + c2 + d2 )

ise
(8 − e)2 ≤ 4(16 − e2 )
olacaktır. Buradan da, e(5e − 16) ≤ 0 ise 0 ≤ e ≤ 16/5 olacaktır. C.S. eşitsizli-
ğinin, eşitlik duruu kullanılırsa a = b = c = d = 6/5 ve emax = 16/5 olacaktır.
(9) a, b, c > 0 ve abc = 1 ise
1 1 1 3
+ + ≥
a3 (b + c) b3 (a + c) c3 (a + b) 2

eşitizliğini kanıtlayınız.
1 1 1
Çözüm. x = a = bc, y = b = ac, z = c = ab alırsak, eşitsizliğin son hali

x2 y2 z2 3
+ + ≥
z+y z+x x+y 2
olacaktır. Burada,
x √ y √ z √
x+y+z = √ z+y+ √ x+z+ √ x+y
z+y x+z x+y

eşitliğine C.S.E. eşitsizliğini uygularsak,

x2 y2 z2
(x + y + z)2 ≤ ( + + )((z + y) + (x + z) + (y + x))
z+y x+z x+y
4
United States Of America Math Olympiads, 1978
44 BÖLÜM 2. KONULAR

olacaktır. Son eşitsizliği ve A.O. − G.O. eşitsizliklerini kullanırsak,

x2 y2 z2 x+y+z 3√ 3
+ + ≥ ≥ 3 xyz =
z+y z+x x+y 3 2 2

bulunur.

Yeniden Düzenleme (Permütasyon) Eşitsizliği. a1 ≥ a2 ≥ · · · ≥ an ve


b1 ≥ b2 ≥ · · · ≥ bn ise,

a1 b1 +a2 b2 +· · ·+an bn ≥ a1 br1 +a2 br2 +· · ·+an brn ≥ a1 bn +a2 bn−1 +· · ·+an b1

eşitsizliği vardır. Burada (br1 , br2 , · · · , brn ) dizilimi (b1 , b2 , · · · , bn ) diziliminin


bir permütasyonudur.
(10)(1978, IMO5 ) c1 , c2 , · · · , cn farklı pozitif tamsayılardır. Buna göre,

c2 cn 1 1
c1 + 2
+ ··· + 2 ≥ 1 + + ··· +
2 n 2 n
eşitsizliğini kanıtlayınız.
Çözüm. Varsayalım (a1 , a2 , · · · , an ) dizilimi ci ’lerin artan sırayla dizilimi ol-
sun. ai ’ler farklı pozitif tamsayılar olduğuna göre, a1 ≥ 1, a2 ≥ 2,· · · , an ≥ n
diyebiliriz. Burada,
a1 < a2 < · · · < an
ve
1 1 1
1> 2
> 2 > ··· > 2
2 3 n
ise Y.D.E.’ne göre

c2 cn a2 an 2 n
c1 + 2
+ · · · + 2 ≥ a1 + 2 + · · · + 2 ≥ 1 + 2 + · · · + 2
2 n 2 n 2 n
olacaktır.
(11) Örnek (9)’u Y.D.E. kullanarak yapınız.
5
International Math Olympiads, 1978
2.5. EŞİTSİZLİKLER I 45

Çözüm. x, y, z için tanımlamalarımız (9)’daki gibi olsun. Genelliği kaybet-


meden x ≥ y ≥ z alalım. xyz = 1 ve x2 ≥ y 2 ≥ z 2 olduğuna göre 1/(z + y) ≥
1/(x + z) ≥ 1/(y + x) üçlüsüde ikinci dizilimimiz olsun. Bu noktada Y.D.E.’yi
iki defa uygularsak,

x2 y2 z2 x2 y2 z2
+ + ≥ + +
z+y z+x x+y x+y z+y x+z
x2 y2 z2 x2 y2 z2
+ + ≥ + +
z+y z+x x+y z+x x+y z+y

eşitsizliklerini taraf tarafa toplarsak,

x2 y2 z2 1 y 2 + x2 z 2 + y 2 z 2 + x2
+ + ≥ ( + + )
z+y z+x x+y 2 y+x y+z z+x

eşitsizliğini elde ederiz.


a2 + b2
a2 + b2 ≥
2
eşitsizliğini, sağ tarafın payları için kullandıktan sonra A.O.−G.O. eşitsizliğini
kullanırsak,

x2 y2 z2 1 y+x z+y x+z x+y+z 3√ 3


+ + ≥ ( + + )= ≥ 3 xyz =
z+y z+x x+y 2 2 2 2 2 2 2

eşitsizliği elde edilir.

Chebyshev Eşitsizliği. Eğer a1 ≥ a2 ≥ · · · ≥ an ve b1 ≥ b2 ≥ · · · ≥ bn ise

(a1 + · · · + an )(b1 + · · · + bn )
a1 b1 + a2 b2 + · · · + an bn ≥
n
eşitsizliği vardır.
(12)(1974, USAMO6 ) a, b, c > 0 ise

aa bb cc ≥ (abc)(a+b+c)/3
6
United States Of America Math Olympiads, 1974
46 BÖLÜM 2. KONULAR

olduğunu kanıtlayınız.
Çözüm. Üçlülerimizi (a, b, c) ve (log a, log b, log c) olarak seçersek,
1
a log a + b log b + c log c ≥ (a + b + c)(log a + log b + log c)
3
(a + b + c)
log aa bb cc ≥ log(abc)
3
log aa bb cc ≥ log(abc)(a+b+c)/3
aa bb cc ≥ (abc)(a+b+c)/3

olacaktır.
(13) 0 ≤ ak < 1, k = 1, 2, 3, · · · , n ve S = a1 + a2 + · · · + an ise
n
X ak nS

1 − ak n−S
k=1

eşitsizliğini kanıtlayınız.
Çözüm. Genelliği kaybetmeden a1 ≥ a2 ≥ · · · an ≥ 0 alabiliriz. Buna göre,
a1 a2 an
0 < 1 − a1 ≤ 1 − a2 ≤ · · · ≤ 1 − an ve ≥ ≥ ··· ≥
1 − a1 1 − a2 1 − an
olacaktır. Chebysev eşitsizliğinden,
a1 a2 an
S = (1 − a1 ) + (1 − a2 ) + · · · + (1 − an )
1 − a1 1 − a2 1 − an
n n n
1 X ak X n − S X ak
≤ (1 − ak ) = olacaktır.
n 1 − ak n 1 − ak
k=1 k=1 k=1

Matematikte ve tabii ki istatistikte sıklıla ortalamalrı kullanmaya ihtiyaç


duyarız. AO, GO, HO dışında kullandığımız Kuvvet Ortalaması ve Simetrik
Ortalar da vardır. Aslında, bu iki ortalama AO ve GO’yıda özel birer durum
olarak içerir.
2.5. EŞİTSİZLİKLER I 47

Kuvvet Ortalaması Eşitsizliği. a1 , a2 , a3 , · · · , an > 0 ve s < t için

as1 + as2 + · · · + asn 1/s at + at2 + · · · + atn 1/t


Ms = ( ) ≤ Mt = ( 1 )
n n
eşitsizlikleri vardır. Eşitlik a1 = a2 = · · · = an durumunda vardır.

Not. Bu eşitsizlikte M1 = A.O., M−1 = H.O. ve M2 ise Karesel Ortalamadır.


Karesel Orta istatistik ve fizikte kullanılır. Ayrıca, limitlerini alırsak, M+∞
ifadesi M AX = max{a1 , a2 , · · · , an } ve M0 ifadesi Geometrik Orta ve de M−∞
ifadesi M IN = min{a1 , a2 , · · · , an } olacaktır. Dolayısıyla elimizde,

M AX ≥ KO ≥ AO ≥ GO ≥ HO ≥ M IN

eşitsizliği oluşacaktır.

Maclaurin Simetrik Orta Eşitsizliği.a1 , a2 , · · · , an > 0 için


1/2
AO ≥ S1 ≥ S2 ≥ · · · ≥ Sn1/n = GO

olacaktır. Mesela, Sj ifadesine n = 4 için bakalım


a1 + a2 + a3 + a4
S1 =
4
a1 a2 + a1 a3 + a1 a4 + a2 a3 + a2 a4 + a3 a4
S2 =
6
a1 a2 a3 + a1 a2 a4 + a1 a3 a4 + a2 a3 a4
S3 =
4
S4 = a1 a2 a3 a4

olacaktır.
(14) x, y, z pozitif sayılar olduğuna göre,
s r s
x 2 y 2 z2
x5 + y 5 + z 5 ≤ x5 + y5 + z5
yz zx xy

eşitsizliğini kanıtlayınız.
48 BÖLÜM 2. KONULAR

√ √ √
Çözüm. Varsayalım a = x, b = y, c = z olsun. Buna göre eşitsizliğimiz,
a13 + b13 + c13
a10 + b10 + c10 ≤
abc
olacaktır. Buna göre, KOE’yi kullanırsak
a13 + b13 + c13 = 3M13
13 10
= 3M13 3
M13 10
≥ 3M10 M03 = (a10 + b10 + c10 )abc
olacaktır.
(15) a, b, c > 0 ise,
1 1 1 a8 + b8 + c8
+ + ≤
a b c a3 b3 c3
eşitsizliğini kanıtlayınız.
Çözüm. Eşitsizliği düzenlersek,
1 1 1
a8 + b8 + c8 ≥ a3 b3 c3 ( + + ) = (abc)2 (bc + ac + ab)
a b c
olacaktır. KOE ve SOE’yi kullanırsak,
a8 + b8 + c8 = 3M88 ≥ 3M18 = 3S18 = 3S16 S12
1/3 1/2
≥ (S3 )6 3(S2 )2 = (abc)2 (bc + ac + ab)
olacaktır.
(16) a1 , a2 , · · · , an ≥ 0 ve (1 + a1 )(1 + a2 ) · · · (1 + an ) = 2n ise,
a1 · a2 · · · · · an ≤ 1
olduğunu gösteriniz.
Çözüm. SO eşitsizliğinden,
2n = (1 + a1 )(1 + a2 ) · · · (1 + an )
µ ¶ µ ¶
n n
= 1 + nS1 + S2 + · · · + Sn−1 + Sn
2 n−1
µ ¶ µ ¶ n−1
1/n n 2/n n
≥ 1 + nSn + S + ··· + Sn n + Sn = (1 + Sn1/n )n
2 n n−1
1/n
ise 2 ≥ 1 + Sn ve 1 ≥ Sn = a1 · a2 · · · · · an olur.
2.5. EŞİTSİZLİKLER I 49

2.5.1 Çalışma Soruları


1. Konu anlatımında verilen tüm örnekleri çözümlerine bakmadan yapınız.
2. x1 , x2 , · · · , xn > 0 için
x21 x22 x2
+ + · · · + n ≥ x1 + x2 + · · · + xn
x2 x3 x1
eşitsizliğini kanıtlayınız.
3. 0 < a, b, c < 1 ve a + b + c = 2 ise

8(1 − a)(1 − b)(1 − c) ≤ abc

eşitsizliğini kanıtlayınız.
4. Eğer a, b, c, d > 0 ve c2 + d2 = (a2 + b2 )3 ise
a3 b3
+ ≥1
c d
eşitsizliğini kanıtlayınız.
5. a1 , a2 , · · · , an > 0 ve a1 + a2 + · · · + an = 1 ise
1 2 1 1
(a1 + ) + (a2 + )2 + · · · + (an + )2
a1 a2 an
ifadesinin en küçük değerini bulunuz.
6. Eğer a, b, c, d > 0 ve S = a2 + b2 + c2 + d2 ise
a3 + b3 + c3 a3 + b3 + d3 a3 + c3 + d3 b3 + c3 + d3
+ + + ≥S
a+b+c a+b+d a+c+d b+c+d
eşitsizliğini kanıtlayınız.
7. Eğer x1 , x2 , · · · , xn > 0 ve x1 + x2 + · · · + xn = 1 ise,
n
X Xn
x 1 √
√ k ≥ √ xk
1 − xk n − 1 k=1
k=1

eşitsizliğini kanıtlayınız.
50 BÖLÜM 2. KONULAR

8. a, b, c bir üçgenin kenar uzunlukları olduğuna göre,

a2 b(a − b) + b2 c(b − c) + c2 a(c − a) ≥ 0

eşitsizliğini kanıtlayınız.

2.6 Eşı̇tsı̇zlı̇kler II
Bazı fonksiyon eşitsizliklerini kanıtını yaparken, fonksiyonunun belli ara-
lıklardaki şeklide önemlidir. Bu ders notumuzda ele aldığımız eşitsizliklerin
çözümleride bu temel prensiplere uyularak yapılmıştır.

Isınma
Tanım [Konveks vs. Konkav]. I aralığı üzerinde süreli bir f fonksiyonu,
x1 + x2 f (x1 ) + f (x2 )
f( )≤ x1 , x2 ∈ I
2 2
eşitsizliğini sağlıyorsa f fonksiyonuna konveks denir. Eğer I aralığı üzerinde
f fonksiyonu konveks ve eşitlik hali x1 = x2 oluyorsa, f tam konveks olur. I
aralığında −f konveks ise f fonksiyonu konkav olur. Bu durumda,
x1 + x2 f (x1 ) + f (x2 )
f( )≥ x1 , x2 ∈ I
2 2
olacaktır. Benzer biçimde eğer −f tam konveks ise f fonksiyonuda konveks
olur.
Tanım [İkinci Türev Testi]. Eğer I = (a, b) aralığında f 00 (x) ≥ 0 oluyorsa,
f fonksiyonu konvekstir. Eğer f 00 (x) > 0 oluyorsa, f fonksiyonu tam konveks
olur. Konkav ve tam konkav içinde tanım benzer biçimdedir. Sadece eşitsizlik
yön değiştirir. Bir fonksiyonun konveksiliğini göstermek için sınır noktalarını
içeren bir aralıkta ve bu aralıkta sürekli olması ile ikinci türev testinin negatif
olmaması yeterlidir. İkinci türev testini kullanarak, aşağıda verilen fonksiyo-
ların tam konvex olduğunu söyleyebiliriz.

xp ∈ [0, ∞) , p > 1 yada xp ∈ (0, ∞), p < 0


ax ∈ (−∞, ∞), a > 1 yada tan x ∈ [0, π/2)
2.6. EŞİTSİZLİKLER II 51

Benzer biçimde aşağıda verilen fonksiyonlarda tam konkavdır.


xp ∈ [0, ∞) , 0 < p < 1 yada loga x ∈ (0, ∞), a > 1
cos x ∈ [−π/2, π/2), yada sin x ∈ [0, π/2π)
Bu noktada, konveksliği ve konkavlığı eşitsizlik sorularında kullanabileceğimiz
en güzel yer sanırız Jensen Eşitsizliği’dir.

Mevzu
Tanım [Jensen Eşitsizliği]. f fonksiyonu I üzerinde konveks ve x1 , x2 , · · · , xn ∈
I ise µ ¶
x1 + x2 + x3 f (x1 ) + f (x2 ) + · · · + f (xn )
f ≤
n n
olacaktır. Burada eşitlik durumu ancak ve ancak x1 = x2 = · · · = xn eşitliğinde
olur.
Tanım [Genelleştirilmiş Jensen Eşitsizliği]. f konveks ve I aralığında
sürekli olmak üzere, x1 , x2 , · · · , xn ∈ I ve 0 < t1 , t2 , · · · , tn < 1, t1 + t2 + · · · +
tn = 1 ise,
f (t1 x1 + t2 x2 + · · · + tn xn ) ≤ t1 f (x1 ) + t2 f (x2 ) + · · · + tn f (xn )

Şekil 2.1: Aralıkta Bir Konveks Fonksiyon


52 BÖLÜM 2. KONULAR

olacaktır. Konkav fonksiyonlarda ise eşitsizlik yön değiştirir.

Egzersiz
[1.] Bir ABC üçgeninde,

3 3
sin A + sin B + sin C ≤
2
eşitsizliğini gösteriniz. Eşitlik durumu hangi durumda ortaya çıkar, açıklayınız.
Çözüm. f (x) = sin x fonksiyonu [0, π] arasında konkavdır. Öyleyse,
µ ¶ µ ¶ √
A+B+C A+B+C 3 3
sin A+sin B+sin C = f (A)+f (B)+f (C) ≤ 3f = 3 sin =
3 3 2
olacaktır. Eşitlik durumu ancak ve ancak A = B = C = π/3 yani ABC bir
eşkenar üçgense gerçekleşir.
[2.] a, b, c > 0 ve a + b + c = 1 ise,
1 1 1
(a + )10 + (b + )10 + (c + )10
a b c
ifadesinin en küçük değerini bulunuz.
Çözüm. 0 < a, b, c < 1 olarak zaten verilmiş. f (x) = (x+ x1 )10 ise f fonksiyonu
I = (0, 1) aralığında konveksdir. Çünkü,
1 8 1 1 2
f 00 (x) = 90(a + ) (x − 2 )2 + 10(x + )9 ( 3 ) > 0 olacaktır.
x x x x
Öyleyse JE’den
1 1 1 a+b+c 1 1010
f (a)+f (b)+f (c) = (a+ )10 +(b+ )10 +(c+ )10 ≥ 3f ( ) = 3f ( ) = 9
a b c 3 3 3
olarak bulunur.
Çözüm.(Alternatif Yöntem) Soruda verilen eşitsizliği çözmenin bir diğer
yöntemide Chebychev Eşitsizliğini kullanmak olabilirdi. Bu yöntemi size bıra-
kıyoruz.7
7
Chebychev Eşitsizliği ile alakalı olarak www.sbelian wordpress.com adresinden Ye-
niden Düzenleme Eşitsizliği Ders Notları’nı indirebilirsiniz.
2.6. EŞİTSİZLİKLER II 53

[3.] Aritmetik Orta - Geometrik Orta eşitsizliği JE kullanarak göstermeye ça-


lışalım. Buna göre, eğer a1 , a2 , a3 , · · · , an ise f (x) = log x’de (0, ∞) aralığında
konkav olduğuna göre,

a1 + a2 + · · · + an log a1 + log a2 + · · · + log an √


log( )≥ = log( n a1 a2 · · · an )
n n
ise istenen eşitsizlik kanıtlanmış olur.
1 1
[4.](Hölder) p, q > 1, p + q = 1 ve a1 , a2 , · · · , an b1 , b2 , · · · , bn reel sayılarsa,

¯ n ¯ à n !1/p à n !1/q
¯X ¯ X X
¯ ¯ p q
¯ ai bi ¯ ≤ |ai | |bi |
¯ ¯
i=1 i=1 i=1

eşitsizliğini kanıtlayınız.
Çözüm. Varsayalım
n
X n
X
p
A= |ai | ve B = |bi |q
i=1 i=1

olsun. Eğer A veya B sıfır ise, ya tüm ai ’ler yada tüm bi ’ler sıfırdır. Bu da zaten
eşitsizliğin iki tarafınıda sıfır yapar. Buna göre biz A 6= 0 ve B 6= 0 durumunu
inceleyelim. Varsayalım t1 = p1 ve t2 = 1q olsun. Öyleyse, 0 < t1 , t2 < 1 ve
t1 + t2 = 1 olacaktır. Eğer

|ai |p |bi |q
xi = ve yi =
A B
ise
n
X n
X
xi = 1 ve yi = 1
i=1 i=1

olur. f (x) = ex fonksiyonu (−∞, +∞) aralığında konveks olduğundan Genel-


leştirilmiş Jensen Eşitsizliğini kullanabiliriz. Buna göre,

1/p 1/q xi yi
xi · yi = f (t1 ln xi + t2 ln yi ) ≤ t1 f (ln x1 ) + t2 f (ln yi ) = +
p q
54 BÖLÜM 2. KONULAR

olacaktır. Buna göre,

Xn n n
|ai ||bi | 1X 1X
≤ x i + yi = 1
i=1
A1/p B 1/q p
i=1
q
i=1

olacaktır. Bundan dolayıda,


¯ n ¯
¯X ¯ X n ³X ´1/p ³X ´1/q
¯ ¯
¯ ai bi ¯ ≤ |ai ||bi | ≤ A1/p B 1/q = |ai |p |bi |q bulunur
¯ ¯
i=1 i=1

Şimdi de, Jensen eşitsiliğinin bir başka uygulamasına geçelim.


Tanım (Majorization8 ). Eğer x1 , · · · , xn ve y1 , · · · , yn aşağıdaki şartları
sağlıyorsa, yani

x1 ≥ x2 ≥ · · · ≥ xn , y1 ≥ y2 ≥ · · · ≥ yn

x1 ≥ y1 , x1 + x2 ≥ y1 + y2 , · · · , x1 + x2 + · · · + xn−1 ≥ y1 + y2 + · · · + yn−1
ve
x1 + x2 + · · · + xn = y1 + y2 + · · · + yn
ise (x1 , x2 , · · · , xn ) majorize (y1 , y2 , · · · , yn ) denir ve

(x1 , x2 , · · · , xn ) Â y1 , y2 , · · · , yn )

ile gösterilir.
Tanım (Majorization Eşitsizliği). I = [a, b] aralığında f fonksiyonu kon-
veks ve
(x1 , x2 , · · · , xn ) Â y1 , y2 , · · · , yn ), xi , yi ∈ I
ise
f (x1 ) + f (x2 ) + · · · + f (xn ) ≥ f (y1 ) + f (y2 ) + · · · + f (yn )
olur. Yanlız ve yanlız xi = yi durumu için eşitlik vardır. Konkav fonksiyonlar
içinse eşitsizlik yön değiştirir.
8
http://en.wikipedia.org/wiki/Majorization
2.6. EŞİTSİZLİKLER II 55

[5.] Dar açılı bir ABC üçgeni için,


3
1 ≤ cos A + cos B + cos C ≤
2
eşitsizliğini kanıtlayınız.
Çözüm. Genelliği kaybetmeden, varsayalım A ≥ B ≥ C olsun. Buna göre
A ≥ π/3 ve C ≤ π/3 olacaktır. π/2 ≥ A ≥ π/3, π ≥ A + B = (−π − C) ≥ 2π 3
olacağından
π π π π π
( , , 0) Â (A, B, C) Â ( , , )
2 2 3 3 3
alabiliriz. f (x) = cos x fonksiyonu [0, π/2] aralığında konkav olduğuna göre,
majorization teoreminden,
π π π π π
1 = f ( ) + f ( ) + f (0) ≤ f (A) + f (B) + f (C) ≤ f ( ) + f ( ) + f ( )
2 2 3 3 3
ise soruda göstermemiz istenen,
3
1 ≤ cos A + cos B + cos C ≤
2
elde edilir.
[6.] Eğer x1 ≥ x2 ≥ · · · ≥ xn ise

(x1 , x2 , · · · , xn ) Â (x, x, x, · · · , x)

durumu vardır. Burada x değeri, x1 , x2 , · · · , xn değerlerinin aritmetik ortası-


dır.(Bunu Majorization üzerine uygularsak Jensen Eşitsizliğini elde ederiz.)
Buna göre, k = 1, 2, · · · , n − 1 için x1 + x2 + · · · + xk ≥ kx durumunu göster-
memiz yeterli olacaktır. Buna göre,

(n − k)(x1 + x2 + · · · + xk ) ≥ (n − k)kxk ≥ k(n − k)xk+1 ≥ k(xk+1 + · · · + xn )

olacağından
(n − k)(x1 + · · · + xk ) ≥ k(xk+1 + · · · + xn )
bulunur. Bu eşitsizliğin iki tarafınada k(x1 + · · · + xk ) eklersek

n(x1 + · · · + xn ) ≥ k(x1 + · · · + xn ) = knx


56 BÖLÜM 2. KONULAR

olacağından
x1 + x2 + · · · + xk ≥ kx
olacaktır.
[7.] −1 ≤ a, b, c ≤ 1 ve a + b + c = 1/2 ise a12 + b12 + c12 en fazla kaç olur?
Çözüm. [−1, 1] aralığında f (x) = x12 fonksiyonu konvekstir. Öyleki, f 00 (x) =
132x10 ≥ 0 olacaktır. Eğer, 1 ≥ a ≥ b ≥ c ≥ −1 ve a + b + c = −1/2 ise major
üçlülerimizi
1
(1, − , −1) Â (a, b, c)
2
olarak seçebiliriz. Çünkü, 1 ≥ a ve 12 = 1 − 12 ≥ −c − 12 = a + b olacaktır. Buna
göre, majorization eşitsizliğinden
1
a12 + b12 + c12 = f (a) + f (b) + f (c) ≤ f (1) + f (2) + f (−1) = 2 +
212
olacaktır. Zaten, a = 1, b = − 12 ve c = − 21 içinde en büyük değer doğrulanır.
[8.](1999,IMO9 ) n ≥ 2 bir tamsayıdır. Buna göre,

(a.)
 4
X X
xi xj (x2i + x2j ) ≤ C  xi 
1≤i<j≤n 1≤i≤n

eşitsizliğini x1 , x2 , x3 , · · · , xn ≥ 0 reel sayıları için sağlayan en küçük C


sabitini bulunuz.

(b.) Bu C değeri için, eşitlik durumunu araştırınız.

Çözüm. İlk olarak n = 2 durumuna bakalım. Varsayalım x1 = m + h ve


x2 = m − h yani
x1 + x2 x1 − x2
m= ve h =
2 2
olsun. Buna göre,
1
x1 x2 (x21 + x22 ) = 2(m4 − h4 ) ≤ 2m4 = (x1 + x2 )4
8
9
International Math Olympiads, 1999
2.6. EŞİTSİZLİKLER II 57

olacaktır.
n > 2 durumu için varsayalım
xi
ai = ve a1 + a2 + · · · + an = 1 olsun.
x1 + x2 + · · · + xn
Buna göre, ai = [0, 1] olacaktır. Eğer, ai cinsinden yazarsak, ispatlanacak
eşitsizlik X
ai aj (a2i + a2j ) ≤ C
1≤i<j≤n
olacaktır. Eşitsizliğin sol tarafı açılıp düzenlenirse,
n
X n
X
a3i (a1 + · · · + ai−1 + ai+1 + · · · + an ) = a3i (1 − ai )
i=1 i=1

olacaktır. f (x) = x3 (1−x) = x3 −x4 fonksiyonunun [0, 1/2] aralığında konveks


olduğu açıktır. Öyle ki,
f 00 (x) = 6x − 12x2 = 6x(1 − 2x) > 0
olacaktır. Eşitsizliğimiz ai ’lere göre simetrik olduğundan, a1 ≥ a2 ≥ · · · ≥ an
alabiliriz.
Buan göre, eğer a1 ≤ 1/2 ise,
1 1
( , , 0, 0, · · · , 0) Â (a1 , a2 , · · · , an )
2 2
olacağından majorization eşitsizliğinden
1 1 1
f (a1 ) + f (a2 ) + · · · + f (an ) ≤ f ( ) + f ( ) + f (0) + · · · + f (0) =
2 2 8
olacaktır. Eğer, a1 > 1/2 ise 1 − a1 , a2 , · · · , an ∈ [0, 1/2) olacaktır.
(1 − a1 , 0, 0, · · · , 0) Â (a2 , a3 , · · · , an )
olduğundan majorization eşitsizliği ve n = 2 durumunu göz önünde bulundu-
rursak,
f (a1 ) + f (a2 ) + · · · + f (an ) ≤ f (a1 ) + f (1 − a1 ) + f (0) + f (0) + · · · + f (0)
1
= f (a1 ) + f (1 − a1 ) ≤ olacaktır.
8
Buna göre, eşitlik durumu ancak ve ancak iki değişken eşit ve geri kalan (n−2)
değişken 0 ise vardır.
58 BÖLÜM 2. KONULAR

2.6.1 Çalışma Soruları


1. Jensen Eşitisizliğini kullanarak aa bb cc ≥ (abc)(a+b+c)/3 eşitsizliğini kanıtlayınız.(a, b, c
pozitif reel sayılar.)
2. x1 , · · · , xn ∈ [0, 1] ve a1 , · · · , an > 0 olmak üzere, a1 +· · ·+an = 1 olarak
veriliyor. Buna göre,
n
X ai 1

1 + xi 1 + x1 · · · xa11
a1
i=1

eşitsizliğini kanıttlayınız. Eşitlik durumu hangi şartlar altında gerçekle-


şir?
3. Eğer a, b, c, d > 0 ve c2 + d2 = (a2 + b2 )3 ise
a3 b3
+ ≥1
c d
eşitsizliğini Hölder Eşitsizliği kullanarak kanıtlayınız.
4. P noktası ABC üçgeni içersinde

m(P AB) = m(P BC) = m(P CA) = α

eşitliğini sağlayan bir nokta ise α açısının π/6 olduğunu kanıtlayınız.


5. Dar açılı bir ABC üçgeninin iki açısı π/6’dan küçük veya eşittir. Buna
göre,
A B C π π π
sin sin sin ≥ sin sin sin
2 2 2 4 6 12
eşitsizliğini kanıtlayınız.
6. x, y, z > 1, xyz = 4096 ve max(x, y, z) < 32 olarak veriliyor. Buna göre,
x + y + z toplamının en büyük ve en küçük değerlerini bulunuz.
7. Eğer a, b ≥ 0 ise,
q q q q
3 √ 3 √
3 3 √
3 3 √
a+ 3a+ b+ b≤ a+ b+ b+ 3a

eşitsizliğini kanıtlayınız.
2.7. İNDİRGEMELİ DİZİLER 59

2.7 İndı̇rgemelı̇ Dı̇zı̇ler


2.7.1 Birinci Dereceden İndirgemeler
Herhalde matematik olimpiyatları sınavlarına hazırlanıpta Fibonacci Sa-
yıları’nı bilmeyen yoktur. Zaten Fibonacci sayılarıda f0 = 0, f1 = 1 olmak
üzere
fn+1 = fn + fn−1 , n ≥ 1
yinelemesi ile tanımlanır. Bir yinelemenin derecesi ise en büyük ve en küçük
alt terimlerin farkına eşittir. Mesela,

un+2 − un+1 = 2

birinci derecedendir, ve
un+4 + 9u2n = n5
yinelemesi ise dördüncü derecedendir. Eğer bir yinelemede tüm ifadelerin üst-
leri bir ise bu yinelemeye Doğrusal Yineleme denir. Mesela,

un+2 − un+1 = 2

bir doğrusal yinelemedir. Ancak

x2n + nxn−1 = 1 ve xn + 2xn −1 = 3

yinelemeleri lineer değillerdir. Eğer bir yinelemenin tüm terimlerinin kuvvetleri


aynı kuvvettense, bu yinelemeye Homojen’dir denir. Mesela,

xm+3 + 8xm+2 − 9xm = 0

yinelemesi homojendir. Ancak,

xm+3 + 8xm+2 − 9xm = m2 − 3

yinelemesi homojen değildir. Bir yinelemin sadece indis değişkenine göre ta-
nımlanan denkleme ise o yinelemenin Kapalı Form’u denir. Kapalı form sa-
yesinde rahatlıkla yinelemenin her hangi bir terimini bulabiliriz. Biz genel
manada bu ders notunda ilk olarak,

xn = axn−1 + f (n), a 6= 1
60 BÖLÜM 2. KONULAR

formundaki (f burada bir polinomdur) yinelemelerin çözümleriyle ilgilenece-


ğiz. İlgilenirken de aşağıda verdiğimiz basamakları takip edeceğiz.

1. Önce xn = axn−1 formunda verilen yinelemenin indislerini üste alarak


yani, karakteristik denklem oluşturarak, xn = axn−1 denklemini elde ede-
riz. Sadeleştirmeler yaptığımızda x = a olacaktır. Buna göre, xn = axn−1
homojen formundaki yinelememiz bize xn = Aan denklemini verecektir.
Burada A bir sabit sayıdır.

2. Daha sonra bulunan xn = Aan + g(n) formu test edilir. Burada g, f ile
aynı dereceden bir polinomdur.

Örnek 1. x0 = 7 ve xn = 2xn−1 , n ≥ 1 ise xn kapalı formunu bulunuz.


Çözüm. Alt indisleri üs olarak yazarsak karakteristik denklemimizi xn =
2xn−1 olacaktır. Sadeleştirme yaparsak, x = 2 olacaktır. Buna göre bizim,
xn = A2n formunda çözüm yapmamız gerekmektedir. Burada, 7 = x0 = A20
ise A = 7 olacağından, kapalı form xn = 7(2)n olarak bulunur.
Örnek 2. x0 = 7 ve xn = 2xn−1 + 1, n ≥ 1 formunda verilen yinelemenin
kapalı formunu bulunuz.
Çözüm. Karakteristik denklemi yazıldığında xn = 2xn−1 veya x = 2 elde
edilir. Buradan çözümlerden birinin xn = A(2)n olduğu açıktır. Ancak yinele-
menin bir parçasıda f (n) = 1 polinomu olduğuna göre, kapalı denklemin xn =
A2n +B formunda olması gerekmektedir. Buradan, 7 = x0 = A20 +B = A+B
ve 15 = x1 = 2A + B olduğuna göre bu iki denklemin çözümünden A = 8 ve
B = −1 olacaktır. Öyleyse, soruda istenen kapalı form,

xn = 8(2n ) − 1 = 2n+3 − 1

olacaktır.
Not. Örnek 2.’nin çözümünde dikkat edilirse oluşturula karakteristik denklem
iki parçadan oluşuyor. Bunlardan ilki zaten alt indislerin üs olarak yazılmasıyla
elde edilirken, ikincisi bir polinom ve bu polinomun derecesi yineleme içersinde
ki polinomun derecesi ile aynı. Bundan sonraki çözümlerde de polinom seçimi
benzer şekilde olacaktır.
Örnek 3. x0 = 2 ve xn = 9xn−1 − 56n + 63 ise xn kapalı formunu bulunuz.
2.7. İNDİRGEMELİ DİZİLER 61

Çözüm. Karakterisitk denklem yazıldığında karakterisitik denklemimiz xn =


9xn−1 veya x = 9 olacaktır. Buna göre, kapalı formun bir kısmı xn = A(9)n
formunda olacaktır. Ancak, soruda verilen yinelemenin ikinci kısmını f (n) =
−56n + 63 polinomu oluşturduğundan, çözüm olarak kullanacağımız polinom
g(n) = Bn + C olacaktır. Buna göre, yinelememizin kapalı formu

xn = A9n + Bn + C

olacaktır. x0 , x1 , x2 için çözümlere bakıldığında,

2 = A+C
25 = 9A + B + C
176 = 81A + 2B + C

eşitlikleri için A = 2, B = 7 ve C = 0 olacaktır. Buna göre soruda istenen


kapalı form, yada genel çözüm

xn = 2(9n ) + 7n

olarak bulunur.
Örnek 4. x0 = 1 ve xn = 3xn−1 − 2n2 + 6n − 3 ise xn kapalı formunu bulunuz.
Çözüm. Yinelemenin karakterisitk denklemini yazdıktan sonra kolaylıkla xn =
A(3)n indirgemesini elde edebiliriz. Ancak yinelememizin bir parçasıda f (n) =
−2n2 + 6n − 3 şeklinde ki ikinci dereceen bir polinom olduğundan özel çözü-
mümüzde kullanacağımız g(n) polinomu Bn2 + Cn + D olmalıdır. Buna göre,
yinelememizin indirgenmiş hali,

xn = A3n + Bn2 + Cn + D

şekline olacaktır. Eğer bilinen xi , i = 0, 1, 2, 3 için katsayaları bulmaya çalışır-


sak,

1 = A + D,
4 = 3A + B + C + D,
13 = 9A + 4B + 2C + D,
36 = 27A + 9B + 3C + D
62 BÖLÜM 2. KONULAR

denklemlerini elde ederiz. Buradan da, A = B = 1, C = D = 0 olacağından,


istenen kapalı form
xn = 3n + n2
olacaktır.
Örnek 5. x0 = 2 ve xn = 2xn−1 + 3n−1 ise kapalı formu bulunuz.
Çözüm. Eğer gerekli işlemleri yaparsak, genel formun

xn = A2n + B3n

denklemi elde edilir. Burada, x0 = 2 ve x1 = 2(2) + 30 = 5 denklemlerinden,

2 = A+B
7 = 2A + 3B

eşitliklerini elde edilir. Buna göre, A = 1 ve B = 1 olacağından istenen kapalı


form
xn = 2n + 3n
olacaktır.
Örnek 6. x0 = 7 ve xn = xn−1 + n, n ≥ 1 ise xn için kapalı formu bulunuz.
Çözüm. Şimdi bu çözümü siz yapmaya çalışın. Kapalı formu,

n(n + 1)
xn = 7 +
2
olarak bulmanız gerekmektedir.
Şimdiye kadar çözdüklerimizde genel olarak, lineer yinelemeler hakimdi. Şim-
diki örneğimizde de lineer olmayan ancak lineerleştirilebilir, birinci drecen yi-
nelemeler den birini çözeceğiz.
Örnek 6. u0 = 3 ve u2n+1 = un , n ≥ 1 ise yinelemin kapalı formunu bulunuz.
Çözüm. Varsayalım, vn = log un olsun. Buna göre,

1/2 1 vn − 1
vn = log un = log un−1 = log un−1 =
2 2
2.7. İNDİRGEMELİ DİZİLER 63

olacaktır. Burada,
vn−1
vn =
2
olduğundan,
v0
vn =
2n
olacaktır. Buradan,
log u0
log un =
2n
olduğundan istenilen kapalı form,
n
un = 31/2

olarak bulunur.

2.7.2 İkinci Dereceden İndirgemeler


Bir evvelki konumuzda birici dereceden yinelemeleri ele almıştık. Öyleki,
her bir terim kendisinden bir önceki terime bağımlı olarak veriliyordu. Şimdi
vereceğimiz formda ise durum artık biraz daha farklı. Öyleki artık karşılaca-
ğımız yinelemeler
xn = axn−1 + bxn−2
şeklinde olacaktır. Bu tür yinelemelerin çözümleri içinde takip etmemiz gerekn
bazı çözüm basamakları vardır. Buna göre,

1. Önce alt indisleri üs olarak alıp karakteristik denklemi xn = axn−1 +


bxn−2 olduğundan kökleri r1 ve r2 olan x2 − ax − b = 0 olarak bulunur.

2. Eğer kökler birbirinden farklı ise genel form

xn = A(r1 )n + B(r2 )n

şeklinde olacaktır.

3. Eğer kökler aynı ise genel form

xn = A(r1 )n + Bn(r1 )n

şeklinde olacaktır.
64 BÖLÜM 2. KONULAR

Örnek 7. x0 = 1, x1 = −1 ve xn+2 + 5xn+1 + 6xn = 0 yinelemesinin kapalı


formunu bulunuz.
Çözüm. Soruda verilen yinelemenin karakteristik denklemi

x2 + 5x + 6 = (x + 3)(x + 2) = 0

olarak elde edilir.Buna göre kapalı formumuz

xn = A(−2)n + B(−3)n

olacaktır. Buradan da, A = 2 ve B = −1 olacağından soruda istenilen kapalı


form,
xn = 2(−2)n − (−3)n
olacaktır.
Örnek 8. Fibonacci yinelemesi için kapalı formu, f0 = 0, f1 = 1 ve fn =
fn−1 + fn−2 bilgilerini kullanarak bulunuz.
Çözüm. Karakterisitk denklemimiz f 2 − f − 1 = 0 olacağından kapalı formu-
muz à √ !n à √ !n
1+ 5 1− 5
fn = A +B
2 2
şeklinde olacaktır. Başlangıç değerleri kullanıldığında,

0 = A+B
à √ !n à √ !n √ √
1+ 5 1− 5 1 5 5
1=A +B = (A + B) + (A − B) = (A − B)
2 2 2 2 2

olacağından
1 1
A = √ , B = −√
5 5
olacaktır. Sonuç olarakta Cauchy-Binet Formülü olarakta bilinen
à √ !n à √ !n
1 1+ 5 1 1− 5
fn = √ +√
5 2 5 2

kapalı form bulunacaktır.


2.7. İNDİRGEMELİ DİZİLER 65

Örnek 8.(Tübitak Deformesi10 ) x0 = 1, x1 = 4, xn = 4xn−1 − 4xn−2 ise


kapalı formu bulunuz.
Çözüm. Karakteristik denklemimiz x2 − 4x + 4 = (x − 2)2 = 0 olacaktır.
Burada köklerin birbirine eşit olduğu açıktır. Buna göre kapalı formumuz,
xn = A2n + Bn2n
formunda olacaktır. Eğer başlangıç değerlerini kullanırsak,
1 = A
4 = 2A + 2B
A = 1 ve B = 1 olarak bulunur. Buna göre, istenilen kapalı form
xn = 2n + n2n
olacaktır.

2.7.3 Alıştırmalar
Aşağıdaki sorulardan (1 − 5) için kapalı formları bulunuz.
xn−1 +4
1. x0 = 3, xn = 3

2. x0 = 1, xn = 5xn−1 − 20n + 25
3. x0 = 1, xn = xn−1 + 12n
4. x0 = 5, xn = 2xn−1 + 9(5n−1 )
5. a0 = 5, aj+1 = a2j + 2aj , j ≥ 0
6. (AIME, 1984) x19 = 94 ve
xn + xn−1 = n2 , n ≥ 1
ise x94 ’ün 1000 ile bölümünden kalan kaçtır?

Aşağıdaki (6 − 10) için ikinci dereceden yinelemelerin kapalı formlarını


bulunuz.
10
Benzer bir soru TÜBİTAK matematik olimpiyatlarında da sorulmuştur.
66 BÖLÜM 2. KONULAR

7. x0 = 0, x1 = 1, xn = 10xn−1 − 21xn−2

8. x0 = 0, x1 = 1, xn = 10xn−1 − 25xn−2

9. x0 = 0, x1 = 1, xn = 10xn−1 − 21xn−2 + n

10. x0 = 0, x1 = 1, xn = 10xn−1 − 21xn−2 + 2n

11. Bir düzlem üzerine çizilen n çember düzlemi parçalara ayırmaktadır.


Buna göre, düzlem üzerindeki n çemberin ayırdığı parçaların sayısını
veren denklemi bulunuz.

12. Bir düzlem üzerine çizilen n doğrunun düzlem üzerinde ayırdığı parçala-
rın sayısını veren denklemi bulunuz.

2.8 Yenı̇den Düzenleme [Rearrangement] Eşı̇tsı̇zlı̇ğı̇


Varsayalım elimizde iki reel üçlü (a1 , a2 , a3 ) ve (b1 , b2 , b3 ) olsun. Eğer ikinci
üçlünün tüm permütasyonlarını düşünürsek elimizde 3! = 6 tane üçlünün ola-
cağı açıktır. Biz bu üçlülerin kümesine P ve (x1 , x2 , x3 ) ∈ P olsun diyelim.
Buna göre varsayalım,
S = a1 x1 + a2 x2 + a3 x3
toplamıda elimizde bulunsun. Burada bizi ilgilendiren asıl soru S toplamının
ne zaman en büyük, ne zaman en küçük olduğudur. Bu soruya cevap aramaya
başlamadan evvel ileriki basamaklarda kullanacağımız bazı terimleri açıklaya-
lım.
Tanım. Varsayalım elimizde (a1 , a2 , a3 ) ve (b1 , b2 , b3 ) reel üçlüleri olsun. Buna
göre,

• Eğer iki üçlünün elemanlarıda artan veya iki üçlünün elemanlarıda aza-
lan bir sırada yazılmışsa bu ikiliye Benzer Düzenli diyelim. Yani a1 ≤
a2 ≤ a3 ve b1 ≤ b2 ≤ b3 veya a1 ≥ a2 ≥ a3 ve b1 ≥ b2 ≥ b3 durumları
sağlansın.

• Eğer iki üçlüden biri artan diğeri azalan sırada yazılmışsa bu üçlüyede
Aykırı Düzenli diyelim.
2.8. YENİDEN DÜZENLEME [REARRANGEMENT] EŞİTSİZLİĞİ 67

Örnek.
a. (−1, 1, 3) ve (2, 5, 7) benzer düzenlidirler.
b. Eğer 0 < a ≤ b ≤ c ise (a, b, c) ve ( a1 , 1b , 1c ) aykırı düzenlidir. Ama, (a, b, c)
1 1 1
ve ( b+c , a+c , a+b ) benzer düzenlidirler.
c. Eğer 0 < a ≤ b ≤ c ve m ∈ R+ ise (a, b, c) ve (am , bm , cm ) benzer
düzenliyken, (a, b, c) ve ( a1m , b1m , c1m ) aykırı düzenlidir.
d. Eğer a ≤ b ≤ c ve n bir tek tamsayı ise (a, b, c) ve (an , bn , cn ) benzer
düzenlidir.
Artık eşitsizliğimizi daha yakından tanıma zamanı geldi. Teorem [Re-
arragement Inequality]. (a1 , a2 , a3 ) ve (b1 , b2 , b3 ) iki reel üçlü olmak üzere
(x1 , x2 , x3 ) üçlüsü (b1 , b2 , b3 ) üçlüsünün bir permitasyonu olsun. Buna göre,
• Eğer (a1 , a2 , a3 ) ve (b1 , b2 , b3 ) benzer düzenli ise
a1 b1 + a2 b2 + a3 b3 ≥ a1 x1 + a2 x2 + a3 x3
olacaktır.
• Eğer (a1 , a2 , a3 ) ve (b1 , b2 , b3 ) aykırı düzenli ise
a1 b1 + a2 b2 + a3 b3 ≤ a1 x1 + a2 x2 + a3 x3
olacaktır.
Kanıt. Varsayalım elimizde (a1 , a2 , a3 ) ve (b1 , b2 , b3 ) artan sırayla dizilmiş iki
üçlü olsun. (x1 , x2 , x3 ) üçlüsüde (b1 , b2 , b3 ) üçlüsünün bir permütasyonu olsun
ve x1 ≥ x2 olsun. S ve S 0 toplamlarınıda
S = a1 x1 + a2 x2 + a3 x3
ve
S 0 = a1 x2 + a2 x1 + a3 x3
olarak alalım. Burada S 0 toplamı, S toplamındaki x1 ve x2 ’nin yer değiştirme-
siyle elde edildiği açıktır. Eğer bu iki toplamı farkını alırsak,
S 0 − S = (x1 − x2 )(a2 − a1 ) ≥ 0
| {z } | {z }
+ +
68 BÖLÜM 2. KONULAR

olacaktır. Demekki S 0 ≥ S’dir.


Buna göre, x1 ve x2 ’nin yer değiştirmesi sadece S toplamının değerini ar-
tırmaktadır. Öyleyse, eğer tüm (xi , xj ) ikililerinin (xi ≥ xj , i < j) yerleri
değiştirilirse toplam ancak en büyük olabilir. En büyük olacak toplamda za-
ten a1 b1 + a2 b2 + a3 b3 olacaktır. Benzer biçimde eğer (a1 , a2 , a3 ) ve (b1 , b2 , b3 )
üçlülerinin ikiside azalan birer üçlü olarak seçilseydi, ispatın ikinci kısmıda
buradan yapılabilirdi.
Şimdi, bu yeni eşitsizliğimizi birkaç örnek üzerinde uygulayalım.

Örnek. a, b, c ∈ R olmak üzere


i. a2 + b2 + c2 ≥ ab + bc + ca
ii an + bn + cn ≥ an−1 b + bn−1 c + cn−1 a
eşitsizliklerini gösteriniz.
Çözüm. Sorunun birinci şıkkı zaten ikinci şıkkın özel bir durumu olduğun-
dan sadece ikinci şıkkı çözmemiz yeterli olacaktır. Buna göre eğer üçlülerimizi
(a, b, c) ve (an−1 , bn−1 , cn−1 ) benzer düzenlileri olarak belirlersek, istenen eşit-
sizlik yeniden düzenleme eşitsizliği ile,
aan−1 + bbn−1 + ccn−1 ≥ abn−1 + bcn−1 + can−1
olarak bulunur.

Örnek. a, b, c > 0 oduğuna göre, aşağıdaki eşitsizlikleri kanıtlayınız.


a+b+c 1 1 1
i. abc ≤ a2
+ b2
+ c2
a2 b2 c2 b
ii. b2
+ c2
+ a2
≥ a + cb + a
c
a2 b2 c2
iii. b + c + a ≥a+b+c
Çözüm.
• a ≤ b ≤ c olarak kabul edelim ve ( a1 , 1b , 1c ) ve ( a1 , 1b , 1c ) benzer düzenlilerini
seçelim. Buna göre,
11 11 11 11 11 11
+ + ≥ + +
aa b b cc ab bc ca
1 1 1 1 1 1
+ + ≥ + +
a2 b2 c2 ab bc ac
2.8. YENİDEN DÜZENLEME [REARRANGEMENT] EŞİTSİZLİĞİ 69

olacaktır.

• Eğer ( ab , cb , ac ) ve ( ab , cb , ac ) benzer sıralılarını alırsak,

aa bb c c ab bc ca
+ + ≥ + +
b b cc aa bc ca ab
elde edilecektir.

• Bu seferde sıralı ikililerimizi aykırı düzenliler arasından seçelim. Yani,


(a2 , b2 , c2 ) ve ( a1 , 1b , 1c ) kullanacağımız üçlülerimiz olsun. Buna göre,

1 1 1 1 1 1
a2 + b2 + c2 ≤ a2 + b2 + c2
a b c b c a
elde edilecektir.

Örnek. [1963, Moskova Matematik Olimpiyatı] a, b, c > 0 olduğuna


göre,
a b c 3
+ + ≥
b+c a+c a+b 2
eşitsizliğini kanıtlayınız.
Çözüm. Varsayalım a ≤ b ≤ c olsun ve benzer sıralılarımız da (a, b, c) ve
1 1 1
( , , ) (2.1)
a+b a+c a+b
olsun. Buna göre,
1 1 1 a b c
a +b +c ≥ + + (2.2)
b+c c+a a+b c+a a+b b+c
1 1 1 a b c
a +b +c ≥ + + (2.3)
b+c a+c a+b a+b b+c a+c
eşitsizliklerini elde edebiliriz. Eğer [1] ve [2.3] eşitsizlerini altalta toplarsak,
a b c a b c a b c
2( + + )≥ + + + + + =3
b+c a+c a+b a+c a+b b+c a+b b+c a+c
eşitsizliğindende soruda istenen [2.1] eşitsizliğini elde ederiz.
70 BÖLÜM 2. KONULAR

Örnek. [Chebyshev Eşitsizliği] Eğer (a1 , a2 , a3 ) ve (b1 , b2 , b3 ) benzer


düzenliler ise
a1 b1 + a2 b2 + a3 b3 a1 + a2 + a3 b1 + b2 + b3
≥( )( ) (2.4)
3 3 3
eşitsizliğini kanıtlayınız.
Çözüm. Soruda zaten benzer düzenli üçlüler verildiğine göre, yeniden dü-
zenleme metodunu kullanalım

a1 b1 + a2 b2 + a3 b3 = a1 b1 + a2 b2 + a3 b3 (2.5)
a1 b1 + a2 b2 + a3 b3 ≥ a1 b2 + a2 b3 + a3 b1 (2.6)
a1 b1 + a2 b2 + a3 b3 ≥ a1 b3 + a2 b1 + a3 b2 (2.7)

eşitsizliklerini elde ederiz. Eğer [2.5], [2.6] ve [2.7] eşitsizliklerini altalta toplar-
sak,

3(a1 b1 + a2 b2 + a3 b3 ) ≥ a1 (b1 + b2 + b3 ) + a2 (b1 + b2 + b3 ) + a3 (b1 + b2 + b3 ))

eşitsizliğini ve sonuç olarakta soruda verilen [2.4] eşitsizliğini elde ederiz.

Not. Benzer biçimde, (a1 , a2 , a3 ) ve (b1 , b2 , b3 ) aykırı düzenlileri içinde,


a1 b1 + a2 b2 + a3 b3 a1 + a2 + a3 b1 + b2 + b3
≤( )( ) (2.8)
3 3 3
eşitsizliği elde edilebilir.

Örnek. [Aritmetik Orta - Karesel Orta Eşitsizliği] a1 , a2 , a3 reel


sayılar olmak üzere verilen
r
a1 + a2 + a3 a21 + a22 + a23
≤ (2.9)
3 3
eşitsizliğini kanıtlayınız.
Çözüm. Sorunun çözümü için Örnek 0.7’de kanıtladığımız eşitsizliği kul-
lanmamız kafidir. Buna göre, (a1 , a2 , a3 ) ve (a1 , a2 , a3 ) benzer düzenlileri için,

a21 + a22 + a23 a1 + a2 + a3 a1 + a2 + a3 a1 + a2 + a3 2


≤( )( )=( ) (2.10)
3 3 3 3
2.8. YENİDEN DÜZENLEME [REARRANGEMENT] EŞİTSİZLİĞİ 71

olduğuna göre, soruda istenen [2.9] eşitsizliği kanıtlanmış olur.

Örnek. [Aritmetik Orta-Geometrik Orta Eşitsizliği]a1 , a2 , a3 pozitif sa-


yılar olmak üzere verilen
a1 + a2 + a3 √
≥ 3 a1 a2 a3 (2.11)
3
eşitsizliğini kanıtlayınız.

Çözüm. P = 3 a1 a2 a3 olmak üzere, sorua kullanacağımız aykırı düzenli-
lerimizi,
a1 a1 a2 a1 a2 a3
x1 = , x2 = , x3 = = 1 ve (2.12)
P P2 P3
1 1 1
y1 = , y2 = , y3 = =1 (2.13)
x1 x2 x3

olarak seçelim. Burada (x1 , x2 , x3 ) artan sıralı ise (y1 , y2 , y3 ) azalan sıralı ola-
caktır. Buna göre,

x1 y1 + x2 y2 + x3 y3 ≤ x1 y3 + x2 y1 + x3 y2
a1 a2 a3 a1 + a2 + a3
1+1+1 ≤ + + =
P P P P
olacaktır. Buradan da,

3P ≤ a1 + a2 + a3
a1 + a2 + a3
P ≤
3
olacağından, istenen kanıt tamamlanmış olur.

Örnek. Chebyshev eşitsizliğini iki reel sayı için gösteriniz.


Çözüm. Varsayalım benzer düzenli ikililerimiz (a1 , a2 ) ve (b1 , b2 ) olsun.
Buna göre,

a1 b1 + a2 b2 = a1 b1 + a2 b2
a1 b1 + a2 b2 ≥ a1 b2 + a2 b1
72 BÖLÜM 2. KONULAR

eşitsizlikleri taraf tarafa toplanırsa,

2(a1 b1 + a2 b2 ) ≥ (a1 + a2 )(b1 + b2 )


a1 b1 + a2 b2 a1 + a2 b1 + b2
≥ ( )( )
2 2 2
olacaktır.

Örnek.
an + bn 1
≥ (an−1 + bn−1 )
a+b 2
eşitsizliğini kanıtlayınız.
Çözüm. Örnek 0.11’de kanıtladığımız eşitsizliği kullanalım. Buna göre
ikililerimiz (a, b) ve (an−1 , bn−1 ) ikilileri olsun. Buna göre,

aan−1 + bbn−1 a + b an−1 + bn−1


≥ ( )( )
2 2 2
an + bn 1 n−1
≥ (a + bn−1 )
a+b 2

Örnek. a, b ≥ 0 olmak üzere, aşağıdaki eşitsizlikleri kanıtlayınız.

a. 2(a5 + b5 ) ≥ (a3 + b3 )(a2 + b2 )

b. a9 + b9 ≥ a2 b2 (a5 + b5 )

c. (a + b)n ≤ 2n−1 (an + bn )

Çözüm.

a. (a2 , b2 ), (a3 , b3 ) benzer düzenlileri olsun. Buna göre,

a2 a3 + b2 b3 a2 + b2 a3 + b3
≥( )( ) ⇒ 2(a5 + b5 ) ≥ (a2 + b2 )(a3 + b3 )
2 2 2
olarak bulunur.
2.8. YENİDEN DÜZENLEME [REARRANGEMENT] EŞİTSİZLİĞİ 73

b. Bu şıktada benzer düzenlilerimizi (a4 , b4 ) ve (a5 , b5 ) olarak seçelim. Buna


göre,
a4 a5 + b4 b5 a4 + b4 a5 + b5 √ a5 + b5 a5 + b5
≥( )( ) ≥ a4 b4 ( ) = a2 b2 ( )
2 2 2 2 2
olacaktır. Bu eşitsizliktende,a9 + b9 ≥ a2 b2 (a5 + b5 ) eşitsizliği elde edilir.
c. Soruyu Chebyshev Eşitsizliği’ni kullanarak çözmeye çalışalım. İkilileri-
miz (an−1 , bn−1 ) ve (a, b) olsun. Buna göre,
1 n−1 1
an + bn ≥ (a + bn−1 )(a + b) = (an−2 a + bn−2 b)(a + b)
2 2
1 n−2
an + bn ≥ (a +b n−2
)(a + b)(a + b)
22
1 1
an + bn ≥ n−1
(a + b)(a + b) · · · (a + b) = n−1 (a + b)n olacaktır.
2 2
Buna göre istenen eşitsizlik,
2n−1 (an + bn ) ≥ (a + b)n
elde edilmiş olur.

Örnek. a, b, c > 0 ve n ∈ Z+ ise


an bn cn an−1 + bn−1 + cn−1
+ + ≥
b+c a+c a+b 2
eşitsizliğini kanıtlayınız.
Çözüm. a ≤ b ≤ c olmak üzere benzer düzenlilerimiz (an , bn , cn ) ve
1 1 1
( b+c , a+c , a+b ) olarak seçelim. Buna göre,
an bn cn an bn cn
+ + ≥ + + (2.14)
b+c a+c a+b a+c a+b b+c
an bn cn an bn cn
+ + ≥ + + (2.15)
b+c a+c a+b b+a b+c a+c
eşitsizliklerini elde ederiz. Eğer bu iki eşitsizliği taraftarafa toplarsak,
an bn cn an + bn bn + cn an + cn
2( + + )≥ + +
b+c a+c a+b a+b b+c a+c
74 BÖLÜM 2. KONULAR

eşitsizliğini elde ederiz. Bundan sonraki basamaktada Örnek 0.12 ’de kanıt-
ladığımız eşitsizliği kullanırsak,
an bn cn an + bn bn + cn an + cn
2( + + ) ≥ + +
b+c a+c a+b a+b b+c a+c
1 n−1 1 1
≥ (a + bn−1 ) + (bn−1 + cn−1 ) + (an−1 + cn−1 )
2 2 2
= an−1 + bn−1 + cn−1

eşitsizliğinden, soruda bizden istenen eşitsizliğe ulaşılır.

Örnek. a, b, c > 0 olmak üzere,


a+b+c
aa bb cc ≥ (abc) 3

eşitsizliğini kanıtlayınız.
Çözüm. a ≤ b ≤ c olarak alalım. Üçlülerimizide, biraz sıradışı bir se-
çim yaparak, (a, b, c) ve (log a, log b, log c) olarak alalım. Buna göre,Chebyshev
eşitsizliğinden
a log a + b log b + c log c a + b + c log a + log b + log c
≥( )( )
3 3 3
eşitsizliği elde edilir. Buna göre,

log aa bb cc a + b + c log abc


≥( )( )
3 3 3
olacağından istenen eşitsizlik,
a+b+c a+b+c
log aa bb cc ≥ log(abc) 3 ⇒ aa bb cc ≥ (abc) 3

olarak bulunur.

Örnek. A, B, C bir üçgenin açıları olmak üzere (radyan cinsinden), a, b, c


uzunukları da bu üçgenin kenar uzunluklarıdır. p = 12 (a + b + c) olduğuna göre,

A B C 3π
+ + ≥ (2.16)
p−A p−b p−c p
2.9. TRİGONOMETRİK DEĞİŞKEN DEĞİŞTİRME 75

eşitsizliğini kanıtlayınız.
Çözüm. Varsayalım A ≤ B ≤ C olsun. Buna göre benzer düzenli üçlüle-
1 1 1
rimizi (A, B, C) ve ( p−a , p−b , p−c ) seçebiliriz. Chebyshev eşitsizliğinden,

1 A B C A+B+C 1 1 1 1
( + + ) ≥ ( )( + + )
3 p−A p−b p−c 3 p−a p−b p−c 3
π 1 1 1 1
= ( + + )
3 p−a p−b p−c 3
π 9
≥ ·
9 p
eşitsizliğindende [2.16] eşitsizliği kolaylıkla çıkarılır.

Örnek. [1995,IM0] a, b, c ∈ R+ ve a · b · c = 1 olduğuna göre,


1 1 1 3
+ 3 + 3 ≥ (2.17)
a3 (b + c) b (a + c) c (a + b) 2
eşitsiliğini kanıtlayınız.
Çözüm. Varsayalım x = a1 , y = 1b , z = 1c olsun. a · b · c = 1 olduğundan,
x · y · z = 1 olacağı açıktır. Buna göre, yeni değişkenlere göre eşitizliğimiz
düzenlersek,
x2 y2 z2 3
+ + ≥
y+z x+z y+x 2
eşitsizliğini elde ederiz. Çözümün bundan sonraki basamakları sadece Örnek
0.14’deki eşitsizliğin kanıtının kullanılmasıyla olacaktır. Buna göre,
x2 y2 z2 x+y+z 3√ 3
+ + ≥ ≥ 3 xyz =
y+z x+z y+x 2 2 2
olacaktır.

2.9 Trı̇gonometrı̇k Değı̇şken Değı̇ştı̇rme


Trigonometrik özdeşliklerin çokluğu, bu denklemleri kullanarak cebirsel
denklemlere oldukça kolay çözümler geliştirmemize yardımcı olur. Değişken de-
ğiştirmede, hangi özdeşliği kullanacağımız aslında çoğu zaman verilen sorunun
76 BÖLÜM 2. KONULAR

içine gömülüdür. Bu sebebten sorulara çözmeye başlamadan önce denklemi


nasıl yeniden düzenleyeceğimizin kararına varmamız yerinde olur.

Örnek. Aşağıdaki denklem sisteminin tüm reel çözümlerini bulunuz.


x3 − 3x = y
y 3 − 3y = z
z 3 − 3z = x

Çözüm. Soruda verilen x3 − 3x formu kosinüs üç kata açı özdeşliğini


anımsatıyor. Ancak tabiki x3 değişkeninin önündeki katsayı yok. Bu yüzden
değişken yerine çift kat açı kosinüsü yerine, sadece cos x değişkeni almamız
yerinde olacaktır. Buna göre x = 2 cos u, y = 2 cos v, z = 2 cos w u, v, w ∈ [0, π]
ise
2 cos 3u = 2 cos v
2 cos 3v = 2 cos w
2 cos 3w = 2 cos u
sistemini elde ederiz. Kosinüs üç kat açı formülünü cos 3u ve cos v için kul-
lanırsak cos 9u = cos 3v olacaktır. Bu eşitliği ikinci denklem de kullanırsak
cos 9u = cos w olur. Aynen daha önceki gibi cos 27u = cos 3w ise üçüncü denk-
lem cos 27u = cos u olacaktır.Demek ki, u = kπ/14, k = 0, 1, 2, · · · , 14 ve
u = kπ/13, k = 1, 2, · · · , 12 olacağından
x = 2 cos kπ/14, y = 2 cos 3kπ/14, z = 2 cos 9kπ/14, k = 0, 1, · · · , 14
ve
x = 2 cos kπ/13, y = 2 cos 3kπ/13, z = 2 cos 9kπ/13, k = 1, 2, · · · , 12
olacağından 3 × 3 × 3 = 27 tane çözüm elde edilir.
Şimdide tanjant fonksiyonunu kullanarak sıradaki örneği çözmeye çalışalım.

Örnek. {xn }∞
n=1 olmak üzere,

3xn − 1
xn+1 = √ , n>1
xn + 3
2.9. TRİGONOMETRİK DEĞİŞKEN DEĞİŞTİRME 77

İse , bu dizinin periyodik olduğunu kanıtlayınız.

Çözüm. Tanjant fark formülünü hatırlarsak


tan a − tan b
tan (a − b) =
1 − tan a · tan b

olacaktır. tan (π/6) = 1/ 3 olduğunuda biliyoruz. Buna göre,

xn − 1/ 3
xn+1 = √
1 + xn 1/ 3
olarak yazılabilir. Eğer x1 = tan t alırsak x2 = tan (t − π/6) olacağından,
xn = tan (t − (n − 1) π/6) , n > 1
olur. Tanjant fonksiyonuda periyodik olduğuna göre, demek ki, xn = xn+6
olacağından soruda verilen dizimiz periyodiktir ve periyodu 6 dır.

Örnek. a, b, c, x, y, z > 0 ise


¡ 2 ¢¡ ¢¡ ¢
a + x2 b2 + y 2 c2 + z 2 > (ayz + bzx + cxy − xyz)2
eşitsizliğini kanıtlayınız.

Çözüm. a = x tan α, b = y tan β, c = z tan γ α, β, γ ∈ [0, π/2] olarak


alalım. Buna göre,
1 ≥ cos2 (x + y + z) = (sin x · sin (y + z) − cos x · cos (y + z))2
= (sin x · sin y · sin z + sin x · cos y · sin z + cos x · sin y · sin z − cos x · cos y · cos z)2
eşitsizliğinin iki tarafınıda
cos2 α cos2 β cos2 γ
ile bölersek,
sec2 α sec2 β sec2 γ > (tan α + tan β + tan γ − 1)2
olacağından,
¡ 2 ¢¡ ¢¡ ¢
x tan2 α + x2 y 2 tan2 β + z 2 z 2 tan2 γ + z 2 > x2 y 2 z 2 (tan α + tan β + tan γ − 1)2
olacaktır. Buda zaten soruda istenen eşitsizliği doğrular.
78 BÖLÜM 2. KONULAR

2.9.1 Çalışma Soruları


1. a reel parametresinin hangi değerleri için
p
1 − x2 > a − x

eşitsizliğini sağlayan bir x reel değeri vardır?

2. (0, 1) aralığından dört farklı sayı alınıyor. Buna göre, bu sayılardan iki
tanesi olan xve y’nin
p p 1
0 < x 1 − y 2 − y 1 − x2 <
2
eşitliğini sağlayacağını kanıtlayınız.

3. Seçilen 4 farklı reel sayı arasından


1 + ab 1
√ √ >
2
1+a 1+b 2 2
eşitsizliğini sağlayan bir (a, b) ikilisinin bulunabileceğini kanıtlayınız.
¡ ¢2
4. x ∈ R olmak üzere verilen x2 + 4x3 − 3x = 1 denkleminin tüm reel
çözümlerini bulunuz.

5. Z q

I= 2+ 2 + · · · + 2 + xdx

integralini hesaplayınız.(Not: İfade içinde n ≥ 1 tane kök vardır.)



6. {xn }∞
n>1 dizisi xn+2 + 2 6 xn 6 2 eşitsizliğini sağlandığına göre, x1986
’nın alabileceği tüm değerleri bulunuz.

7.

2x + x2 y = y
2y + y 2 z = z
2z + z 2 x = x

denklem sisteminin tüm reel çözümlerini bulunuz.


2.9. TRİGONOMETRİK DEĞİŞKEN DEĞİŞTİRME 79

8. Aşağıdaki denklem sisteminin tüm reel çözümlerini bulunuz.


1
x1 − = 2x2
x1
1
x2 − = 2x3
x2
1
x3 − = 2x4
x3
1
x4 − = 2x1
x4

9. x, y ∈ R olmak üzere, aşağıdaki eşitsizliği kanıtlayınız,


1 (x + y) (1 − xy) 1
− 6 2 2
6
2 (1 + x ) (1 + y ) 2

10. x ∈ R , {xn }n ve x1 = x olmak üzere


1 1
xn+1 = −
xn 1 + xn
eşitliği veriliyor. Eğer xn = ±1 ise dizimiz kilitleniyor, çünkü xn+1 ta-
nımlanamıyor. Buna göre, 8. terimden sonra kilitlenen bu şekilde bir dizi
var mıdır?

11. Reel tanımlı {ak }∞ +


k=1 dizisi, k ∈ Z olmak üzere

k · ak + 1
ak+1 =
k − ak
olarak veriliyor. Buna göre bu dizinin sonsuz sayıda pozitif ve sonsuz
sayıda negatif terimi olduğunu kanıtlayınız.

12. −1 6 a1 6 a2 6 · · · 6 an 6 1 ise
n−1 r q¡ √
X ¢ ¡ ¢ π 2
1 − ai ai+1 − 1 − a2i 1 − a2i+1 <
2
i=1

eşitsizliğini kanıtlayınız.
80 BÖLÜM 2. KONULAR

P
n
13. x0 = 0, x1 , x2 , · · · , xn > 0 ve xk = 1 ise
k=1

n
X xk π
√ √ <
1 + x0 + x1 + · · · + xk−1 xk + xk+1 + · · · + xn 2
k=1

eşitsizliğini kanıtlayınız.

14. x, y, z ∈ (0, 1) olmak üzere

x2 + y 2 + z 2 + 2xyz = 1

eşitliğini sağlayan tüm (x, y, z) tamsayı üçlülerini bulunuz.

15. a, b, c pozitif tamsayılar olmak üzere verilen

x+y+z = a+b+c
4xyz − (a x + b2 y + c2 z) = abc
2

eşitliklerini sağlayan x, y, z reel sayılarını bulunuz.

16. cos 2π/5 ifadesinin değerini bulunuz.

2.9.2 Çözümler
1. t ∈ [0, π] olmak
√ üzere√cos t = x olsun. Çünkü |x| ≤ 1 olmak zorundadır.
Buradan 1 − x2 = 1 − cos2 x = sin t olacaktır. Çünkü t ∈ [0, π] için
sinüs fonksiyonu pozitiftir. Buna göre eşitsizliğimiz,

sin t + cos t > a

olacaktır.
π ³ π´ √ ³ π´
f (t) = sin t + cos t = 2 sin cos t − = 2 cos t −
4 4 4

olacağından, f (t)’nin [0, π] aralığındaki
√ en büyük değeri 2 olacaktır.
Buna göre, a’nın bulunduğu aralık 2’den büyük olmayan reel sayılar
kümesi olacaktır.
2.9. TRİGONOMETRİK DEĞİŞKEN DEĞİŞTİRME 81

2. Varsayalım sayılarımız a1 , a2 , a3 , a4 olsun. Burada, ak = sin tk , tk ∈


(0, π/2) alalım. Buna göre aslında problemde sorulan soru i ve j gibi
iki indisin
1
0 < sin ti cos tj − sin tj cos ti <
2
eşitsizliğini sağlayacak biçimde varolup olmadıklarıdır. Eğer eşitsizliği-
mizi düzenlersek

sin ti cos tj − sin tj cos ti = sin (ti − tj )

olacaktır. Buna göre, ispatlamamız gereken durum artık ti > tj ve


ti − tj < π/6 olacak şekilde i ve j’nin bulunduğunu göstermek olacak-
tır. Burada devreye güvercin yuvası ilkesi girer öyleki, seçeceğimiz dört
sayıdan iki tanesi kesinlikle

(0, π/6] , (0, π/3] , (π/3, π/2)

aralığında olacaktır. Buna göre eşitsizliği sağlayan ti ve tj vardır, dola-


yısıyla x ve y’de vardır.

3. x ∈ (0, π) olmak üzere, tüm reel sayılar tan x cinsinden ifade edilebilir.
Buna göre a = tan x ve b = tan y olarak alırsak
1 + tan x · tan y
√ p = cos x · cos y + sin x · sin y = cos (x − y)
1 + tan2 x · 1 + tan2 y
olacaktır. Buna göre şimdi problemimizi yeniden kuralım. Yeni durumu-
muz,
cos(x − y) > 1/2
olduğundan (x − y) < π/3 olmalıdır. Gerçektende (0, π) aralığında seçe-
ceğimiz dört sayıdan ikisinin farkı π/3’ten küçük11 olacaktır.

4. Dikkat edilirse |x| ≤ 1 ve |4x3 − 3x| ≤ 1 olarak verilmiştir. Buna göre,


x = cos a, 0 ≤ a ≤ π alabiliriz. Eğer cos 3a ve cos a ifadelerini kullanırsak
soruda verilen denklemimiz

cos2 a + cos2 3a = 1
11
Pigeon Hole Principle yada Güvercin Yuvası İlkesi bu küçüklüğü garantiler.
82 BÖLÜM 2. KONULAR

olacaktır. Buradan da,

1 + cos 2a 1 + cos 6a
+ =1
2 2
olacağından 2a ve 4a değerleri

π 3π π 3π 5π 7π
2a = , ve 4a = , , ,
2 2 2 2 2 2
q √
2± 2
olarak bulunur. Buna göre istenilen çözümler, ± π2 ve ± 2 değerle-
rini alacaktır.

5. Varsayalım x = 2 cos t olsun, buna göre


r q r q
√ √
2 + 2 + ··· + 2 + x = 2 + 2 + · · · + 2 + 2 cos t
s r
t
= 2 + 2 + · · · + 2 cos
2
t
= 2 cos n
2
olacaktır. Buradan integralimiz
Z Z
t t
I = 2 cos n dx = −2 cos n 2 sin t dt
2 2
Z
t
= −4 sin t · cos n dt
2
Z µ ¶
2n + 1 2n − 1
= −2 sin t − sin t dt
2n 2n

eşitliğinden istenilen cevap,


µ ¶ µ ¶
2n+1 2n+1 x 2n+1 2n − 1 x
cos arccos − cos arccos +c
2n + 1 2n + 1 2 2n − 1 2n 2

olarak bulunur.
2.9. TRİGONOMETRİK DEĞİŞKEN DEĞİŞTİRME 83

6. 0 ≤ xn ≤ 2 olduğuna göre, xn = 2 cos yn , yn ∈ [0, π/2] değişken değiştir-


mesini kullanabiliriz. Buna göre
p
xn+2 + 2 ≤ xn

eşitsizliği ve
cos 2α + 1 = 2 cos2 α
eşitliğinden µ ¶
yn + 2
cos ≤ cos yn
2
eşitsizliği bulunacaktır. Seçilen aralıkta kosinüs fonksiyonu azalan bir
fonksiyon olduğuna göre,
yn+2
≥ yn
2
olacaktır. Buradan da
yn+2k
yn ≤ , k ∈ Z+
2k
elde edilir. Burada k değeri sonsuza giderken yn ≤ 0 yani yn = 0 olacak-
tır. Dolayısıyla
xn = 2 cos yn = 2 · 1 = 2
olacağından xn = 2 ve x1986 = 2 bulunur.

7. Eğer değişkenlerden herhangi biri, varsayalım x = ±1 olursa

2x + x2 y = y ⇒ 2 · ±1 + y = y ⇒ 2 = 0

olacaktır ki, bu durum imkansızdır. Buna göre denklem sistemimizi ye-


niden yazarsak,
2x
= y
1 − x2
2y
= z
1 − y2
2z
= x
1 − z2
84 BÖLÜM 2. KONULAR

eşitliklerini elde ederiz. Burada da x = tan a, a ∈ (−π/2, π/2) olarak


alalım. Buna göre ilk iki denklemden y = tan 2a ve z = tan 4a olacaktır.
Son denklemde ise tan 8a = tan a eşitliği elde edilir. Buna göre 8a −
a = kπ ise a = kπ/7 olacaktır. a ∈ (−π/2, π/2) olduğuna göre k =
−3, −2, −1, 0, 1, 2, 3 değerlerini alabiliriz. Buradan da istenilen değerler
bulunabilir.

8. Sorunun çözümüne yardımcı olacak trigonometrik özdeşlik aslında soru-


nun içine gömülmüştür. Eğer çift kat açı için kotanjant toplamını kontrol
edersek
1
2 cot 2α = cot α −
cot α
eşitliğini elde ederiz. Eğer x1 = cot α, α ∈ (0, π) olarak alırsak x2 =
cot 2α olacağından, x3 = cot 4α ve x4 = cot 8α olacaktır. Benzer şe-
kilde döngü başa geleceğinden x1 = cot 16α olacaktır. Demek ki cot α =
cot 16α yani 16α − α = kπ, k ∈ Z eşitliği bulunacaktır. Buradanda
çözümlerimiz

α= , k = 1, 2, 3, · · · , 14
15
olacağından
kπ 2kπ 4kπ 8kπ
x1 = cot , x2 = cot , x3 = cot , x4 = cot
15 15 15 15
olacaktır.

9. Varsayalım x = tan a ve y = tan b olsun. Buradan,

sin(a + b)
x + y = tan a + tan b =
cos a cos b
cos(a + b)
1 − xy = 1 − tan a tan b =
cos a cos b
eşitlikleri elde edilir. Ayrıca,
1
= cos2 a
1 + x2
1
= cos2 b
1 + y2
2.9. TRİGONOMETRİK DEĞİŞKEN DEĞİŞTİRME 85

eşitlikleri elde edilir. Eğer yerine koyarsak,


sin(a+b) cos(a+b)
1 cos a cos b · cos a cos b 1
− 6 1 1 6
2 cos2 a cos2 b
2

ise −1 ≤ 2 sin(a + b)cos(a + b) ≤ 1 eşitsizliğini kanıtlamamız gerekir.


Zaten bu eşitsizlik −1 ≤ sin(2a + 2b) ≤ 1 olduğundan kanıt tamamlanır.

10.
1 1 2xn
− =
1 − xn 1 + xn 1 − x2n
eşitliğini yazabiliriz. Eğer x1 = tan β, β ∈ (−π/2, π/2) alırsak

2 tan β
x2 = = tan 2β
1 − tan2 β

olacaktır. Bu şekilde devam edersek, xn = tan 2n−1 β olacağından x8 =


tan 27 β = tan 128β olacaktır. 8 terim uzunluğundaki bir dizi için, tan 128β =
±1 olmalıdır. Buna göre

(2k + 1)π
128β =
4
olacaktır. Dolayısıyla,
µ ¶
(2k + 1)π
x = tan ± , k = −128, · · · , 128
512

olacaktır.

11. Eğer b1 = tan−1 a1 ve bk+1 = bk + tan−1 (1/k), k = 1, 2, 3, · · · olacak


biçimde bir dizi tanımlarsak tanjant toplam formülünden
tan x + tan y
tan(x + y) =
1 − tan x tan y
eşitliğinin ışığında ak = tan bk olduğunu görebiliriz.
tan x
lim =1
x→0 x
86 BÖLÜM 2. KONULAR

olduğuna göre,
tan−1 (1/k)
lim =1
k→∞ 1/k
olacaktır. Buradan da,

X 1
b0 + tan−1
k
k=1
ıraksak olacaktır. Öte yandan serinin elemanları k → ∞ iken, sıfıra ya-
kınsayacaktır. (2πn, 2πn + π/2) aralığında sonsuz çoklukta parçalı top-
lam olacaktır. Benzer biçimde (2πn + π/2, (2n + 1)π) aralığında da son-
suz çoklukta toplam olacaktır. Fakat, parçalı toplamlardam birisi bm ve
am = tan bm olduğundan, demek ki sonsuz çoklukta pozitif ve negatif
am değeri bulunmaktadır.
13. Her bir ai değerimiz [−1, 1] aralığında olduğuna göre, ai = cos xi , xi ∈
[0, π] şeklinde seçim yapabiliriz. Buna göre sorudaki eşitsizliğin sol tarafı
n−1
X n−1
Xp
p
1 − cos xi cos xi+1 − sin xi sin xi+1 = 1 − cos (xi+1 − xi )
i=1 i=1
√ n−1
X xi+1 − xi
= 2 sin olacaktır.
2
i=1

Sinüs fonksiyonu [0, π] aralığında aşağı konkav olduğuna göre Jensen


Eşitsizliğinden
n−1 µ ¶ Ã n−1
!
1 X xi+1 − xi 1 X xi+1 − xi
sin 6 sin
n−1 2 n−1 2
i=1 i=1

eşitsizliği elde edilir. Buna göre,


µ ¶
√ n−1X xi+1 − xi √ xn − xn−1 √ π
2 sin 6 (n − 1) 2 sin 6 2 (n − 1) sin
2 2 (n − 1) 2 (n − 1)
i=1

olacaktır. Burada xn − xi ∈ (0, π) dir. Aynı aralıkta, sin x < x, x > 0


eşitsizliği kullanılırsa
µ ¶
√ π √ π
2 (n − 1) sin 6 2
2 (n − 1) 2
2.9. TRİGONOMETRİK DEĞİŞKEN DEĞİŞTİRME 87

olacaktır.

13. Tüm xi değerleri pozitif ve toplamlarıda 1 olduğuna göre,

x1 + x2 + · · · + xk = sin ak

ve
π
a0 = 0 < a 1 < a 2 < · · · < a n = , k = 0, 1, · · · , n
2
seçimlerini yapabiliriz. Buna göre,
n
X sin ak − sin ak−1 π
√ √ <
1 + sin ak−1 1 − sin ak−1 2
k=1

olacaktır. Bu eşitsizliği yeniden düzenlersek


³ ´ ³ ´
ak −ak−1 ak +ak−1
X n 2 sin 2 cos 2
cos (ak − 1)
k=1

olacaktır. 0 < x < π/2 aralığında kosinüs fonksiyonu azalandır ve sin x <
x eşitsizliği vardır. Buna göre eşitsizliğin sol tarafı
n
X a −a
2 k 2 k−1 cos ak−1
cos ak−1
k=1

ise
n
X π
(ak − ak−1 ) =
2
k=1
olacaktır ki bu da zaten ispatı tamamlar.

14. Denklemi sağlayan üçlülerden herbiri dar açılı bir üçgenin açılarının ko-
sinüs değerleridir. Önce, eğer A, B, C açıları bir üçgenin açı değerleri
ise
cos2 A + cos2 B + cos2 C + 2 cos A cos B cos C = 1
olduğunu gösterelim. Aslında,

cos A = − cos (B + C) = sin B sin C − cos B cos C


88 BÖLÜM 2. KONULAR

olacağına göre
= cos2 A + cos2 B + cos2 C + 2 cos A cos B cos C
¡ ¢¡ ¢
= (cos A + cos B + cos C)2 + 1 − 1 − cos2 A 1 − cos2 C
= (sin B sin C)2 + 1 − sin2 B sin2 C = 1
olarak bulunur. Demek ki, A + B + C = π eşitliğini sağlayan A, B, C
değerleri birer çözümdür.
15. İkinci denklemi düzenlersek
a2 b2 c2 abc
+ + + =4
yz zx xy xyz
olacaktır. Buna göre,
a b c
x1 = √ , y1 = √ , z1 = √
yz zx xy
olarak alırsak, eşitliğimiz
x21 + y12 + z12 + x1 y1 z1 = 4
olacağından 0 < x1 < 2, 0 < y1 < 2, 0 < z1 < 2 olacaktır. Buna göre,
x1 = 2 cos A, y1 = 2 cos B, z1 = 2 cos C
olarak alabiliriz. Bu değerleri kullanırsak,
√ √ √
2 yz cos A = a, 2 zx cos B = b, 2 xy cos C
olacaktır. Eğer x + y + z = a + b + c ise
√ √ √
x + y + z − 2 yx cos A − 2zx cos B − 2 xy cos C = 0
olacaktır. Bu eşitliğin sol tarafını iki kare toplamı şeklinde yazmaya ça-
lışırsak
√ √ √
= x + y + z − 2 yz cos A − 2 zx cos B − 2 xy cos C
√ √ √
= x + y + z − 2 yz cos A − 2 zx cos B + 2 xy (cos A cos B − sin A sin B)
¡ 2 ¢ ¡ ¢ √
= x sin B + cos B 2 + y sin2 A + cos2 A + z − 2 yz cos A
√ √ √
− 2 zx cos B + 2 xy cos A cos B − 2 xy sin A sin B
¡√ √ ¢2 ¡√ √ √ ¢
= x sin B − y sin A + x cos B + y cos A − z
2.9. TRİGONOMETRİK DEĞİŞKEN DEĞİŞTİRME 89

olacaktır. Burada iki kare toplamı sıfır olduğuna göre


√ √
x sin B − y sin A = 0

ve √ √

x cos B + y cos A = z
olacaktır. Buradan,
√ √ b √ a b+a
z= x √ + y √ = √
2 zx 2 yz 2 z

olacaktır. Demek ki
a+b
z=
2
’dir. Simetriden dolayı
a+c b+c
y= ve x =
2 2
olarak bulunur.

16. Önce bildiğimiz bazı trigonometrik soruda kullanabileceğimiz bazı ge-


ometrik eşitlikleri yazalım. Buna göre,

cos 2θ = 2 cos2 θ − 1

ve
cos 3θ = 4 cos3 θ − 3 cos θ
eşitliklerini kullanmamız yerinde olacaktır. Eğer x = 2π/5 alırsak, cos 6π/5 =
cos 4π/5 olacaktır. Şimdi denklemi x değişkenine bağımlı olarak yazalım.
Burada
4x3 − 2x2 − 3x + 1 = 0
ise ¡ ¢
(x − 1) 4x2 + 2x − 1 = 0
olacaktır. Buradan √
5−1
x = 1 veya x =
2
90 BÖLÜM 2. KONULAR

olacağından cos(2π/5) 6= 1 ise istenilen cevap


µ ¶ √
2π 5−1
cos =
5 2

olacaktır.

2.10 Cebı̇rde Teleskopı̇k Toplamlar ve Çarpımlar


Bu konuda da cebirde sonsuz toplamlar ve farklarla alakalı soruları çözece-
ğiz. Aslında çözümlerde kullanacağımız genel yöntem oldukça basittir. Soruda
verilen toplamı yada çarpımı, farkların toplamı biçiminde yazarak sonuca ulaş-
maya çalışacağız. Buna göre elde edeceğiz farkların toplamı
n
X
[F (k) − F (k − 1)]
k=2

formunda olacağından ve 2 ≤ k ≤ n − 1 indislerine sahip değerler sadeleşeceği


için, toplamımız
F (n) − F (1)

olacaktır.

Örnek.
n
X
k!k
k=1

toplamını hesaplayınız.

Çözüm. Eğer k! · k = k!(k + 1 − 1) = (k + 1)! − k! değişken değiştirmesini


yaparsak,
Xn
(k + 1)! − k! = (n + 1)! − 1
k=1

olacaktır.
2.10. CEBİRDE TELESKOPİK TOPLAMLAR VE ÇARPIMLAR 91

Örnek.
n
X 1
√ √
k=1
(k + 1) k + k k + 1

toplamını hesaplayınız.

Çözüm. Temel düşünce paydayı rasyonel yapmak olmalıdır. Yani,


³ √ √ ´³ √ √ ´
(k + 1) k − k k + 1 (k + 1) k + k k + 1 = k (k + 1)2 − (k + 1) k 2
= k(k + 1)

olduğuna göre,
n √ √ n µ ¶
X (k + 1) k − k k + 1 X 1 1 1
= √ −√ =1− √
k (k + 1) k k+1 n+1
k=1 k=1

olacaktır.

Örnek. n ∈ Z+ olmak üzere aşağıda verilen eşitsizliği kanıtlayınız.

1 1 2n − 1
n−1< √ √ +√ √ + ··· + q √ <n
1+ 2 2+ 5 (n − 1)2 + 1 + n2 + 1

Çözüm. Soruda verilen eşitsizliği iki yönünüde ispatlamadan evvel, bazı


genellemeler yapalım. Buna göre,
µ q ¶
√ 2
(2k − 1) 2
k + 1 − (k − 1) + 1
2k − 1
q √ =
2
(k − 1) + 1 + k 2 + 1 k 2 + 1 − (k − 1)2 − 1
p q
= k + 1 − (k − 1)2 + 1
2

ise
92 BÖLÜM 2. KONULAR

1 3 2n − 1
= √ √ +√ √ + ··· + q √
1+ 2 2+ 5 (n − 1)2 + 1 + n2 + 1
√ √ √ p q
= 2 − 1 + 5 − 2 + · · · + n + 1 − (n − 1)2 + 1
2
p
= n2 + 1 − 1
olacaktır.
Buna göre, yeni eşitsizliğimiz
p
n−1< n2 + 1 < n
olacaktır ki, bu eşitsizliği göstermek oldukça kolaydır.

Örnek. x, Q(x) polinomunun bir kökü değildir. Buna göre,


P (x) P (x + 1) 1
− =
Q (x) Q (x + 1) x (x + 2)
eşitliğini sağlayan tüm P ve Q polinomlarını bulunuz.

Çözüm. Önce soruda verilen eşitliği düzenleyelim, buna göre


P (x) P (x + 1) 2x + 1 2 (x + 1) + 1
− = −
Q (x) Q (x + 1) 2x (x + 1) 2 (x + 1) (x + 2)
eşitliği elde edilir. Buna göre, n ∈ Z+ ve yeterince büyük bir x değeri için,
P (x + k) P (x + k + 1) 2 (x + k) + 1 2 (x + k + 1) + 1
− = −
Q (x + k) Q (x + k + 1) 2 (x + k + 1) (x + k + 2) 2 (x + k + 1) (x + k + 2)
olacaktır. Eğer eşitlikleri taraf tarafa toplarsak,
P (x) P (x + n) 2x + 1 2 (x + n) + 1
− = −
Q (x) Q (x + n) 2x (x + 1) 2 (x + n) (x + n + 1)
eşitliğini elde ederiz. Burada n ⇒ ∞ iken, P (x+n)/Q(x+n) ifadeside sonsuza
gidecektir. Buna göre,
P (x) 2x + 1
= + c, c ∈ R
Q (x) 2x (x + 1)
2.10. CEBİRDE TELESKOPİK TOPLAMLAR VE ÇARPIMLAR 93

olacaktır. Eğer işlemi devam ettirirsek,


2x (x + 1) P (x) = (2x + 1 + 2cx (x + 1)) Q (x)
eşitliğini elde ederiz. Buradan istenen polinomlar
P (x) = (2x + 1 + 2cx (x + 1)) R (x) ve Q (x) = 2x (x + 1) R (x)
olarak bulunur.

Örnek.
∞ µ
Y ¶
1 1
1− 2 =
n 2
n=2
eşitliğini kanıtlayınız.

Çözüm.
YN µ ¶ N µ
Y ¶µ ¶ N
Y N
1 1 1 n−1 Y n+1
lim 1 − 2 = lim 1− 1+ =
N →∞ n N →∞ n n n n
n=2 n=2 n=2 n=2
1 N +1 N +1
= · =
N 2 2N
olduğuna göre,
N +1 1
lim = bulunur.
N →∞ 2N 2

2.10.1 Çalışma Soruları


1.
n
X ¡ ¢
k! k 2 + k + 1
k=1
toplamını hesaplayınız.
2. a1 , a2 , · · · , an bir aritmetik dizinin elemanları ve ortak farkları d ise
n
X 1
ak · ak+1
k=1

toplamını hesaplayınız.
94 BÖLÜM 2. KONULAR

3.

X 6k
(3k − 2k ) (3k+1 − 2k+1 )
k=1

toplamını hesaplayınız.

4. {xn }∞ 2
n=1 dizisinde x1 = 1/2 , xk+1 = xk + xk ise

1 1 1
+ + ··· +
x1 + 1 x2 + 1 x100 + 1

toplamından küçük en büyük tamsayı değerini bulunuz.

5. Fn bir Fibonacci dizisi olmak üzere F1 = 1, F2 = 1, Fn+1 = Fn + Fn−1


olarak veriliyor. Buna göre aşağıdaki toplamları hesaplayınız.

a.

X Fn
Fn−1 · Fn+1
n=2

b.

X 1
Fn−1 · Fn+1
n=2

6. r r r
1 1 1 1 1 1
1+ 2 + 2 + 1 + 2 + 2 + ··· + 1+ 2
+
1 2 2 3 1999 20002
toplamını hesaplayınız.

7.
1 1 1
√ √ +√ √ + ··· + √ √ > 24
1+ 3 5+ 7 1997 + 9999
eşitsizliğini kanıtlayınız.

8. 1 6 m < n m, n ∈ Z olduğuna göre, aşağıdaki eşitsizliği kanıtlayınız.


¡√ √ ¢ 1 1 1 1 ¡√ √ ¢
2 n + 1 − m < √ +√ +· · ·+ √ +√ < 2 n − m − 1
m m+1 n−1 n
2.10. CEBİRDE TELESKOPİK TOPLAMLAR VE ÇARPIMLAR 95

9.
k
ak = 3
(k − 1) + k 4/3 + (k + 1)4/3
ise
999
X
ak < 50
k=1
eşitsizliğini kanıtlayınız.
10.

X 1
√ <2
n=1
(n + 1) n
eşitsizliğini kanıtlayınız.
11. Fn , fibonacci serisi olmak üzere verilen,

X 1
F2n
n=0

toplamının eşitini bulunuz.


12.

Y n3 − 1 2
3
=
n +1 3
n=2
eşitliğini kanıtlayınız.
13.
∞ µ
X ¶
1
1 + 2n
2
n=0
çarpımını hesaplayınız.
14. L1 = 2, L2 = 2 ve Ln+2 = Ln+1 + Ln , n > 1 olarak tanımlanan Lucas
Dizisi veriliyor. Buna göre,
m
Y
L2k +1 = F2m+1
k=1

eşitliğini kanıtlayınız.
96 BÖLÜM 2. KONULAR

Édouard Lucas12 (1842-1891)


Fransız matematikçi ve bilimadamı olan Lucas
daha çok kendi adı ile anılan Lucas Dizileri ve Fi-
bonacci Dizisinin n. terimini veren formülü ile anı-
lır. Üniversite eğitimini École Normale Supérieure’de
tamamlayan Lucas uzun bir süre Paris Gözleme-
vinde’de çalışmalarını sürdürmüş ve Profesörlük ün-
vanınıda yine aynı şehirde Paris’te almıştır. Ayrıca
N
X
n2 = M 2
n=1

Diphantine denkleminin yegane çözümünün N = 24


ve M = 70 ikilisi olduğunu savunmuştur. Bu ikili-
den başka ikililerin olmadığı ise ancak 1918 yılında
hiperbolik fonksiyonlar kullanılarak kanıtlanabilmiş-
tir. Ayrıca, sayıların asallığı üzerinede çalışmalar ya-
pan Lucas 19 yıllık bir çalışma sonucunda, 2127 − 1 sayısının asal olduğunu
kanıtlamış ve bu kanıtı sonucunda ulaştığı sayı o zamana kadar ulaşılabilen
en büyük Mersenne Asalı olarak kabul edilmiştir. Matematiğin eğlenceli tara-
fıylada alakadar olan Lucas, özellikle Hanoi Kuleleri’ni keşfetmesi ve çözüm
yöntemleri geliştirmesi bulmaca meraklılarının büyük saygısını kazanmıştır.

2.10.2 Çözümler
1.
n
X n h
X i
¡ ¢
k! k 2 + k + 1 = (k + 1)2 − k k!
k=1 k=1
n
X
= [(k + 1)! (k + 1) − k!k]
k=1
= (n + 1)! (n + 1) − 1 olacaktır.
12
François Édouard Anatole Lucas
2.10. CEBİRDE TELESKOPİK TOPLAMLAR VE ÇARPIMLAR 97

2. µ ¶
n
X n n
1 1 X ak+1 − ak 1X 1 1
= = −
ak ak+1 d ak ak+1 d ak ak+1
k=1 k=1 k=1
olacağından, toplam
1 an+1 − a1
d an+1 a1
olacaktır. an+1 − a1 = nd olduğuna göre
1 nd n
=
d (nd + a1 ) a1 (nd + a1 ) a1
olacaktır.

3. Kesri parçalara bölerek çözüme gitmek yerinde olacaktır. Buna göre,


6k A B
= k − k+1
(3k − 2k ) (3k+1 − 2k+1 ) 3 − 2k 3 − 2k+1
eşitliğinde ³ ´ ³ ´
3k+1 − 2k+1 A − 3k − 2k B = 6k

olacaktır. Buradan A = 2k ve B = 2k + 1 olarak kolaylıkla bulunabilir.


Buna göre,

X 6k 2 2k+1
= − lim =2
(3k − 2k ) (3k+1 − 2k+1 ) 3 − 2 k→∞ 3k+1 − 2k+1
k=1

olarak bulunur.
Not. Benzer biçimde A = 3k ve B k + 1 bulunabilirdi, ancak bu değerler
sonucu değiştirmezdi. Deneyip görebilirsiniz.

4. İndirgemeli dizi çözüm yöntemini kullanalım. Buna göre, xk+1 = x2k + xk


ise
1 1 1 1
= = −
xk+1 xk (xk + 1) xk xk + 1
olacağından
1 1 1 1 1
+ + ··· + = −
x1 + 1 x2 + 1 x100 + 1 x1 x101
98 BÖLÜM 2. KONULAR

olacaktır. x1 = 1/2 ve 0 < 1/x101 < 1 olduğuna göre istenen değer 1


olacaktır.

5. a.

X ∞
X X∞ µ ¶
Fn Fn+2 − Fn−1 1 1
= = −
Fn−1 Fn+1 Fn−1 Fn+1 Fn−1 Fn+1
n=2 n=2 n=2
µ ¶
1 1 1 1 1 1
= lim + − − = + = 2 olacaktır.
n→∞ F1 F2 Fn Fn+1 F1 F2

b.

X ∞
X X∞
1 Fn Fn+1 − Fn−1
= =
Fn−1 Fn+1 F F F F F F
n=2 n=2 n−1 n n+1 n=2 n−1 n n+1
X∞ µ ¶
1 1
= −
Fn−1 Fn Fn Fn+1
n=2
µ ¶
1 1 1
= lim − = =1
n→∞ F1 F2 Fn Fn+1 F1 F2

olacaktır.

6. n ∈ Z+ olmak üzere,
¡ 2 ¢
1 1 n2 (n + 1)2 + (n + 1)2 + n2 n +n+1
1+ 2 + = =
n (n + 1)2 n2 (n + 1)2 n2 (n + 1)2

ise s
1 1 n2 + n + 1 1
1+ 2
+ 2 = 2
=1+
n (n + 1) n +n n (n + 1)

olacaktır. Buna göre,


1997
1
¶ Xµ
1999
1 1

1
1+ = 1+ − = 2000 −
n (n + 1) n n+1 2000
n=1 n=1

olur.
2.10. CEBİRDE TELESKOPİK TOPLAMLAR VE ÇARPIMLAR 99

7. Teleskopik toplam yapmak mantıklı görünsede, bazı elemanlar eksik ol-


duğundan çözüme gidemeyiz. Buna göre eşitsizliğin sol kısmı,

1 1 1
√ √ +√ √ + ··· + √ √
3+ 5 7+ 9 9999 + 10001

toplamından küçüktür. Eğer sorudaki eşitsizlikle son yazdığımız eşitsiz-


liği toplarsak

1 1 1 1
√ √ +√ √ +√ √ + ··· + √ √ > 48
1+ 3 3+ 5 5+ 7 9999 + 10001

olacaktır. Şimdi artık teleskopik toplamı kullanabilriz. Buna göre,

1 ³√ √ √ √ √ √ √ √ ´ 1 ³√ ´
3 − 1 + 5 − 3 + 7 − 5 + · · · + 10001 − 9999 = 10001 − 1 > 48
2 2
olacaktır.
¡√ ¢ ¡√ ¢
8. k ∈ R için k+1−k k + 1 + k = 1 olduğuna göre,
³√ √ ´ 2 1
2 k+1− k = √ √ <
k+1+ k k

ve
1 2 ³√ √ ´
√ <√ √ =2 k− k−1
k k+ k−1
olacaktır. Buna göre,
³√ √ ´ 1 ³√ √ ´
2 k+1− k < √ <2 k− k−1
k

olacaktır. Eğer bu eşitsizlikleri toplarsak,

¡√ √ ¢ 1 1 1 1 ¡√ √ ¢
2 n + 1 − m < √ +√ +· · ·+ √ +√ < 2 n − m − 1
m m+1 n−1 n

olacağından soruda verilen eşitsizlik kanıtlanmış olur.


100 BÖLÜM 2. KONULAR

9. ilk önce an ifadesinin paydasını küçültelim ve rasyonel hale getirelim.


Buna göre, k 3 yerine (k − 1)2/3 (k + 1)2/3 yazılırsa,

k
ak <
(k − 1) + (k − 1) (k + 1)2/3 + (k + 1)4/3
4/3 2/3
³ ´
k (k + 1)2/3 − (k − 1)2/3 1³ ´
2/3 2/3
= = (k + 1) − (k − 1) olacağından
(k + 1)2 − (k − 1)2 4

1 X³ ´
999
X 999
an = (k + 1)2/3 − (k − 1)2/3
4
k=1 k=1
1³ ´ 1
= 10002/3 + 9992/3 − 12/3 − 02/3 < (100 + 100 − 1) < 50 bulunur.
4 4

10. Verilen toplamı düzenlersek


√ √ √
1 n n n
√ = = −
n (n + 1) n (n + 1) n n+1

olacağından yeni toplamımız


∞ √ √
X n− n−1
1+
n
n=2

olacaktır. Bu toplam teleskopik değildir, ancak üstten sınırlıdır. Öyleki


∞ √ √ ∞ µ ¶
X n− n−1 X 1 1
1+ √ √ =1+ √ −√
n=2
n n−1 n−1 n
k=2

olacakır. Ve bu toplam da 2’ye yakınsar. İspat tamamlanır.

11. Sorunun çözümüne tümevarım ile başlayalım. Buna göre, tümevarım


yöntemi kullanılarak

F2m Fm−1 − F2m−1 Fm = (−1)m Fm , m > 1


2.10. CEBİRDE TELESKOPİK TOPLAMLAR VE ÇARPIMLAR 101

olduğu bulunabilir. Eğer m = 2n−1 alınırsa


F2n F2n−1 −1 − F2n −1 F2n−1 = F2n−1 , n > 2
veya
1 F n−1 F2n −1
= 2 −1 − , n>2
F2n F2n−1 F2n
olacağından
∞ µ ¶ √
X 1 1 1 F1 F2N −1 1 7− 5
= + + lim − =3− √ =
F2n F1 F2 N →∞ F2 F2N 5+1 2
n=0 2
olacaktır.
12.
N N ¡ ¢
Y n3 − 1 Y (n − 1) n2 + n + 1
lim = lim
N →∞ n3 + 1 N →∞ (n + 1) (n2 − n + 1)
n=2 n=2
YN N
n − 1 Y (n + 1)2 − (n + 1) + 1
= lim
N →∞ n+1 n2 − n + 1
n=2 n=2
³ ´
1 · 2 · (N + 1)2 − (N + 1) 4 + 1 2
= lim =
N →∞ 3N · (N + 1) 3
olarak bulunur.
13. Eğer soruda verilen ifadeyi açarsak
Y∞ µ ¶ µ ¶ ∞ µ ¶
1 1 Y 1
1 + 2n = 2 lim 1 − 0 1 + 2n
2 N →∞ 2 2
n=0 n=0

olacaktır. Buradan,
µ ¶µ ¶ µ ¶
1 1 1
1 − 2n 1 + 2n = 1 − 2n+1
2 2 2
olacağından, çarpımımızı teleskopik hale getirebiliriz. Buradan çarpım
1
1−
22N +1
1
olacaktır. N → ∞ ve 1− → 1 ise çarpımın sonucu 2 olarak bulunur.
22N +1
102 BÖLÜM 2. KONULAR

14. Eğer kontrol edilirse Fibonacci ve Lucas dizilerinin


2 2
Fn+1 + Fn−1 = Ln+1 ve F2n = Fn+1 − Fn−1 , n>1

için sağlandığı görülebilir. Öyleyse,

F2n = (Fn+1 + Fn−1 ) (Fn+1 − Fn−1 ) = Ln+1 Fn

olacaktır. Buna göre,


F2n
Ln+1 = , n>1
Fn
olacaktır. Öyleyse,
m
Y m
Y F2k+1 F m+1
L2k +1 = = 2 = F2m+1
F2k F2
k=1 k=1

olacaktır. İspat tamamlanır.

2.11 Tamdeğer Fonksı̇yon Problemlerı̇


x ∈ R olmak üzere, x değerini geçmeyen en büyük tamsayı değeri [x] ile
gösterilir. Bu foksiyona tam değer fonsiyon (floor function) denir. Buna göre
x − 1 < [x] ≤ x olduğu açıktır. Bu ifade ayrıca [x] ≤ x < [x] + 1 olarakta
yazılabilir. Ayrıca sayının ondalıklı kısmıda {x} ile temsil edilir. Buna göre;
{x} = x − [x] dir. Buna göre x = [x] + {x}, 0 ≤ {x} < 1 dir. Tam değer
fonksiyonun en temel teoremleri aşağıda verilmiştir.

Teorem. α, β ∈ R, a ∈ Z, n ∈ N ise;

1. [α + a] = [α] + a

2. [ αn ] = [ [α]
n ]

3. [α] + [β] ≤ [α + β] ≤ [α] + [β] + 1

Kanıt.
2.11. TAMDEĞER FONKSİYON PROBLEMLERİ 103

1. Varsayalım m = [α + a] olsun. Buna göre m ≤ α + a < m + 1 dolayısıyla


da m − a ≤ α < m − a + 1 olacaktır. m − a = [α] olduğuna göre ispat
tamamlanmış olur.
α
2. n ifadesini αn = [ αn + θ], 0 ≤ θ < 1 olarak yazalım. n · [ αn ] ifadesi olduğnan
artık elimizde 1 durumu oluştu. Yani;

[α] = [n[α/n] + nθ] = n[α/n] + [nθ] olur.

Şimdi, 0 ≤ [nθ] < nθ < n ve bundan dolayı 0 ≤ [nθ/n] < 1 olur.Eğer


Θ = [nθ]/n olduğunu varsayarsak [α] α
n = [ n ] + Θ, 0 ≤ Θ < 1 olur ki, buda
zaten istenen sonuçtur.

3. Varsayalım α = n + r, β = m + t ve 0 ≤ r < 1, 0 ≤ t < 1 olsun.

[α] + [β] = n + m ≤ [n + r + m + t] = [α + β] olur.


= m + n + [r + t] ≤ n + m + 1
= [α] + [β] + 1 olur.

Örnek. [ x3 ] = x
2 + 1 denklemini sağlayan x tamsayılarının toplamını
bulunuz.

Çözüm. Varsayalım [ x3 ] = t olsun. Buna göre, t ≤ x3 < t + 1 ise 3t ≤


x < 3t + 3 olmalıdır. [ x3 ] = t = x2 + 1 olduğuna göre x = 2t − 2 olur. Bu
son eşitliği eşitsizliğimizde yerine koyarsak 3t ≤ 2t − 2 < 3t + 3 ise −5 < t ≤
−2 bulunur. t = −2, −3, −4 olabileceğinden x değerleride −8, −10, −6 olarak
bulunur. istenen cevap −24 olur.

Teorem.(Hermite Özdeşliği) [nx] = [x] + [x + n1 ] + [x + n2 ] + · . . . · +[x + n−1


n ]
dir.
Kanıt. Eğer teoremde verilen eşitliği düzenlersek;
n
X k
[x + ] = [nx]
n
k=0
104 BÖLÜM 2. KONULAR

ifadesini elde ederiz. Öyleyse bu eşitliği ispatlamamız yeterlidir. x = [x] + {x}


olduğunu zaten biliyoruz. Buna göre; {1, 2, 3, . . . , n} kümesinin elemanı olan
kesin bir k 0 değeri vardır ki;
k0 − 1 k0
[x] = [x + ] ≤ x < [x + ] = [x] + 1
n n
ise
k0 − 1 k0
0 = [{x} + ] ≤ {x} < [{x} + ] = 1
n n
olacaktır. Buna göre;
k0 k0 − 1
1− ≤ {x} < 1 − ⇒ n − k 0 ≤ n{x} < n − k 0 + 1 olur.
n n
En başta yazdığımız eşitliği kullanırsak
n−1 kX0
−1 n−1
X k X
[x + ] = [x] + ([x] + 1) = n · [x] + n − k 0
n 0
k=0 k=0 k=k
= n[x] + [n{x}]
= [n[x] + n{x}] = [nx] olur.

Örnek. [x]+[2x]+[4x]+[8x]+[16x]+[32x] = 12345 denklemini sağlayan


x değerlerini bulunuz.

Çözüm. Bu soruda amacımız önce, denkliği sağlayan x değerlerinin var-


lığını araştırmak ve bu değerleri bulmak olacaktır. Konunun en başında ver-
diğimiz teoremlerden biliyoruz ki, x − 1 < [x] ≤ x olacaktır. Buna göre;

x−1+2x−1+4x−1+. . .+32x−1 < [x]+[2x]+. . .+[32x] ≤ x+2x+4x+. . .+32x

ise 63x − 6 < 12345 ≤ 63x ve x değeride 195 < x < 196 arasında olacaktır.
Eğer x değerini 2 lik tabanda yazarsak x = 195 + a21 + a222 + . . ., ak = 0 veya 1.
Buna göre;
[2x] = 2 · 195 + a1
[4x] = 4 · 195 + 2a1 + 2a2
[8x] = 8 · 195 + 4a1 + 2a2 + a3
2.11. TAMDEĞER FONKSİYON PROBLEMLERİ 105

[16x] = 16 · 195 + 8a1 + 4a2 + 2a3 + a4


[32x] = 32 · 195 + 16a1 + 8a2 + 4a3 + 2a4 + a5

Eğer bu değerleri alt alta toplarsak 63·195+31·a1 +15·a2 +7a3 +3·a4 +a5
olacaktır yani; 31 · a1 + 15 · a2 + 7 · a3 + 3 · a4 + a5 = 60 olur. Ancak 0 ≤ ai ≤ 1
olduğundan ifade en fazla 31 + 15 + 7 + 3 + 1 = 57 değerini alabilir. Çelişki
vardır. Çözümü sağlayan bir x değeri yoktur.

Örnek. [x], x in tamdeğer foksiyonu olmak üzere, {x} = x − [x] olarak


tanımlansın. Her x reel sayısı için x = f (x) − 2 · f ({x}) eşitliğini sağlayan f
fonksiyonunun x = − 13
5 noktasındaki değerini bulunuz.

Çözüm. {− 13 13
5 } = − 5 − (−3) =
2
5 oluğundan; x = 2
5 için

2 2 2 2 2
= f ( ) − 2 · f ({ }) ⇒ f ( ) = −
5 5 5 5 5

olacaktır. x = − 13
5 için

13 13 13 13 2
− = f (− ) − 2 · f ({− }) = f (− ) − 2 · f ( )
5 5 5 5 5
olduğuna göre
13 17
f (− )=−
5 5
olur.

Teorem. a ve b aralarında asal doğal sayılar olmak üzere;

a 2a 3a (a − 1) · b (a − 1) · (b − 1)
[ ] + [ ] + [ ] + ... + [ ]=
b b b b 2
eşitliği vardır.
Kanıt. Aslında teoremde verilen ifadeyi düzenlersek;
a−1
X b−1
X
kb ka (a − 1)(b − 1)
[ ]= [ ]=
a b 2
k=1 k=1
106 BÖLÜM 2. KONULAR

eşitliğini elde ederiz. Öyleyse bu eşitlikleri ispatlamamız yeterli olacaktır.Varsayalım


analitik düzlem üzerinde, köşe koordinatları (0, 0), (0, b), (a, 0), (a, b) olan bir
dikdörtgenimiz olsun. Bu dikdörtgen üzerinde (a − 1)(b − 1) tane latis (lattice)
noktası, yani koordinat bileşnleri tamsayıar olan noktalar vardır. Bu dikdört-
geni y = xb a doğrusu ile iki parçaya ayıralım. Bu doğru üzerindeki başlangıç ve
bitiş noktaları hariç hiçbir noktanın koordinatlarının tamsayı olmadığı açıktır.
n
Eğer böyle bir nokta varsa, mesela (m, n), 0 < m < a, 0 < n < b, m = ab
n
olmalıdır. Demek ki m kesri, ab sadeleşebilen kesrinin bir sadeleşmi halidir.
Ancak bu durum açık bir çelişkidir. Çünkü (a, b) = 1 kabul etmiştik.
Lk = (k, kb a ), 1 ≤ k ≤ a − 1 oktalarının her biri doğrunu üzerindedir. Buna
göre [ a ] ifadesi (k, 0) dan (k, kb
kb
a ) noktasına giden doğrunun üzerindeki latis
noktalarıdır. Yani
a−1
X kb
[ ]
a
k=1
ifadesi dikdörtgenin alt yarısındaki latis noktalarının sayısıdır. Benzer biçimde,
b−1
X ka
[ ]
b
k=1

ifadeside üst yarıdaki noktalar olduğuna göre ve bu noktalar alt ve üst yarıda
eşit sayıda bulunabildiğine göre toplam nokta sayısının yarısı
(a − 1)(b − 1)
2
sitenen cevap olur. ispat tamamlanır.
√ √ √
Örnek. n ∈ Z+ olmak üzere, [ n + n + 1] = [ 4n + 2] eşitliğini
kanıtlayınız.
√ √ √
√ Çözüm. İfadenin karesini alırsak n + n + 1 toplamının 4n + 1 ile
4n + 3 ifadelerinin arasında olduğunu görmek zor değildir. Tam kare ifadeler
mod4 altında 0 veya 1 olduklarından√4n + 2 ve 4n √ + 3 ifadelerinin birer tam
kare olmadıkları açıktır. Buna göre [ 4n + 2] = [ 4n + 3] olduğu açıktır.

Teorem.(De - Polignac ve Legendre Formülü) n! ifadesini bölen p asal


2.11. TAMDEĞER FONKSİYON PROBLEMLERİ 107

sayısının en büyük kuvveti α ise



X n
α= [ k]
p
k=1

eşitliği vardır.

Örnek. 999! ifadesinin ondalık yazılımının sonunda kaç tane 0 vardır?

Çözüm. Sıfırların sayısı çarpımdaki 2 ve 5 sayılarına bağlıdır buna göre


5α |999! olmasını sağlayan en büyük α değeri istenen cevap olacaktır. Buna
göre;
999 999 999 999
[ ] + [ 2 ] + [ 3 ][ 4 ] = 179 + 39 + 7 + 1 = 246 olur.
5 5 5 5

Örnek. µ ¶
1000
7|
500
ifadesi doğruluğunu araştırınız.

Çözüm. 7α |1000! olmasını sağlayan en büyük α değeri‘;


1000 1000 1000
[ ] + [ 2 ] + [ 3 ] = 164
7 7 7
olacaktır. Benzer biçimde 7α |500! olmasını sağlayan en büyük α değeride 82
bulunur. µ ¶
1000 1000!
=
500 (500!)2
olduğundan, bu ifadeyi bölen 7 nin en büyük kuvveti 164 − 2 · 82 = 0 olduğun-
dan, soruda verilen ifade doğru değildir.

2.11.1 Çalışma Soruları


1. √ √ p
3 3 3
[ 1] + [ 2] + . . . + [ x3 − 1] = 400
denkleminin doğal sayılardaki çözüm kümesinin elemanlarını bulunuz.
108 BÖLÜM 2. KONULAR


2. (6 + 35)1980 açılımındaki virgülden sonraki ilk 1000 basamağın 9 oldu-
ğunu gösteriniz.
3. [x2 ] = [x] denkleminin çözüm kümesini bulunuz.
4.
8 n 7
< <
15 n+k 13
eşitsizliğini sağlayan en büyük n değeri kaçtır? (k değeri tektir).
5. r ∈ R olmak üzere;
91
X k
[r + ] = 546
100
k=19
ise [100r] ifadesinin eşitini bulunuz.
6. f (n) fonksiyonu n1/4 sayısına en yakın tamsayıyı temsil etmek üzere,
1995
X 1
f (n)
n=1

toplamının tamdeğerini bulunuz.


7. [x] ifadesi x sayısının tamdeğerini temsil etmek üzere
12 22 19972
[ ], [ ], . . . , [ ]
1998 1998 1998
dizisindeki farklı tamsayıların sayısını bulunuz.

2.11.2 Çözümler
1. x ∈ N olduğuna göre;
√3

3
1, . . . , 7 → 1 · 7 = 7
√3
√3
8, . . . , 26 → 2 · 19 = 38
√3
√3
27, . . . , 63 → 3 · 37 = 111

3
√3
64, . . . , 124 → 4 · 61 = 244
ise 7 + 38 + 111 + 244 = 400 ise tek çözüm x = 5 olmalıdır.
2.11. TAMDEĞER FONKSİYON PROBLEMLERİ 109

√ √ √
2. (6 + 35)1980 + (6 − 35)1980 = 2k, k ∈ Z olur. √ Burada 6 − 35 <
1 1
10 olmalıdır aksi durumda yani eğer 10 < 6 − 35 olursa, iki tarafın
karesinden 3500 < 3481 olur.
√ 1980 1 √ 1980
0 < (6 − 35) < 10−1980 ⇒ 2k − < (6 +
35) < 2k
101980 √ 1980
⇒ 2k − 1 + 0, 999 . . . 9 < (6 + 35) < 2k
| {z }
1979 tane 9

3. x2 ≥ 0 ise [x2 ≥] olur. x ∈ [0, 1] aralığındaki tüm sayıların sağladığı


açıktır. Biz (1, 2) aralığına bakalım;
x = 1 + {x}, [(1 + {x}2 )] = [1 + {x}] ise [1 + 2{x} + {x}2 ] = [1 + {x}]
[1 + 2{x} + {x}2 ] = 1 0 ≤ 2{x} + {x}2 < 1 ⇒ {x} > 0 veya {x}2 +
2{x} −√1 < 0 durumları oluşur. Buradan eğer √ y = {x} olarak alırsak
y ∈ (0, 2 − 1) olmalıdır. Buradan da, x√∈ (0, 2) arasında olur. 0 zaten
ahil olduğundan istenilen aralık x ∈ [0, 2] bulunur.

4. Verilen ifadeyi düzenlersek


6n 7n
<k<
7 8
olacaktır. Fakat buradan 7n 6n n
8 − 7 = 56 bulunur. Buna göre eğer n > 112
6n 7n
ise, bu aralıkta kesinlikle, yani ( 7 , 8 ) aralığında, iki tamsayı olacaktır.
n = 112 için ise, 96 < k < 98 olacağından cevap 112 olmalıdır.

5. 546 = 7 · 73 + 35 = 38 · 7 + 35 · 8 olur. Demek ki

7 = [r + 0, 19] = [r + 0, 20] = [r + 0, 21] = . . . [r + 0, 56]


8 = [r + 0, 57] = [r + 0, 58] = . . . = [r + 0, 91]

olacağından 7, 43 ≤ x < 7, 44 ise 74, 3 ≤ 100r < 74, 4 ve [100r] = 74


olur.

6. f (k) ifadesi 1, 2, 3, . . . , 7 değerlerinden birini alır. Sınır değerler


1 1 1
(n + )4 = n4 + 2n3 + · (3n2 + n) +
2 2 16
110 BÖLÜM 2. KONULAR

olduğuna göre

5 tanesi : 1
34 tanesi : 1/2
111 tanesi : 1/3
260 tanesi : 1/4
505 tanesi : 1/5
870 tanesi : 1/6
210 tanesi : 1/7

olacağına göre, toplam 400 olur.


2
998 9992
7. [ 1998 ] = 498 < 499 = [ 1998 ] olduğuna göre önce k = 1, 2, 3, . . . , 998 ve
k = 999, 1000, . . . , 1997 durumlarını ayrı ayrı inceleyelim. Buna göre,
k = 1, 2, 3, . . . , 997 için
(k + 1)2 k2 2k + 1
− = <1
1998 1998 1998
olacaktır. Buna göre
12 9982
[ ] = 0, 1, 2, . . . , 498 = [ ]
1998 1998
dizisinde her bir sayı en az bir defa tekrarlanacaktır. Buna göre dizinin
bu bölümünde 499 terim vardır. k = 999, 1000, . . . , 1996 ise
(k + 1)2 k2 2k + 1
− = >1
1998 1998 1998
ise burada da, 1997−999+1 = 999 farklı terim vardır. Buna göre dizideki
farklı tamsayıların sayısı 1498 olacaktır.

2.12 Bölünebı̇lme ve Asal Sayılar


Tanım. a 6= 0, b ∈ Z olmak üzere, eğer a · c = b eşitliğini sağlayan bir
c ∈ Z bulunuyorsa bu duruma a böler b denir ve a|b olarak yazılır.
2.12. BÖLÜNEBİLME VE ASAL SAYILAR 111

Teorem.

1. a, b, c, m, n ∈ Z olmak üzere c|a, c|b ise c|(a · m + b · n) dir.

2. x, y, z ∈ Z olmsk üzere; x|y, y|z ise x|z dir.

Kanıt.

1. s, t ∈ Z olmak üzere varsayalım s · c = a, t · c = b olsun. Buna göre


am + bn = c(sm + tn) ise c|(am + bn) olur.

2. Ve varsayalım u, v ∈ Z olmak üzere xu = y, yv = z olsun. Buna göre


xuv = z ise x|z olduğu açıktır. Ayrıca eğer a|b ve b 6= 0 ise ≤ |a| < |b|
durumu vardır.

Örnek. (n + 1)|(n2 + 1) durumunu doğrulayan tüm n ∈ Z+ değerlerini


bulunuz.

Çözüm. n2 + 1 = n2 − 1 + 2 = (n − 1) · (n + 1) + 1 ise (n − 1) · (n + 1)
sayısı (n + 1) ise bölünür. Bu durumda (n + 1)|2 olmalıdır. |n + 1| < |2| ise
n = 1 olur.

Örnek. n herhangi bir doğal sayı olmak üzere; n3 + 2n sayısının 3 ile


bölünebildiğini gösteriniz.

Çözüm. n değeri 3k + 1, 3k + 2 ve 3k formunda olabilir.


n = 3k ise (3k)3 + 2 · (3k) = 3 · (32 · k 3 ) + 3 · (2k) ise 3|n3 + 2n
n = 3k +1 ise (3k +1)3 +2·(3k +1) = (3k +1)3 +2·3k +2 ≡ 1+0+2 ≡ 0(mod3)
ise 3|n3 + 2n
n = 3k +2 ise (3k +2)3 +2·(3k +2) = (3k +2)3 +2·3k +4 ≡ 2+0+1 ≡ 0(mod3)
ise 3|n3 + 2n.

Örnek. 7|3x + 2 ise 7|(15x2 − 11x − 14) olduğunu kanıtlayınız.

Çözüm. 15x2 − 11x − 14 = (3x + 2) · (5x − 7) = 7s · (5x − 7) ise kanıt


tamamlanmış olur.
112 BÖLÜM 2. KONULAR

Örnek. a, b, c doğal sayılar olmak üzere, a2 + b2 = c2 eşitliği sağlanmak-


tadır. Buna göre, a, b ve c den en az birinin 3 ile bölünebildiğini gösterelim.

Çözüm. Eğer a, b ve c değerleri için bir genellme yaparsak, varsayalım


hiçbiri 3 ile tam bölünmesin. Öyleyse, a = 3k + 1, b = 3t + 1, c = 3r + 1 olsun.
Buna göre,
(3k + 1)2 + (3t + 1)2 ≡ 1 + 1(mod3)
ise c2 ≡ (mod3) olur, ancak tamsayının karesi bu denkliği sağlamaz.

Teorem. Ardışık n tamsayının çarpımı n! ile tam bölünür.


Kanıt. Varsayalım ardışık n tamsayımız m + 1, m + 2, m + 3, . . . , m + n for-
munda olsun ve bu dizinin her bir elemanı pozitif tamsayı olsun. Buna göre,
göstermemiz gereken durum;

n!|m + 1, m + 2, m + 3, . . . , m + n

dir. Burada binom katsayıların tamsayı olma özelliğini kullanırsak


µ ¶
m+n (m + n)! (m + n)! (m + 1) · (m + 2) · (m + 3) · . . . · (m + n)
= = = ∈Z
n (m + n − m)!(m + n − n)! n!m! n!

ise ispat tamamlanır.

Örnek. 6|n3 − n olduğunu gösteriniz.

Çözüm. n3 − n = n · (n2 − 1) = n · (n − 1) · (n + 1) ise daha önce verilen


teoremden dolayı doğrudur.

Teorem. n ∈ Z+ olmak üzere, n bir bileşik sayı13 ise, n sayısının p ≤ n
olacak şekilde bir asal çarpanı vardır.

Kanıt. Varsayalım n = a · b, 1 < a ≤ b olsun. Eğer bu iki değerde n de
√ √
büyükse, n = a · b = n · n = n olur ki, bu bir çelişkidir. Demek ki, n

sayısının 1 den farklı ve n den küçük eşit olan bir çarpanı vardır. Buna göre,

p bir asal çarpan olmak üzere, bu değerde n den küçük olur.
13
2 den büyük olan ve asal olmayan sayılar
2.12. BÖLÜNEBİLME VE ASAL SAYILAR 113

Örnek. 101 sayısının asal olup olmadığını anlamak için bu sayıyı en az


kaç sayıya bölmeliyiz.

Çözüm. 101 ≈ 10 ise p = 2, 3, 5, 7 olmalıdır. Toplamda 4 sayı yeterlidir.

Örnek. Elemanlarından hiçbirisi asal sayı olmayan gelişigüzel uzunlukta


bir ardışık sayı dizisi bulunabilir mi?

Çözüm. k ∈ Z, k ≥ 2 olmak üzere

k! + 2, k! + 3, . . . , k! + k

dizisinin elemanları ardışıktır ve asal değildir. Yani

A = (n + 1)! + 2, (n + 1)! + 3, . . . , (n + 1)! + (n + 1)

dizisini ele alalım. 2 ≤ k < n + 1 bağıntısını sağlayan herbir k tamsayısı için


k|(n + 1)! dir. k|(n + 1)! + k ise ispat tamamlanır. Mesela

23! + 1, 23! + 2, . . . , 23! + 28

dizisindeki sayılardan hiçbirisi asal değildir.

Not. p > 3 olmak üzere bütün asal sayılar, n ∈ Z olmak üzere p = 6n ± 1


formundadır.

Örnek. p ve p2 + 2 asal sayılar ise p3 + 2 ninde asal olduğunu gösteriniz.

Çözüm. p değeri tek olmalıdır. p = 3 için p2 + 2 = 11 ve p3 + 2 = 29


dur. Burada p > 3 ise p = 6n ± 1 olarak alınırsa, p2 + 2 sayısı 3 ile bölünebilir.
Demek ki, p = 3 dışında soruda verilen durumu doğrulayan bir sayı yoktur.
Tek çözüm, p = 3 olur.

Örnek. 19 dan 92 ye kadar olan sayılar yanyana yazılarak elde edilen,

19202122 . . . 9192

sayısını bölen 3 ün en büyük kuvvetini bulunuz.


114 BÖLÜM 2. KONULAR

Çözüm. 9 ile bölübeilme kuralına bakalım 19 + 20 + 21 + . . . + 92 = 372 · 3


ise bu sayı 9 ile bölünemez, sadece 3 ile bölünebilir.

Örnek.
p3 + p2 + 11p + 2
ifadesini asal yapan en büyük p asal sayısını bulunuz.

Çözüm. p = 3 için, ifademiz 71 değerini alır. 3 ten büyük değerler için


yani p = 3k + 1 ve p = 3k + 2 değerlerinde ifademiz sırasıyla 3 · (9k 3 + 12k 2 +
16k + 5) ve 9 · (3k 3 + 7k 2 + 9k + 4) değerlerini alır. Ancak bu değerlerin ikiside
asal değildir. Demek ki tek değer p = 3 tür.

Örnek. 2a + 1 ve 3a + 1 birer tamkare ise 5a + 3 sayısının asal olmadığını


gösteriniz.

Çözüm. 2a + 1 = x2 ve 3a + 1 = y 2 ise

5a + 3 = 4 · (2a + 1) − (3a + 1) = 4x2 − y 2 = (2x − y)(2x + y)

durumu vardır. 5a+3 asal ise, 2x−y = 1 ve 2x+y = 5a+3 olmalı. 4x = 5a+3
ise 5x2 −8x+3 = 0 olur. Burada bulunan her bir kök tamsayı olmak zorundadır.
Buradan istenen tamsayı x değeri −1 bulunur. Buna göre, sadece (1, 1, 3) için
durum sağlanır diğer durumlarda sağlanmaz.

2.12.1 Çalışma Soruları


1. n ∈ Z+ olmak üzere,
(n + 10)|(n3 + 100)
olmasını sağlayan en büyük n tamsayı değerini bulunuz.

2. n5 −5n3 +4n iadesinin daima 120 ile kalansız bölünebileceğini gösteriniz.

3. p > 3 ve p asal bir sayı olmak üzere, 24|(p2 − 1) olduğunu gösteriniz.

4. Herhangi bir tamsayının karesinin 4k veya 4k + 1 formunda olduğunu


gösteriniz.
2.12. BÖLÜNEBİLME VE ASAL SAYILAR 115

5. 11, 111, 1111, 11111, . . . dizisindeki sayılardan hiçbirinin tamkare olmadı-


ğını kanıtlayınız.

6. Bir tamsayının 5. kuvvetine eşit olan 6p + 1 formundaki tüm p asal


sayılarını bulunuz.

7. 21 · p + 1 sayısını tamkare yapan kaç p asal sayısı vardır?

8. 105 sayısı bir kaç ardışık doğal sayının toplamı olarak kaç farklı biçimde
yazılabilir?

9. Kaç tane p asal sayısı için p2 + 17 sayısının 4 tane farklı pozitif böleni
vardır?

10. n > 3 olmak üzere, (n − 3)|n2 − n durumunu doğrulayan bütün n tam-


sayılarını bulunuz.

11. {1, 2, 3, 4, . . . , 23, 24, 25} kümesinden en az kaç eleman atılmalı ki geriye
kalan elemenların çarpımı bir tam kareye eşit olsun.

12. 13p + 289 sayısının bir tamsayı olmasını sağlayan en büyük p asal sa-
yısını bulunuz.

13. a. N sayısı b tabanında 777 dir. N nin bir tamsayının dördüncü kuv-
vetine eşit olmasını sağlayan en küçük b tamsayısını bulunuz.
b. n bir tamsayı olsun. n2 nin onlar basamağındaki rakam 7 ise, n2
nin birler basamağındaki rakamı bulunuz.

14. n+1 ve 16n+1 ifadelerinin ikisinide tamkare yapan n ≥ 1 tamsayılarının


sayısını bulunuz.

15. (n + 1) · (n4 + 2n) + 3 · (n3 + 57) ifadesinin (n2 + 2) ile bölünmesini


sağlayan en büyük n tamsayısının değerini bulunuz. (ipucu: polinom
bölmesini kullanınız)

16.
49, 4489, 444889, 44448889, . . . , |44 .{z
. . 44} 88
| .{z
. . 88} 9, . . .
n tane 4 n-1 tane 8

dizisindeki her bir terimin tamkare olduğunu kanıtlayınız.


116 BÖLÜM 2. KONULAR

17. 111
| {z. . . 11} sayısının bileşik sayı olduğunu gösteriniz.(ipucu: 221 = 17·13)
221 tane 1

. . 1111}, b = 1 000
18. a = |1111 .{z | .{z
. . 000} 5 ise a · b + 1 sayısının bir tamkare
m tane 1 m-1 tane 0
olduğunu gösteriniz.

19.
111
| .{z
. . 111} − 222
| .{z
. . 222}
2n tane 1 n tane 2

farkının bir tamkare olduğunu gösteriniz.

20. 2x2 − x − 36 ifadesini bir asal sayının karesi yapan bütün x tamsayılarını
bulunuz.

21. En az iki ardışık pozitif tamsayının toplamı şeklinde yazılabilen pozitif


bir tamsayının 2 nin kuvvetine eşit olamayacağını gösteriniz.

22. 1 den 100 e kadar (100 dahil) sayılardan kaç tanesi, iki veya daha fazla
ardışık pozitif tamsayının toplamı olarak yazılabilir?

23. İlk terimi n olan 98 ardışık sayının toplamı 192 ile bölünmektedir. Buna
göre en küçük n ∈ Z değerini bulunuz.

24. a · b = c · d eşitliğini sağlayan a, b, c, d > 0 tamsayıları verildiğinde a2 +


b2 + c2 + d2 toplamının hiç bir zaman asal olamayacağını gösteriniz.

25. 2903n −803n −464n +261n ifadesinin n in alabileceği herhani bir tamsayı
değeri için 1897 ile bölünebileceğini gösteriniz.

26. 1492n − 1770n − 1863n + 2141n ifadesinin n nin alabileceği herhangi bir
tamsayı değeri için 1946 ile bölünebileceğini gösteriniz.

2.12.2 Çözümler
1. Eğer polinom bölmesi özeliğini kullanırsak elimizde

n3 + 100 = (n + 10)(n2 − 10n + 100) − 900


2.12. BÖLÜNEBİLME VE ASAL SAYILAR 117

eşitliği oluşur. Ancak bölme işleminin kalansız olabilmesi için bölme iş-
leminde kalanın sıfır olması gerekir. Demek ki,

n + 10|900

olmalıdır. n + 10 ≤ 900 ise n değerinin en büyük değeri 890 olur.

2. n(n4 − 5n2 + 4) = (n2 − 1) · (n2 − 1) · n = (n − 2) · (n + 2) · (n − 1) · (n + 1) · n


ise ifade ardışık 5 sayının çarpımıdır ve 5! ile kalansız bölünür.

3. p > 3 ise p = 6k ± 1 formundadır. Buna göre,

(6k+1)2 −12 = (6k+1−1)·(6k+1+1) = 6k·(6k+2) = 6k·2(3k+1) = 12k·(3k+1)

olur. k sayısı tek ise 3k + 1 = 2A, A ∈ Z ⇒ 12k · (2A) = 24 · A · K olur.

4.

2a, 2a + 1 ⇒ (2a)2 = 4a2


(2a + 1)2 = 4a2 + 4a + 1 = 4(a2 + a) + 1 olur.

5. Bir önceki örnekten dolayı tamkare olmadığı açıktır. Çünkü tamkare bir
ifade 4k veya 4k + 1 formundadır.

6. (x)5 = (6p + 1) ise x sayısı da kesinlikle asaldır. x = 3 ise x5 = 243


olur. 240 = 6 · 40 + 3 ise x = 3 olamaz. 6p = x5 − 1 = (x − 1) ·
4 3 2
( x
| + x +{zx + x + 1} ). Demek ki (x − 1) de 6 nın çarpanı ise
Bu kısım x=2 için 6 dan büyük olur.
x = 2, 3, 4 veya 7 olur. Bunlardan sadece 7 sağlar. 75 = 6 · 2801 + 1 olur.

7.

21 · p + 1 = x2
21 · p = x2 − 1
21 · p = (x − 1) · (x + 1) ⇒ (x + 1) − (x − 1) = 2 olmalıdır.

Yani, p = 23 veya p = 19 olabilir.3 · 7 · p = (x − 1) · (x + 1) ⇒ p = 3


olabilir.
118 BÖLÜM 2. KONULAR

8. n = k + (k + 1) + (k + 2) + . . . + (k + m) olarak seçelim. Buna göre

(m + 1) 1
n=
m + 2k 2
olur. Burada, (2k + m) − (m + 1) = 2k − 1 ifadesi tek sayı olduğuna göre

(m + 1) · (m + 2k) = 210

sayısını veren

(105, 2); (70, 3); (42, 5); (35, 6); (30, 7); (21, 10); (15, 14)

biçiminde 7 sayı vardır.

9. p2 + 17 sayısında p asal ise p = 2 için 4 + 17 = 21 sayısının 4 farklı böleni


ve p = 3 için p2 + 17 = 26 = 2 · 13 ise 4 tane bölende burada vardır. p ≥ 3
ise p2 + 17 = x ve p2 − 1 = (x − 18) olduğuna göre, (p − 1)(p + 1) = x − 18
olur. Burada (p − 1)(p + 1), 3 ile tam bölünür. Buna göre;

(p − 1)(p + 1) ≡ 0 ≡ x − 18 ≡ x(mod3) ise 3|x durumu vardır.


(p − 1)(p + 1) ≡ 0 ≡ x − 18 ≡ x(mod2) ise 2|x durumu vardır.

x = a · b veya x = c3 olabilir. x = 2 · 3 = 6 olmalıdır. x = 6 ise p2 + 17 = 6


durumu olmaz. Demek ki sadece 2 ve 3 için 4 asal böleni vardır.

10. Eğer doğrulayan n değerlerini bulmak istiyorsak, ifademizi

n2 − n = (n − 3) · (n + 2) + 6

olarak yazabiliriz. Tam bölünebilme olabilmesi için (n − 3)|6 olmalıdır.


Buna göre n değerleri, {4, 5, 6, 9} olmalıdır.

11. Varsayalım

A = 25! = 2a1 · 3a2 · 5a3 · 7a4 · 11a5 · 13a6 · 17a7 · 19a8 · 23a9

olsun. Burada, a1 = 22, a2 = 10, a3 = 6, a4 = 3, a5 = 2, a6 = 1, a7 =


1, a8 = 1, a9 = 1 olacağından bu çarpımdan çıkarılması gereken sayılar
{23, 19, 17, 13, 7} olacaktır. Bu sayıların atılması yeterlidir.
2.12. BÖLÜNEBİLME VE ASAL SAYILAR 119


12. 13p + 289 = x, x ∈ Z+ olduğuna göre, 13p + 289 = x2 ve 13p =
(x − 17) · (x + 17) bulunur. p asal olduğundan böleni yoktur. x − 17 = 13
ve x = 30 bulunur. Demek ki, 13 · p = 13 · 4 ise p = 47 bulunur.

13. a. N = (777)b = 7b0 + 7b1 + 7b2 = x4 ⇒ 7(b2 + b + 1) = x4 eşitliğinden


b2 + b + 1 = 73 · A olmalıdır. A = 1 olursa, b · (b + 1) = 73 − 1 ise
buradan b = 18 bulunur.
2
b. Varsayalım n = 10x+y olsun. Buradan n2 = 100
| {z· x} + 20 · xy + y 2 .
| {z } |{z}
çift çift tek olmalı
Buna göre y2 = 36 yada 16 olabilir. Buna göre istenen cevap 6 ol-
malıdır.

14. Varsayalım

x2 = n + 1
y 2 = 16n + 1

olsun. Buna göre, y 2 − x2 = (y − x) · (y + x) = 15 · n ifadesinde n yerine


x2 − 1 yazarsak yeni ifademiz (4x − y) · (4x + y) = 15 elde edilir. Buradan
istenen tek değer n = 3 olur.

15. Bu sorunun çözümü okuyucuya bırakılmıştır.

16.

44
| .{z
. . 44} 88 . . 88} 9 = 44 . . . 4 · 10n + 88 . . . 8 · 10 + 9
| .{z
n tane 4 n-1 tane 8
4 n 8
= (10 − 1) + (10n−1 − 1) + 9
9 9
4 4 1
= · 102n + · 10n +
9 9 9
1 n 2 2 · 10n + 1 2
= · (2 · 10 + 1) = ( ) olur.
9 3

17. Bu sorunun çözümü okuyucuya bırakılmıştır.

18. Bu sorunun çözümü okuyucuya bırakılmıştır.


120 BÖLÜM 2. KONULAR

19. Bu sorunun çözümü okuyucuya bırakılmıştır.

20. 2x2 − x − 36 = p2 ise (x + 4) · (2x − 9) = p2 olacaktır. Buna göre;

i.

x + 4 = p2
2x − 9 = 1 ⇒ x = 5, p = 3

ii.

x+4 = p
2x − 9 = p ⇒ x = 13, p = 17

iii.

x+4 = 1
2x − 9 = p2 ⇒ x = −3, p2 = −15 olamaz.

iv.

x + 4 = −p2
2x − 9 = −1 ⇒ x = 4, p2 = −8 olamaz.

v.

x + 4 = −p
2x − 9 = −p ⇒ x = 13, p = −17 olamaz.

vi.

x + 4 = −1
2x − 9 = −p2 ⇒ x = −5, p = 19 olamaz.

21. Varsayalım ardışık sayılarımız,

m, (m + 1), (m + 2), . . . , (m + k)
2.12. BÖLÜNEBİLME VE ASAL SAYILAR 121

ise Bu sayıların toplamı,


1
(k + 1)(2m + k)
2
olacaktır. Eğer bu toplam 2 nin bir kuvvetine eşit ise, u ∈ Z için,
1
2u = (k + 1)(2m + k) ⇒ 2u+1 = (k + 1)(2m + k)
2
eşitliği ele edilir. Buradan da,
k tek ise, m çift olacaktır. Buna göre, (T + 1)(C + T ) = C · T →
Tek sayı 6= 2u+1
k çift ise, m tek olacaktır. Buna göre, (C + 1)(C + C) = T · C →
Tek sayı 6= 2u+1 olacaktır.

22. Önceki çözülen örneklerin sonucundan faydalanırsak, 2 nini kuvvetleri-


nin yazılamayacağını görürüz. Eğer ilk yüz sayıdan ikinin kuvvetlerini
çıkarırsak, istenen sonuca ulaşabiliriz.

23.

n + (n + 1) + (n + 2) + · · · + (n + 97) = 192 · K
97 · 98
98 · n + = 192 · K
2
98 · n + 97 · 49 = 192 · K
49 · (2n + 97) = 192 · K
(2n + 97) = 192 = 361 ise n = 32 olur.

24. Varsayalım, a = p · q, b = r · s, c = p · r ve d = q · s olsun. Buna göre


soruda verilen toplam

p2 q 2 + r2 s2 + p2 r2 + q 2 s2 = (p2 + s2 )(q 2 + r2 )

elde edilirki buda çözüm için yeterlidir.

25. (x − y)|(xn − y n ) olduğunu zaten biliyoruz. Buna göre, 2903n − 803n


ifadesi 2903 − 803 = 2100 = 7 · 300 ve 261n − 464n ifadeside 261 − 464 =
−203 = −29 · 7 ile bölünebilir. Demek ki, verilen ifade 7 ile bölünebilir.
122 BÖLÜM 2. KONULAR

Benzer biçimde, 2903n − 464n ifadeside 2903 − 464 = 2439 = 9 · 271 ve


−803n + 261n ifadeside −803 + 261 = −542 = −2 · 271 ise deekki, ifade
271 ilede tam bölünür. Buna göre, ifade 7·271 = 1897 ilede tam bölünür.

26. Bu sorunun çözümü okuyucuya bırakılmıştır.

2.13 Trı̇gonometrı̇de Sonsuz Toplam ve Farklar


Aslında başlık olarak hernekadar cebirdeki sonsuz toplam ve farklarıanım-
satsa da aynısı değildir. Ancak genel mantıkta benzeşmeler ve hatta örtüşme-
ler olduğu soruların çözümleri yapılırken göze çarpar.İlerleyen örneklerde ve
çalışma sorularında sizde farkına varacaksınız ki, genel amacımız verilen bir
sonsuz toplamı fark formunda, yani

n
X
[F (k) − F (k − 1)]
k=2

formunda yazarak F (n) − F (1) sonucuna ulaşmak olacaktır. Önce nispeten


daha kolay bir örnekle başlayalım.
Size tavsiyemiz bu bölümdeki sorulara çalışmaya başlamadan evvel trigo-
nometrik özdeşliklere bir göz atmanız yönünde olacaktır. Böylelikle konuya
daha hakim olabilirsiniz.

Örnek.
n
X
cos kx
k=1

toplamını hesaplayınız.

P
n
Çözüm. Eğer cos kx = A eşitliğinde eşitliğin iki tarafınıda 2 sin x2
k=1
ile çarparsak, çarpımdan toplama geçebilmek için özdeşlikleri kullanabiliriz.
2.13. TRİGONOMETRİDE SONSUZ TOPLAM VE FARKLAR 123

Öyleki
Xn
x x
2 sin · A = sin cos kx
2 2
k=1
Xn µ µ ¶ µ ¶ ¶
1 1
= sin k + x − sin k − x
2 2
k=1
µ ¶
1 x
= sin n + x − sin olacaktır.
2 2
Buna göre, bu eşitlikten
¡ ¢
sin n + 12 x 1
A= ¡ ¢ −
2 sin x2 2
bulunur.
İkinci örneğimiz de tanjant toplamını kullanarak rahatlıkla çözüme ulaşa-
bileceğimiz bir örnek.

Örnek. µ ¶
n
X 1
tan−1
k2 + k + 1
k=0
toplamını hesaplayınız.(tan−1 x = arctan x)

Örnek. Tanjant fark formülü


tan a − tan b
tan (a − b) =
1 + tan a tan b
olduğuna göre
u−v
tan−1 u − tan−1 v = tan−1
1 + uv
olacaktır. Daha da basite indirgemek için ak = tan−1 k olarak alalım. Buna
göre,
tan ak+1 − tan ak
tan (ak+1 − ak ) =
1 + tan ak+1 tan ak
k+1−k 1
= = 2 olacaktır.
1 + k (k + 1) k +k+1
124 BÖLÜM 2. KONULAR

Öyleyse toplamımız,
n
X n
X
tan−1 (tan (ak+1 − ak )) = (ak+1 − ak ) = an+1 − a0
k=0 k=0
= tan−1 (n + 1) olacaktır.

Benzer çözüm biçimleri verilen çalışma soruları üzerinde uygulanabilir. Buna


göre sonsuz bir toplamı çözerken, ifadeyi farkların sonsuz toplamı biçiminde
yazarak sadeleştirmeler yapmak yerinde olacaktır.

2.13.1 Çalışma Soruları


1.
sin x sin 2x sin nx cos (n + 1) x
+ 2
+ ··· + n
= cot x −
cos x cos x cos x sin x · cosn x
eşitliğini kanıtlayınız.

2.
1 1 1 cos 1
+ + ··· + =
cos 0 · cos 1 cos 1 · cos 2 cos 88 · cos 89 sin2 1
eşitliğini kanıtlayınız.

3. n ∈ Z+ ve a ∈ R olmak üzere a/π bir rasyonel sayı olarak veriliyor. Buna


göre,

1 1 1
+ + ··· +
cos a − cos 3a cos a − cos 5a cos a − cos (2n + 1) a

toplamının eşitini bulunuz.

4.
n
X µ ¶
−1 1 n
tan = tan−1
2k 2 n+1
k=1

eşitliğini kanıtlayınız.
2.13. TRİGONOMETRİDE SONSUZ TOPLAM VE FARKLAR 125

5. a 6= kπ, k ∈ Z olmak üzere,



X 1 a
tan n
2n 2
n=1

toplamının eşitini bulunuz.

6.

X a 1
3n−1 sin3 n
= (a − sin a)
3 4
n=1
eşitliğini kanıtlayınız.

7. n = 2, 4, 6, · · · , 180 olmak üzere verilen n · sin n◦ sayılarının aritmetik


ortalamasının cot 1◦ olduğunu kanıtlayınız.

8. m, n ∈ Z+ ve x 6= m olduğuna göre,

1 1 1
+ + ··· + = cot x − cot 2n x
sin 2x sin 4x sin 2n x
eşitliğini kanıtlayınız.

9.
tan 1 tan 2 tan 2n
+ + ··· +
cos 2 cos 4 cos 2n+1
toplamını hesaplayınız.

10. Sıfırdan farklı her x reel sayısı için,



Y x sin x
cos n
=
2 x
n=1

eşitliğini kanıtlayınız.

11. n > 1 ve n ∈ Z için


2π 4π 2n π 1
cos n
· cos n
· · · · · cos n
= n
2 −1 2 −1 2 −1 2
eşitliğini kanıtlayınız.
126 BÖLÜM 2. KONULAR

12.
n µ
Y k

2 2 π
1 − tan n
2 +1
k=1

çarpımının eşitini bulunuz.

13. (1 − cot 1◦ ) (1 − cot 2◦ ) (1 − cot 3◦ ) · · · (1 − cot 44◦ ) çarpımını hesaplayı-


nız.

14.
µ ¶µ ¶µ ¶µ ¶
1 π 1 3π 1 9π 1 27π 1
+ cos + cos + cos + cos =
2 20 2 20 2 20 2 20 16

eşitliğini kanıtlayınız.
¡π ¢
15. n ∈ Z+ , x 6= 2k+1 3 + lπ ve k = 1, 2, · · · , n, l ise

n ³
Y x´
1 − 2 cos k
2
k=1

çarpımının sonucunu bulunuz.

16.
n µ
X ¶
2π · 3k
1 + 2 cos n =1
3 +1
k=1

eşitliğini kanıtlayınız.

2.13.2 Çözümler
1. sin kx · sin x = cos kx · cos x − cos (k + 1) x ise bu eşitsizliğin iki tarafınıda
sin x · cosk x ile bölelim. Buradan

sin kx cos kx cos (k + 1) x


k
= k−1

cos x sin x · cos x sin x · cosk x
2.13. TRİGONOMETRİDE SONSUZ TOPLAM VE FARKLAR 127

olacaktır. Buna göre,


sin x sin 2x sin 3x sin nx
= + 2
+ 3
+ ··· +
cos x cos x cos x cosn x
cos x cos 2x cos 2x cos 3x cos nx cos (n + 1) x
= − + − + ··· + −
sin x sin x · cos x sin x · cos x sin x · cos2 x sin x · cosn−1 x sin x · cosn x
cos (n + 1) x
= cot x −
sin x · cosn x
olarak bulunacaktır.
2. Eşitliğin iki tarafınıda sin 1 ile çarparsak,
sin 1 sin 1 sin 1 cos 1
+ + ··· + =
cos 0 cos 1 cos 1 cos 2 cos 88 cos 89 sin 1
olacaktır. Buradan
sin (1 − 0) sin (2 − 1) sin (89 − 88)
+ + ··· + = cot 1
cos 1 cos 0 cos 2 cos 1 cos 89 cos 88
eşitliği elde edilir. Burada
sin (a − b)
= tan a − tan b
cos a cos b
ise son yazdığımız eşitliğin sol tarafı
89
X
[tan k − tan (k − 1)] = tan 89 − tan 0 = cot 1
k=1
olacaktır.
3. Soruda verilen eşitliği düzenlersek,
n
X n
1 1X 1
=
cos a − cos (2k + 1) a 2 sin ka · sin (k + 1) a
k=1 k=1
eşitliğini elde ederiz. Bir önceki problemin çözümünde olduğu gibi, eğer
sin a ile çarparsak, toplam
n n
1 X sin ((k + 1) a − ka) 1X
= (cot ka − cot (k + 1) a)
2 sin ka · sin (k + 1) a 2
k=1 k=1
1
= (cot a − cot (n + 1) a)
2
128 BÖLÜM 2. KONULAR

olacağından sorudaki toplamın eşiti,


(cot a − cot (n + 1) a)
2 sin a
olarak bulunur.
4. Eğer soruda verilen eşitliğin sol tarafını düzenlesek
Xn Xn µ ¶
−1 1 −1 (2k + 1) − (2k − 1)
tan = tan
2k 2 1 − (2k − 1) (2k + 1)
k=1 k=1

eşitliğini elde ederiz. Eğer arktanjant için fark formülünü kullanırsak


n
X ¡ −1 ¢
tan (2k + 1) − tan−1 (2k − 1)
k=1

olacaktır. Bu toplam da
µ ¶ µ ¶
−1 −1 −1 2n + 1 − 1 −1 n
tan (2n + 1) − tan (1) = tan = tan
1 + 2n + 1 n+1
olacaktır.
5.
1 1 − tan2 x 1 2
tan x = − = − = cot x − 2 cot 2x
tan x tan x tan x tan2 x
olduğuna göre,
1 a 1 a 1 a
n
tan n = n cot n − n−1 cot n−1
2 2 2 2 2 2
olacaktır. Bu toplam da
x 1
lim x cot ax = lim cos ax =
x→0 x→0 sin ax a
olacağından istenilen değer
1
− cot a
a
olarak bulunur.
2.13. TRİGONOMETRİDE SONSUZ TOPLAM VE FARKLAR 129

6. sin 3x = 3 sin x − 4 sin3 x olduğuna göre,


a 1³ n a a ´
3n−1 sin3 n = 3 sin n − 3n−1 sin n−1
3 4 3 3
olacaktır. Teleskopik toplamdan
1 sin 3an 1 1
lim 1 − sin a = (a − sin a)
4 n→∞
3n
4 4

olacaktır.

7. Öncelikle,
90
X
2k sin (2k) sin 1
k=1
ifadesini hesaplayalım. Çarpımdan, farkların toplamına gidersek
90
X 90
X
k cos (2k − 1) − k cos (2k + 1) = (k − (k − 1)) cos (2k − 1) − 90 cos 181
k=1 k=1
X90
= cos (2k − 1) + 90 cos 1
k=1

olacaktır. cos(180 − x) = − cos x olduğuna göre, son toplamdaki eleman-


lar ikili ikili sadleşecektir. Sonuç olarak geriye sadece 90 cos 1 kalacaktır.
Burayıda sin 1 ile bölersek
90 cos 1
= cot 1
sin 1
olacaktır.

8.
¡ ¢
1 2 cos2 x − 2 cos2 x − 1
=
sin 2x sin 2x
2
2 cos x 2 cos2 x − 1
= −
2 sin x cos x sin 2x
cos x cos 2x
= − = cot x − cot 2x
sin x sin 2x
130 BÖLÜM 2. KONULAR

olduğuna göre, soruda verilen eşitliğin sol tarafındaki teleskopik toplam,


yine bir teleskopik farklar toplamına gideceğinden, eşiti

cot x − cot 2n x

olacaktır.

9.
¡ ¢ ¡ ¢
tan a tan a 1 + tan2 a 2 tan a − tan a 1 − tan2 a
= =
cos 2a 1 − tan2 a 1 − tan2 a
2 tan a
= − tan a = tan 2a − tan a
1 − tan2 a
olduğuna göre soruda verilen toplam,

tan 2 − tan 1 + tan 4 − tan 2 + · · · + tan 2n+1 − tan 2n = tan 2n+1 − tan 1

olacaktır.

10. Sinüs fonksiyonunda yarımaçı formülünden,


sin 2u
cos u =
2 sin u
olacaktır. Bundan dolayı,

Y k
Y k
Y x
x x 1 sin 2n−1
cos n = lim cos n = lim
2 k→∞ 2 k→∞ 2 sin 2xn
n=1 n=1 n=1

eşitliğini elde ederiz. Bu eşitliğe göre,


x
1 sin x sin x k sin x
lim k x = lim 2 x =
k→∞ 2 sin k x k→∞ sin 2k x
2

olacaktır.

11. Eğer eşitliğin sol kısmını



2n sin
2n −1
2.13. TRİGONOMETRİDE SONSUZ TOPLAM VE FARKLAR 131

ile çarpıp sinüs formülünü uygularsak


2π 2π 4π 2n π
= 2n sin · cos · cos · · · · · cos
2n − 1 2n − 1 2n − 1 2n − 1
4π 4π n
2 π
= 2n−1 sin n · cos n · · · · · cos n
2 −1 2 −1 2 −1
µ ¶ µ ¶
2n+1 π 2π 2π
= sin n = sin 2π + n = sin
2 −1 2 −1 2n − 1

olacaktır. Eğer sonucu µ ¶


n 2π
2 sin n
2 −1
ile bölersek, istenen eşitliği elde etmiş oluruz.

12.
2 tan x
1 − tan2 x =
tan 2x
ise k
2k π
2
tan 22n +1
π
1 − tan n =2 k+1
2 +1 tan 22n +1π
olacaktır. Buradanda
tan 2n2π+1 tan n2π
2n n+1 = 2n ³ 2 +1 ´ = −2n
tan 22n +1π tan 2π − 2n2π+1

olacaktır.

13.
√ ³ π π ´ √ sin (x − π/4)
1 − cot x = 2 cos − cos cot x = 2
4 4 sin x
olacağından,

sin (−44) sin (−43) sin (−1)


(1 − cot 1) (1 − cot 2) · · · (1 − cot 44) = 222 · · ··· ·
sin 1 sin 2 sin 44
= 222

olacaktır.
132 BÖLÜM 2. KONULAR

14.
cos 3x
= 4 cos2 x − 3 = 2 (1 + cos 2x) − 3 = 2 cos 2x − 1
cos x
olacaktır. Dolayısıyla,
à ¡ ¢!
1 1 cos 32 x + 3π 1 sin 32 x
+ cos x = − ¡ x π2¢ =
2 2 cos 2 + 2 2 sin x2

olacaktır. Buna göre soruda verilen çarpım

1 sin 3π 9π 27π
40 sin 40 sin 40 sin 81π
40
= · π · 3π · 9π · 27π
16 sin 40 sin 40 sin 40 sin 40
1 sin 81π
40 1
= · π =
16 sin 40 16

olacaktır.

15.
1 − 4 cos2 a 1 − 2 (1 + cos 2a)
1 − 2 cos a = =
1 + 2 cos a 1 + cos a
1 + 2 cos 2a
= −
1 + cos a
ise sorumuzda ki sonsuz çarpım
1 + cos x
(−1)n ·
1 + 2 cos 2xn

olacaktır.

16.
¡ ¢
1 + 2 cos ak = 1 + 2 1 − sin2 ak = 3 − 4 sin2 ak
sin 3ak
=
sin ak
ve buradanda,
3k · 2π
ak =
3n + 1
2.14. KUŞ, GÜVERCİN, YUVA... 133

olacaktır. Buradan sonsuz çarpımımız,


n+1
sin 3an sin 33n +1π
=
sin a1 sin 3n3π+1
olacaktır. Son yazılan kesrin payı
µ ¶
3π 3π
sin 3π − n = sin n
3 +1 3 +1
ise eşitliğin sonucunun 1 olacağı açıktır.

EAT SLEEP DO MATH

2.14 Kuş, Güvercı̇n, Yuva...


Güvercin Yuvası İlkesi her nekadar isminde güvercinlere yer versede aslında
bu güzel canlılarla uzaktan yakından bir alakası yoktur. Ancak temel prensibin
kavranması için verilen örneklerde hep bu canlıyla alakalı örneklemeler yapıl-
mıştır. Peki nedir bu ilke? Açıklayalım. Eğer (n + 1) tane nesneyi (Güvercini),
n tane kutu (Güvercin yuvası) arasından seçersek, bu nesnelerden en az iki
tanesi aynı kutudan gelmiş olacaktır. Veya İstanbul ili sınırları içerisinde ya-
şayan ve saç tellerinin sayısı aynı olan en az iki kişi bulabiriz. Yazdığımız bu
iki önermede güvercin yuvası ilkesinin kavranması için yeterlidir. Yada benzer
biçimde, herhangi bir ormandan seçilecek en az iki ağacın yaprak sayılarının
aynı olmasını sağlayan yeterli ve gerekli şartlar nelerdir? Sorusunun cevabıda
yine aynı ilkeyle bulunacaktır. Şimdi bu ilkenin sorular üzerindeki uygulama-
sını görmek için örnekleri inceleyelim.

Örnek. 1, 2, 3, · · · , 99, 100 sayıları arasında 51 tane sayı seçiliyor. Buna


göre bu 51 sayıdan iki tanesinin aralarında asal olduğunu kanıtlayınız.
134 BÖLÜM 2. KONULAR

Çözüm. Varsayalım bu 100 sayı arasından

(1, 2), (3, 4), · · · , (99, 100)

şeklinde ikililer oluşturalım. 51 tane sayı seçileceğine göre, güvercin yuvası


ilkesi uyarınca bu sayılar arasında (k, k + 1) ikilisi bulunacaktır. Buna göre
k + 1 ve k sayılarını bölen p asalı varsa, bu asal (k + 1) − k = 1 sayısını
da bölecektir ki, bu durum çelişki olur. Demek ki, (k, k + 1) sayılarının ortak
böleni yoktur.

Örnek. 1, 2, 3, · · · , 99, 100 sayıları arasından 51 tane sayı seçiliyor. Buna


göre, bu 51 sayı arasında ki sayılardan birinin, bir diğer sayıyı böleceğini gös-
teriniz.

Çözüm. 50 tek sayıyı, 1, 3, 5, · · · , 99 sayılarını alalım. Her biri için


bir kutu düşünelim. Bu kutular hem sayının kendisini hem de sayı ile 2’nin
kuvvetinin çarpımını içersin. Buna göre ilk kutu 1, 2, 4, 8, 16, · · · , ikinci kutu
3, 6, 12, 24, 48, · · · olarak gidecektir. Eğer 51 sayı seçersek güvercin yuvası il-
kesine göre bu iki sayı mecburen aynı kutudan gelecektir. Yani sayılardan biri
2n k iken diğeri 2m k olacak ve aynı k çarpanına sahip olacaklardır. Dolayısıyla
sayılardan biri diğerini bölecektir.
Çözülen iki örnektede farkedeceğiniz üzere, her nekadar soru içerisinde
bahsi geçmesede kutu formuyla çözüme gidilmiştir. Demek ki çözümdeki anah-
tar basamak kutu prensibini kavramak olacaktır. Şimdi aşağıda verilen örnek-
leri inceleyerek bu formu daha iyi kavramaya çalışınız.

Örnek. Seçilen herhangi dokuz faklı reel sayı arasında


(a − b) √
0< < 2−1
1 + ab
eşitsizliğini sağlayan bir (a, b) ikilisinin bulunabileceğini kanıtlayınız.

Çözüm. Verilen eşitsizliğin ortasındaki ifadeyi incelersek bu ifadenin


tan(x − y) formülüne benzediğini görebiliriz. Yani ifade aslında
tan x − tan y
tan (x − y) =
1 + tan x tan y
2.14. KUŞ, GÜVERCİN, YUVA... 135

biçimindedir. Şimdi artık sorumuza biraz daha trigonometrik yaklaşalım. Buna


göre,(−π/2, π/2] aralığını 8 eşit uzunlukta parçaya ayıralım. Bu parçaların
µ ¸ µ ¸ µ ¸ µ ¸
π 3π 3π π π 3π 3π π
− ,− , − ,− , ··· , , , ,
2 8 8 4 4 8 8 2
olacağı açıktır. Burada xi = arctan ai , i = 1, 2, · · · , 9 olacak biçimde sayılar
seçelim. Güvercin yuvası prensibine göre, xi ’lerden iki tanesi, biz bunlara xj ve
xk diyelim, xj > xk olacak biçimde ayırdığımız 8 aralıktan birinde olacaktır.
Buna göre
π
0 < xj − xk <
8
ve dolayısıyla
aj − ak ³π ´ √
0 < tan (xj − xk ) = < tan = 2−1
1 + aj ak 8
olacaktır.

Örnek. Bir kenarı 1 birim olan bir karenin içine yerleştirilen köşeleri kare
üzerinde bulunan bir üçgenin karenin merkezini içermemesi isteniyor. Buna
göre üçgenin bir kenarının 1 birimden küçük olması gerektiğini kanıtlayınız.

Çözüm. Karenin merkezini C noktası olarak alıp, kareyi 4 eşit parçaya


bölelim. Eğer çizilen üçgenin C noktasını içermemesi isteniyorsa, üçgenin kö-
şelerinin komşu iki parçanın içerisinde olması gerekir. Eğer üçgen oluşturmak
istiyorsak alacağımız 3 noktanında komşu kareler içerisinde olması gerekecek-
tir. Ancak 3 noktadan 2 tanesini aynı kareden alacağımıza göre bu noktalar
arası uzaklık kesinlikle 1 birimden küçük olacaktır.

2.14.1 Çalışma Soruları


1. 1, 2, 3, · · · , 99, 100 sayıları arasından 11 tane sayı seçiliyor. Buna göre,
bu 11 sayının altkümelerinden en az iki tanesinin elemanları toplamının
aynı olacağını kanıtlayınız.
2. Varsayalım koordinatları birer tamsayı olan 9 nokta üç boyutlu koordi-
nat düzleminde seçilsin. Buna göre, bu noktaların birleştirilmesiyle oluş-
turulan doğru parçalarından seçilen bir tanesinin üçüncü bir tamsayı
koordinatlı noktayı içerdiğini kanıtlayınız.
136 BÖLÜM 2. KONULAR

3. Altı kişilik bir grup içinde ya herhangi üç kişinin birbirini tanıdığını yada
en az üç kişinin birbirini tanımadığını kanıtlayınız.

4. Seçilecek herhangi bir 16 basamaklı doğal sayının ardışık basamaklarının


çarpımının birer tamkare olmasını sağlayan en bir zincirin bulunduğunu
kanıtlayınız. Örneğin

12 |{z} 91 8 gibi.
343 21524716 |{z}
36 9

2.14.2 Çözümler
1. Onbir elemanlı bir kümenin 211 − 2 = 2046 tane eleman sayısı 11’den az
olan boş kümeden farklı altkümesi vardır. Bu alt kümelerden herhangi
birinin elemanları toplamı en fazla

91 + 92 + 93 + · · · + 99 + 100 = 955

olacaktır. Buna göre, güvercin yuvası ilkesi uyarınca, elemanları toplamı


birbirine eşit olan iki, tane alt küme bulunacaktır. Eğer bu kümelerin or-
tak elemanları varsa, bu elemanı iki kümedende çıkararak kesişimleri boş
üme olan ve elemanları toplamları aynı olan iki alt küme elde edebiliriz.

2. Dokuz nokta için her bir (x, y, z) koordinatları ya tek yada çift olacaktır.
Bu durumda 23 = 8 farklı teklik çiftlik durumu ortaya çıkar. Buna göre
bu dokuz noktadan iki tanesi mecburen koordinat pariteleri eşit nokta-
lar olacaklardır. Ve dolayısıyla orta noktalarının koordinatlarıda birer
tamsayı olacaktır.

3. Önce elimizdeki 6 kişiyi düzgün bir altıgenin köşelerine yerleştirdiğimiz


düşünelim. Eğer iki kişi biribirini tanıyorsa bu iki köşeyi birleştiren köşe-
gen kırmızıya boyansın, tanımıyorsa da maviye boyansın. Aslında soru-
nun çözümü ile bizim altıgenin köşeleriyle tüm kenarları aynı renk olan
bir üçgen elde edip edemeyeceğimiz sorusunun cevabı özdeşitir. Herhangi
bir köşeyi alalım, güvercin yuvası ilkesi uyarınca, bu köşeden çıkan üç
köşegen aynı renk olsun. Bundan sonraki herbir köşegende rengi, üçgen
oluşturmayacak biçimde seçmeye çalışsak bile mutlaka ortaya her defa-
sında kenarları aynı renk olan bir üçgen ortaya çıkacaktır.
2.15. ÜSTEL DİYOFANT DENKLEMLERİ 137

4. Varsayalım d1 , d2 , d3 , · · · , d11 sayımızın basamakları olsun. Eğer bu sayı-


nın basamaklarından her hangi biri 0, 1, 4 veya 9 ise problemin çözümü
açıktır. Varsayalım sayımızın basamakları 2, 3, 5, 6, 7 ve 8 sayılarından
oluşsun. Eğer x0 = 1 ve xi değeride d1 , d2 , · · · , di i = 1, · · · , 16 sayıları-
nın çarpımı olsun. Her bir xi = 2pi · 3qi · 5ri · 7si olacaktır. Burada üst
değerler zaten ya tektik yada çift sayıladır. Dolayısıyla 24 = 16 farklı du-
rum elde edilebilir. Güvercin yuvası ilkesi uyarınca pi , qi , ri , si değerleri
onyedi xi ’lerin ikisi için, varsayalım bu ikililerde xj ve xk olsun, teklik
çiftlik bakımından aynı pariteye sahip olacaklardır. Buna göre
xk
dj+1 · dj+2 · · · · · dk =
xj
tamkare olacaktır.

2.15 Üstel Dı̇yofant Denklemlerı̇


Üstel diyofant denklemlerinin diğer diyofant denklemlerinden farkları de-
ğişkenlerin genelde üstel durumda bulunmasıdır. Genel bir çözüm yöntemleri
olmamasına rağmen bazı cebirsel oyunlarla çözüm yolları geliştirilebilir. Aşa-
ğıda verilen örnekleri incelediğiniz zaman tam olarak ne demek istediğimizi
daha iyi anlayacaksınız. İlk olarak kolay bir örnekle başlayalım.

Örnek. (x, y, z) ∈ Z+ olmak üzere verilen 3x + 4y = 5z denklemini


sağlayan tüm üçlüleri bulunuz.

Çözüm I. Eğer soruda verilen eşitliğe (mod4) altında bakarsak 3x ≡


1 (mod4) dekliğini elde ederiz. Buna göre x = 2x1 olmalıdır. Benzer biçimde
5z ≡ 1 (mod3) olduğundan z = 2z1 alabiliriz. Bunları denklemde yerine ko-
yarsak
4y = (5z1 + 3x1 ) (5z1 − 3x1 )
olacaktır. Buradan 5z1 + 3x1 = 2s ve 5z1 − 3x1 = 2t olmalıdır. s¡ + t = 2y¢ ve
s > t olacağı
¡ açıktır.
¢ Eğer bu iki denklemi çözersek 5z1 = 2t−1 2s−t + 1 ve
3x1 = 2t−1 2s−t − 1 bulunur. Bu iki eşitlikte de sol kısımlar birer tek sayıdır.
Öyleyse t = 1 olmalıdır. Eğer s − t = u dersek 3x1 = 2u − 1 ve 5z1 = 2u + 1
olacaktır. Eğer birinci denkleme (mod3) altında bakarsak u = 2u1 bulunur.
138 BÖLÜM 2. KONULAR

Sağ tarafı çarpanlara ayırırsak 2u1 + 1 = 3α ve 2u1 − 1 = 3β eşitlikleri elde


edilir. Buradan 3α − 3β = 2 olduğuna göre α = 1 ve β = 0 olacaktır. Buradan
u1 = 1 ve u2 = 2 olduğundan çözümde x = y = z = 2 olarak bulunur.

Çözüm II. Bu sefer farklı bir yönden probleme yaklaşacağız. Birinci


çözümde olduğu üzere x = 2x1 ve z = 2z1 eşitliklerini kullanırsak,

(3x1 )2 + (2y )2 = (5z1 )2

eşitliğini elde ederiz. Eğer x = 3x1 , y = 2y , z = 5z1 olarak alırsak Pisagor


denkleminden
x2 + y 2 = z 2
olur. Burada x, y, z değerleri aralarında asal ve y çift bir sayıdır. Pisagor üç-
lülerini kullanırsak değerlerimiz

x = u2 − v 2 , y = 2uv, z = u2 + v 2

olur. Bu eşitliklerin ikincisinden u ve v ’nin 2 ’nin kuvvetleri oldukları açıktır.


Buna göre v = 1 olmalıdır. Çünkü birinci denklemden, 3’ün kuvvetleri 2 ’nin
kuvvetleri ile bölünemezler. Demek ki,3x1 = 2y − 1 ve 5z1 = 2y + 1 olacaktır.
İkinci denkleme (mod4) altında bakarsak y = 2y1 olur. Buradan

3x1 = (2y1 − 1) (2y1 + 1)

olacağından çarpanlardan her ikisi de 3 ’ün kuvveti olmalıdır. Buna göre y1 = 1


ve x1 = z1 = 1 olmalıdır. Sonuç olarak, x = y = z = 2 olarak bulunur. Sıradaki
sorumuz 2005 Romanya Takım Seçme Sınavı’ndan alınmıştır.

Örnek. (x, y) ∈ Z+ olmak üzere 3x = 2x ·y+1 eşitliğini sağlayan değerleri


bulunuz.

Çözüm. Denklemi yeniden yazarsak 3x − 1 = 2x · y olacaktır. Buna


göre, (3x − 1) ifadesinin asal çarpanlarından birisi ikinin kuvvetidir. Ve x de-
ğeri bu kuvvetten küçüktür. Şimdi bu kuvvetin ne kadar büyük olabileceğini
araştıralım. Ayrıca, eğer n değeri tekse 3n − 1 ≡ 2 (mod4) ve eğer n çift ise
2.15. ÜSTEL DİYOFANT DENKLEMLERİ 139

3n − 1 ≡ 0 (mod4) olacaktır. Buna göre x değişkenini 2m (2n + 1) formunda


yazabiliriz. Buna göre,
m (2n+1)¡ ¢2m
3x − 1 = 32 − 1 = 32n+1 −1
¡ ¢¡ Y ³¡
¢ m−1 ¢2k ´
= 32n+1 − 1 32n+1 + 1 32n+1 +1
k=1

olur. Eğer bu eşitliğe (mod8) altında bakarsak ilk iki çarpan 2 ve 4 olacaktır.
Buna göre (3x − 1) içindeki 2’nin kuvvetleri (m + 2) tane olacaktır. Burada
x ≤ m + 2 ise
2m (2n + 1) ≤ (m + 2) ve 2m ≤ (m + 2)

olacaktır. Burada m = 0, 1, 2 olabilir. Buna göre istenilen çözümler

(1, 1), (2, 2), (4, 5)

olacaktır.

2.15.1 Çalışma Soruları


1. |3x − 2y | = 1 denklemini pozitif tamsayılarda çözünüz.

2. (x, y) pozitif tamsayılar olmak üzere verilen 3x − 2y = 7 denklemini


çözünüz.

3. nx + ny + nz = nt eşitliğini sağlayan tüm (n, x, y, z, t) pozitif tamsayı


beşlilerini bulunuz.

4. a. 3x − y 3 = 1 denkleminin tüm negatif olmayan tamsayı çözümlerini


bulunuz.
b. p bir tek asal sayı olmak üzere verilen px − xp = 1 denklemini
sağlayan tüm negatif olmayan (x, y) tamsayı ikililerini bulunuz.

5. 1n +9n +10n = 5n +6n +11n eşitliğini sağlayan tüm n pozitif tamsayılarını


bulunuz.
140 BÖLÜM 2. KONULAR

2.15.2 Çözümler
1. 3x − 2y = 1 için x = 1, y = 1 ve x = 2, y = 3 birer çözümdür. Eğer y ≥ 2
ise x çift olmalıdır. x = 2z, z > 1 ve 3z = 2m + 1, m > 1 alırsak

2y = (2m + 1)2 − 1 = 4m(4m + 1)

olacaktır ki, bu durum açık çelişkidir.


3x − 2y = −1 için (mod8) altında bakarsak 3x + 1 ifadesi 2 veya 4’e denk
olacaktır. Buna göre y = 1 veya 2 olacaktır. Eğer kontrol edilirse x = 1,
y = 2 tek çözüm olarak bulunacaktır.

2. Varsayalım y > 3 olsun. Buna göre

3x ≡ 7 (mod8)

olacaktır. Ancak 3x sayısı sadece 3 veya 1 sayılarına denk olabilir. Buna


göre x = 1, 2 olabilir. x = 1 ise y ∈ / Z ve x = 2, y = 1 olduğundan tek
çözüm ikilisi (2, 1) olarak bulunur.

3. Bu sorunun çözümünü iki farklı yoldan yapmayı deneyelim. Buna göre,

Çözüm I. n = 1 durumun da denklemin sağlanamayacağı açıktır. n > 1


için bakalım. Genelliği kaybetmeden x 6 y 6 z 6 t olarak alabiliriz.
Eşitliğin iki tarafını nx ile bölersek

1 + ny−x + nz−x = nt−x

denklemini elde ederiz. Eğer denkleme (modn) altında bakarsak y = x


buluruz. Buna göre a = z − x ve b = t − x alırsak 2 + na = nb olur. Eğer
a = 0 ise n = 3, b = 1 ve çözümlerimiz de

(n, x, y, z, t) = (3, x, x, x, x + 1)

olur. Eğer a > 0 ise (modn) altında yine baktığımızda

(n, x, y, z, t) = (2, x, x, x + 1, x + 2) , x ∈ N olur.


2.15. ÜSTEL DİYOFANT DENKLEMLERİ 141

Çözüm II. Denklemimizi

nx−t + ny−t + nz−t = 1

olarak yazalım. Eşitliğin sol kısmındaki tüm terimler pozitif olduğuna


göre x, y, z < t olmalıdır. Böylece sol kısım en fazla 3/n olabilir. n = 2
ve 3 için bakarsak, n = 3 için x = y = z = t − 1 olacaktır. n = 2 için ise
sadece bir terim 1/n olabilir. Buna göre, x = y = t − 2 ve z = t − 1 ve
bu değerlerin permütasyonları bulunur.

4. a. Eğer denklemde (0, 0) ikilisi alınırsa bunun bir çözüm olduğunu


görmek zor olmayacaktır. Denklemi düzenlersek

3x = y 3 + 1

olacağından eşitliğin sol tarafı


¡ ¢
(y + 1) y 2 − y + 1

olacaktır. Burada iki çarpanların ikisi de 3 ’ün kuvveti olacaktır.


Yani çarpanlarımız

y + 1 = 3α ve y 2 − y + 1 = 3β

olacaktır. Burada ilk denklemin karesini alıp ikinci denklemden çı-


karırsak elde edeceğimiz eşitlik
³ ´
β 2α−β
3y = 3 3 −1

olacaktır. Soruda verilen denklemden y ’nin 3 ile bölünemediğini


biliyoruz. Demek ki β = 1 olacaktır. Buna göre y 2 − y + 1 = 3 ola-
cağından y = 2 olarak bulunur. Buradan da x = 2 olarak bulunur.
Demek ki ikililerimiz

(x, y) = (2, 2) , (0, 0)

olarak bulunacaktır.
142 BÖLÜM 2. KONULAR

b. Eğer (x, y) ikilisi soruda verilen denklemin bir çözümü ise


¡ ¢
px = y p + 1 = (y + 1) y p−1 − y p−2 + · · · + y 2 − y + 1

olacağından y + 1 = pn olacaktır. Eğer n = 0 ise y = x = 0 ve p


herhangi bir asal sayı olacaktır. Eğer n > 1 ise

px = (pn − 1)p + 1
µ ¶ µ ¶
np n(p−1) p n(p−2) p
= p − pp + p + ··· + p2n + ppn
2 p−2
olacaktır. p bir asal sayı olduğuna göre tüm katsayılar p ile bölüne-
bilir. Dolayısıyla her bir terim pn+1 ile bölünebilecektir. Buna göre
eşitliğin sağ tarafını bölen, p’nin en büyük kuvveti pn+1 olacaktır.
Ve dolayısıyla x = n + 1 olacaktır. Buradan
µ ¶ µ ¶
np n(p−1) p n(p−2) p
0 = p − pp + p + ··· + p2n
2 p−2
bulunur. p = 3 için
0 = 33n − 3 · 32n
¡ p ¢
ise n = 1 ve x = y = 2 bulunur. p ≥ 5 için p−2 ifadesi p2 ile
bölünemez. Dolayısıyla eşitliğin sağ kısmındaki son terim haricin-
dekiler p2n+2 ile bölünebilir. Terimlerin toplamı sıfır olduğundan,
bu durum imkansızdır. Dolayısıyla çözümler sadece x = y = 0, tüm
p asalları için ve x = y = 2, p = 3 olarak bulunur.

5. Eğer her bir terimin birler basamağına bakarsak

1n , 5n , 6n , 10n ve 11n

ifadelerinin son basamakları sırasıyla 1, 5, 6, 0, 1 olacaktır. Eşitliğin sağın-


daki terimlerin son basamakları toplamı 12 olduğuna göre, 9n sayısının
birler basamağı 1 olmalıdır. Dolayısıyla n ifadesi çift bir sayı olacaktır.
Buradan n = 2 ve n = 4 birer çözüm olacaktır. Eğer n ≥ 6 ise

11n + 6n + 5n > 11n = (10 + 1)n


= 10n + n10n−1 + · · · + 1 ≥ 10n + 9n + 1n
2.16. KALAN SINIFLARI [RESİDUES] 143

olacaktır. Çünkü

n · 10n−1 ≥ 6 · 104 · 10n−5 ≥ 95 · 10n−5 ≥ 9n

eşitsizliği vardır. Buna göre çözümlerimiz n = 2 ve n = 4 olacaktır.

2.16 Kalan Sınıfları [Resı̇dues]


Olimpiyat sınavlarında sorulan soruları eğer bir konu sınıflandırmasına tabi
tutmaya kalkışsak herhalde bu sınavların olmazsa olmaz soru gruplarından
birisi de modüler aritmetik konusu olurdu. İleriki satırlarda anlatılan örnekleri
ve problemleri daha iyi kavrayabilmeniz için bazı teoremleri bilmeniz gerekiyor.
Bunun için ilk olarak, bilmeniz gereken bu teoremlerden başlayacağız.

Teorem. [Çinli Kalanlar Teoremi] a1 , a2 , · · · , ar ikişerli olarak ara-


larında aralarında asal sayılar pozitif tamsayılar olarak veriliyor. Buna göre
(ai , aj ) = 1 ve i 6= j için

x ≡ k1 (moda1 )
x ≡ k2 (moda2 )
·········
x ≡ r1 (modar )
·········

denklik sisteminin mod(a1 · a2 · · · · · ar )’ye göre tek çözümü vardır. Bu çö-


zümde
µ ¶ µ ¶ µ ¶
a a a
x= · b1 · k1 + · b2 · k2 + · · · + · br · kr
a1 a2 ar
olacaktır. Bu eşitlikte
a = a1 · a2 · · · · · ar
ve µ ¶
a
· bi ≡ (modai )
ai
144 BÖLÜM 2. KONULAR

olacaktır.
Çinli kalanlar teoremini herhangi bir problem içerisinde nerede kullanaca-
ğınızı anlamanız oldukça kolaydır. Genelde sorulan tamsayının bir kaç tamsayı
değeri ile bölümünden kalanlar verilir. Aşağıdaki örneğimiz 2009 Tübitak Ma-
tematik Olimpiyatlarında sorulmuştu.

Örnek. 1 ≤ n ≤ 455 ve n3 ≡ 1 (mod455) koşullarını sağlayan kaç n


tamsayı değeri vardır?

Çözüm. 455 = 13 · 7 · 5 olduğuna göre,

n3 ≡ 1 (mod5)
n3 ≡ 1 (mod7)
n3 ≡ 1 (mod13)

olduğuna göre,

n ≡ 1 (mod5)
n ≡ 1, 2, 4 (mod7)
n ≡ 1, 3, 9 (mod13)

olduğuna göre, 1 · 3 · 3 = 9 tane n tamsayı bulunur.

Teorem. [Küçük Fermat Teoremi] p asal sayı ve p - a ise ap−1 ≡


1 (modp) denkliği vardır.
Şimdi bu teoremin ışığında aşağıda verilen örneği ve çözümünü inceleyelim.

Örnek. a1 = 4, an = 4an−1 , n > 1 olduğuna göre, a100 tamsayısının 7 ile


bölümünden kalanı bulunuz.

Çözüm. Küçük Fermat Teoremine göre, 46 ≡ 1 (mod7) olacaktır. Bunun


yanında 4n ≡ 4 (mod6) ve 4n ≡ 4 + 6t olduğunu görmek zor değildir. Buna
göre, soruda istenilen cevap
¡ ¢t
a100 ≡ 4a99 ≡ 44+6t ≡ 44 · 46 ≡ 4 (mod7)
2.16. KALAN SINIFLARI [RESİDUES] 145

olacaktır.

Teorem. [Wilson Teoremi] p bir asal sayı olmak üzere, (p − 1)! ≡


−1 (modp) denkliği vardır.

Teorem. [Euler-φ Teoremi] a ve n birer pozitif tamsayı olmak üzere,


(a, n) = 1 ise aφ(n) ≡ 1 (modn) denkliği vardır.
Euler-φ Teoreminde adı geçen φ fonksiyonu herhangi bir n pozitif tamsayı
değeri için, n sayısından küçük eşit ve n ile aralarında asal olan sayıların
sayısını vermektedir. Buna göre n = pa11 · pa22 · · · · · par r olarak asal çarpanlarına
ayrılırsa, φ(n) ifadesinin eşiti
µ ¶ µ ¶ µ ¶
1 1 1
φ (n) = n · 1 − · 1− · ··· · 1 −
p1 p2 pr
olacaktır.
1000
Örnek. 77 sayısının son iki basamağını bulunuz.

Çözüm. Önce φ(100) = φ(22 ) · φ(52 ) = (22 − 2)(52 − 5) = 40 değerini


bululalım, çünkü ileri basamaklarda kullanacağız. Buna göre, Euler Teore-
minden 740 ≡ 1 (mod100) olacaktır. Benzer biçimde φ(40) = 16 olacağından
716 ≡ 1 (mod40) olarak bulunur. 1000 = 16 · 62 + 8 olduğuna göre,
¡ ¢62 ¡ ¢2
71000 ≡ 716 78 ≡ 162 78 ≡ 74 ≡ 1 (mod40)
olacaktır. Buna göre,
71000 = 1 + 40t
olacaktır. Şimdi yaptığımız bu işlemleri ve sonuçları kullanarak çözüme git-
meye çalışırsak,
1000 ¡ ¢t
77 ≡ 71+40t ≡ 7 · 740 ≡ 7 (mod100)
cevabına ulaşırız. Demek ki, sayının son iki basamağı 07 olacaktır.
Bu konu ile alakalı bilmeniz gereken teoremler örnekleriyle birlikte yukarıda
verildi. Eğer teoremlerden ispatlarını merak ettikleriniz varsa, herhangi bir
sayılar teorisi kitabından kanıtlarını öğrenebilirsiniz. Şimdi, çalışma sorularını
çözmeye geçebilirsiniz.
146 BÖLÜM 2. KONULAR

2.16.1 Çalışma Soruları


1. p bir asal sayı, w, n tamsayılar olmak üzere verilen 2p + 3p = wn denk-
lemini sağlayan tek n değerinin 1 olduğunu kanıtlayınız.
√ √
2. m, n pozitif tamsayılar olmak üzere 7 − m n > 0 ise 7− m
n > mn
1

olduğunu kanıtlayınız.

3. n ∈ Z olmak üzere, 2 28n2 + 1 + 2 bir tamsayı ise, bu tamsayının tam-
kare olduğunu kanıtlayınız.

4. (
x2 + 6y 2 = z 2
6x2 + y 2 = t2
denklem sisteminin (0, 0, 0, 0) dışında bir tamsayı dörtlüsü çözümü ol-
madığını kanıtlayınız.

5. Bir tamkare tamsayının basamakları toplamının alabileceği olası tüm


değerleri bulunuz.

6. x2 − y! = 2001 denklemini sağlayan tüm (x, y) pozitif tamsayı ikililerini


bulunuz.

7. Rakamlarının yerleri değiştirildiğinde 2 nin kuvvetine eşit olan kaç tane


2 nin kuvveti olan tamsayı vardır?

8. 44444444 sayısının basamakları toplamı A ve A sayısının basamakları


toplamı B ise B nin basamakları toplamı kaçtır?

9. y 2 = x5 − 4 denkleminin tamsayılar kümesinde çözümünün olmadığını


kanıtlayınız.

10. 1919 sayısının bir tamsayı küp ve bir tamsayı dördüncü kuvvetin topla-
mına eşit olamayacağını kanıtlayınız.

11. x2 + 3xy − 2y 2 = 122 denklemini sağlayan bir (x, y) tamsayı ikilisinin


olmadığını kanıtlayınız.

12. n > 1 olmak üzere,


2.16. KALAN SINIFLARI [RESİDUES] 147

a. 1! + 2! + 3! + · · · + n! toplamının ancak ve ancak n = 3 olduğunda


bir tamsayının tam kuvvetine eşit olabileceğini kanıtlayınız.
b. (1!)3 + (2!)3 + (3!)3 + · · · + (n!)3 toplamının ancak ve ancak n = 3
olduğunda bir tamsayının tam kuvvetine eşit olabileceğini gösteri-
niz.
13
44
44
sayısının ondalık yazılımda, sondan üçüncü yani yüzler basamağı kaçtır?

2.16.2 Çözümler
1. Eğer p = 2 ise 22 + 3¡2 = 13 ve n = 1 olacaktır.¢Eğer p ≥ 2 ise p tek sayı
olacağından (2 + 3) 2p−1 − 2p−2 3 + · · · + 3p−1 ifadesinden 5| (2p + 3p )
diyebiliriz. Buna göre 5| w olacaktır. Eğer n > 1 ise 25| wn diyebiliriz.
Dolayısıyla
2p + 3p
= 2p−1 − 2p−2 · 3 + · · · + 3p−1
2+3
ifadesi de 5 ile bölünecektir. (−1)k · (2)p−k−1 · 3k çarpımı (mod5) altında
2p−1 olduğuna göre üstte verilen toplam (mod5) altında p·2p−1 olacaktır.
Ancak, p · 2p−1 ifadesi sadece p = 5 olduğunda 5 ile bölünebilir. Buna
göre, 25 + 35 = 2751 olacağından n = 1 olacaktır.
√ √
2. Soruda verilen eşitsizliği düzenlersek, 7 − m n > 0 ise 7> m 2
n ve 7n −
m2 > 0 olacaktır. Pozitif bir tam kare ifadenin 7 ile bölümünden kalanlar
sadece 0, 1, 2 veya 4 olabilir. 2 2 2 2
√ 7n −√m > 0 olduğuna göre 7n − m ≥ 7 −
4 = 3 olacaktır. Buradan 7n ≥ m2 + √3 elde edilir. Bizim ispatlamaya
1
çalıştığımız eşitsizlikte artık m + m ≤ 7n olacaktır. Buradan,
1 p
m+ ≤ m2 + 3
m
olacaktır. Şimdi bu eşitsizliği kanıtlayalım. Aslında
µ ¶
2 2 1 1 2
m +3≥m +2+ 2 = m+
m m
olduğu açıktır. Kanıt tamamlanır.
148 BÖLÜM 2. KONULAR


3. Eğer 2 28n2 + 1 + 2 ifadesi bir tamsayı ise 28n2 + 1 = (2m + 1)2 , m ≥ 0
olacaktır. Buna göre

28n2 + 1 = 4m2 + 4m + 1

olduğuna göre,
7n2 = m (m + 1)
olacaktır. m ve m + 1 aralarında asal olduklarına göre m = 7s2 ve
m + 1 = t2 veya m = u2 ve m + 1 = 7v 2 olacaktır. Burada ikinci du-
rumun olması imkânsızdır. Çünkü (mod7)altında incelenirse kolaylıkla
görülebilir. Demek ki m + 1 = t2 olacağından,
p
2 28n2 + 1 + 2 = 2 (2m + 1) + 2 = 4m + 4 = 4 (m + 1) = 4t2 = (2t)2

olacaktır.

4. Varsayalım soruda verilen dörtlü dışında da bir çözümümüz olsun. Ayrıca


x, y, z, t sayılarının da ortak böleni olmasın. Buna göre soruda verilen
sistemdeki iki denklemi toplarsak
¡ ¢
7 x2 + y 2 = z 2 + t2

olacaktır. Tam kare bir sayının 7 ile bölümünden kalanlar yalnızca 0, 1, 2


ve 4 olabilir. z 2 + t2 ≡ 0 (mod7) ise z ve t sayılarının sırasıyla 7z0 ve 7t0
formunda olacağı açıktır. Buna göre,
¡ ¢
7 x2 + y 2 = (7z0 )2 + (7t0 )2
¡ ¢
olacağından x2 +y 2 = 7 z02 + t20 olacağından bu seferde x ve y sayıları 7
ile kalansız bölünecektir. Buna göre, (x, y, z, t) = 7 olur. Ancak bu durum
çelişkilidir. Çünkü biz çözümün en başında ortak bölenlerinin olmadığını
kabul etmiştik. Demek ki sistemin yegane çözümü olacaktır.

5. Bir tamsayının basamakları toplamı (mod9) altında sayının kendisine


denktir. Dolayısıyla eğer kontrol edilirse tam kare bir sayı (mod9) al-
tında 0, 1, 4 veya 7 olacaktır. Buna göre elimizdeki sayımızın basamak-
ları toplamı 9m, 9m + 1, 9m + 4, 9m + 7 formlarından biri olacaktır.
Şimdi bu formları tek tek inceleyelim. Eğer n = 9m ise (10m − 1)2 =
2.16. KALAN SINIFLARI [RESİDUES] 149

102m − 2 · 10m + 1 sayısının basamakları toplamı n olacaktır ve sayımız


9 · · · 980 · · · 01 formunda olacaktır. Eğer n = 9m + 1 ise (10m − 2)2 =
102m − 4 · 10m + 4 ve 9 · · · 960 · · · 04 olacaktır. Eğer m = 9m + 4 ise
(10m − 3)2 = 102m − 6 · 10m + 9 ve 9 · · · 940 · · · 09 olacaktır. Eğer n =
9m − 2 ise (10m − 5)2 = 102m − 10 · 10m + 25 ve 9 · · · 900 · · · 025 olacaktır.

6. y > 5 için y! sayısı 9 ile tam bölünür. Dolayısıyla y! + 2001 toplamı


(mod9) altında 3 sayısına denktir. Ancak tamkare bir sayının 9 ile bölü-
münden kalanlar sadece 0, 1, 4, 7 olabilir. Demek ki, y = 1, 2, 3, 4 değer-
lerinden birini alabilir. Eğer kontrol edilirse (x, y) = (45, 4) ikilisi çözüm
olarak bulunur.

7. m < n için 2m sayısı 2n sayısının rakamlarının permüte edilmesiyle oluş-


sun. Bu iki sayının da aynı sayıda basamaklarının olduğu açıktır. Dola-
yısıyla 2n < 10 · 2m olacaktır. Buna göre, n − m ≤ 3 olacaktır. Diğer
taraftan 2n ve 2m sayıları (mod9) altında denktir. Dolayısıyla
¡ ¢
2n − 2m = 2m 2n−m − 1

sayısı 9 ile bölünür. Fakat 2m ve 9 aralarında asaldır ve n − m ≤ 3 için


2n−m − 1 ≤ 7 olduğundan, bu durum imkânsızdır. Demek ki, bu şekilde
sayılar yoktur.

8. Önce B sayısının basamakları toplamının küçük bir sayı olduğunu gös-


terelim. Buna göre,
44444444 < 100005000
eşitsizliğinden, 44444444 sayısı 20000 basamaktan azdır. Dolayısıyla, A <
9 · 20000 = 180000 olacaktır. 180000 sayısından küçük sayılar arasında
basamakları toplamı en büyük olan sayı 99999 dur ve basamakları top-
lamı 45 tir. Dolayısıyla B ≤ 45 olacaktır. Buna göre B sayısının basa-
makları toplamı 39 sayısından 12 olur. Demek ki, bizim aradığımız sayı
12 den küçük olacaktır. Diğer taraftan, tüm sayılar (mod9) altında ba-
samakları toplamına denktir. Demek ki, bizim aradığımız sayı 44444444
sayısına (mod9)altında denktir. Buna göre,
¡ ¢555 4
44444444 ≡ 74444 ≡ 78 · 7 ≡ 49 · 49 ≡ 13 · 13 ≡ 7 (mod9)
150 BÖLÜM 2. KONULAR

olacaktır. Demek ki B sayısının basamakları toplamı da (mod9) altında


7 olacaktır. 12 sayısından küçük ve (mod9) altında 7 sayısına denk olan
tek sayı 7 olduğuna göre, istenilen cevap 7 olur.

9. Denklemimize (mod11) altında bakalım. Eğer (x, 11) = 1 ise


¡ 5 ¢2
x = x10 ≡ 0 veya 1 (mod11)

olacaktır. Buna göre, x5 ≡ 0, −1 veya 1 (mod11) olacaktır. Demek ki,


eşitliğin sağ kısmı (mod11) altında 6, 7 veya 8 olacaktır. Ancak y 2 ≡
0, 1, 3, 4, 5, 9 (mod11) olabilir. Demek ki, bu eşitliği sağlayan (x, y) tam-
sayı değerleri yoktur.

10. Varsayalım soruda verilen eşitlik sağlansın ve x, y tamsayılar olmak üzere,

x3 + y 4 = 1919

olsun. Şimdi toplamı oluşturan bileşenleri ayrı ayrı inceleyelim.

x3 ≡ 0, 1, 5, 8, 12 (mod13) ve y 4 ≡ 0, 1, 3, 9 (mod13)

olduğuna göre 1919 ≡ 619 ≡ 612 · 67 ≡ 7 (mod13) olarak bulunur. Buna


göre,
x3 + y 4 ≡ 7 (mod13)
denkliğini sağlayan (x, y) ikilisi olmadığından denklemin çözümü yoktur.

11. Denklemin her iki tarafını 4 ile çarpıp düzenlersek,

(2x + 3y)2 − 17y 2 = 488

eşitliğini elde ederiz. Bu denklemin her iki tarafını (mod17) altında in-
celersek
(2x + 3y)2 ≡ 12 (mod17)
olacaktır. Ancak bir tam kare (mod17) altında sadece 0, 1, 4, 9, 16, 8, 2, 15, 13
olabilir ancak 12 olamaz. Demek ki denklemi sağlayan (x, y) tam sayıları
yoktur.

12. Soruda olduğu gibi çözümüde şıklar halinde inceleyelim.


2.17. VİETE TEOREMI 151

a. n = 3 için 1! + 2! + 3! = 32 olduğunu görmek zor değildir. Şimdi bu


durumun tek olduğunu ve başka değeri bulunmadığını kanıtlayalım.
Varsayalım 1! + 2! + 3! + · · · + n! = k m , m > 1 olsun. Burada
n değerinin 9 dan büyük yada eşit olabileceğini kolaylıkla kontrol
edebiliriz. Eğer n ≥ 5 ise 1!+2!+· · ·+n! sayısının son basamağı 3 ile
biter dolayısıyla m = 2 olamaz. Çünkü tam kare sayılar 0, 1, 4, 5, 6
veya 9 ile biter. Demek ki, m ≥ 3 olmalıdır. Bunun yanında n ≥ 9
ise 1! + 2! + · · · + n! toplamı 3 ile bölünebilir. Dolayısıyla k sayısı da
3 ile bölünebilir. Buna göre, k m sayısı da 27 ile bölünebilir. Demek
ki, faktöriyellerin toplamı da 27 ile bölünebilir. Fakat, 27| a!, a ≥ 9
olduğundan, toplam 1! + 2! + · · · + 8! toplamı (mod27) altında 9
sayısına denk olacaktır. Bu durum açık çelişkidir. Demek ki n sadece
3 olmalıdır.
b. Eğer kontrol edilirse (1!)3 + (2!)3 + (3!)3 = 152 olduğunu görmek zor
değildir. Ayrıca n = 2, 4, 5 ve 6 için (1!)3 + (2!)3 + · · · + (n!)3 toplamı
bir tamsayının tam kuvvetine eşit olamaz. n ≥ 7 için bu toplam

(1!)3 + (2!)3 + (3!)3 + (4!)3 + (5!)3 + (6!)3 ≡ 7 (mod49)

olacaktır. Ancak bu kalan bir tam kuvvetin kalanı olmaz.

13. Bu sorunun çözümü okuyucuya bırakılmıştır. Euler-φ foksiyonu ve Kü-


çük Fermat Teoremlerini uygulayarak kolaylıkla çözüme ulaşabilirsiniz.

2.17 Vı̇ete Teoremi


Viete teoremi (Viyet diye okunur) genel olarak bir polinomun kökleri ile
katsayıları arasındaki ilişkiyi açıklar. Bu teoremi reel katsayılı polinomlarda
uygulayabildiğimiz gibi benzer biçimde complex katsayılı polinomlarda da kul-
lanabiliriz.
Buna göre, varsayalım x1 ve x2 değerleri P (x) = ax2 + bx + c polinomunun
kökleri olsun. Buna göre,

ax2 + bx + c = a (x − x1 ) (x − x2 ) = ax2 − a (x1 + x2 ) x + ax1 x2


152 BÖLÜM 2. KONULAR

olacağından, x1 + x2 = − ab ve x1 · x2 = ac olacaktır. Benzer biçimde eğer x1 , x2


ve x3 değerleri P (x) = ax3 + bx2 + cx + d polinomunun kökleri ise, kökler
arasında ki ilişki aşağıdaki gibi olacaktır.

 b

 x1 + x2 + x3 = −

 a
 c
x1 x2 + x2 x3 + x3 x1 =

 a

 d

 x1 x2 x3 = −
a
Eğer yukarıda verdiğimiz durumu genelleştirirsek, kökleri x1 , x2 , · · · , xn olan
bir p (x) = an xn + an−1 xn−1 + an−2 xn−2 + · · · + a2 x2 + a1 x + a0 polinomunun
kökleri ve katsayıları arasındaki ilişki aşağıdaki gibi olacaktır.
 an−1

 x1 + x2 + · · · + xn = −

 an

 an−2


 x1 x2 + x1 x3 + · · · + xn−1 xn =
an

 .
..





 x1 x2 x3 · · · xn = (−1)n a0

an
Eğer yukarıda yazdığımız eşitlikleri yeniden düzenlersek en genel form olan,
an−i
x1 x2 · · · xi + x1 x2 x4 · · · xi+1 + · · · + xn−i+1 xn−i+2 · · · xn = (−1)i
an
genel forma ulaşılır.
Viete teoeremi her ne kadar bu ders notunun genel başlığı olsada bazı denk-
lem çözümlerinde oldukça işimize yarayacak olan Newton-Girard Formülü’nü
de vereceğiz. Bu formülün kullanımı her nekadar elzem olmasa da, çözümlerin
daha pratik olması açısından önemlidir.

2.17.1 Newton-Girard Formulæ


Newton-Girard Formulæ. Eğer xn + a1 xn−1 + · · · + an = 0 denkleminin
kökleri α1 , α2 , · · · , αn ve bu köklerin kuvvetleri toplamı
Sk = α1k + α2k + α3k + · · · + αnk , k ∈ N+
2.17. VİETE TEOREMI 153

olarak verilirse, denklemin katsayıları ve k. kuvvetten köklerinin toplamları


arasındaki ilişki aşağıdaki gibidir.
S1 + a1 = 0,
S2 + a1 S1 + 2a2 = 0,
S3 + a1 S2 + a2 S1 + 3a3 = 0,
· · · · · · · · · · · · · · · · · · · · · · · · · · · = 0,
Sn + a1 Sn−1 + · · · + an−1 S1 + nan = 0
Buna göre, genel denklem
Sk + a1 Sk−1 + · · · + an Sk−n = 0, k > n
biçiminde olacaktır. Şimdi bu denklemi kullanarak problemleri çözmeye çalı-
şalım.

Örnek. a, b, c sayıları x3 − x2 + 2 = 0 denkleminin kökleri ise


a2 + b2 + c2
a3 + b3 + c3
a4 + b4 + c4

toplamlarının eşitlerini bulunuz.

Çözüm. Her ne kadar bu sorunun çözümü üstte verdiğimiz formülle


çözülebiliyor olsa da bu yöntemi sizin kullanacağınızı düşünerek çözüme daha
da elementer bir yöntemle gidelim. Buna göre,
a2 + b2 + c2 = (a + b + c)2 − 2 (ab + ac + bc) = 12 − 2 (0) = 1
olacaktır. İkinci denklemin eşiti ise, x3 = x2 − 2 olduğuna göre,
a3 + b3 + c3 = a2 − 2 + b2 − 2 + c2 − 2 = a2 + b2 + c2 − 6 = 1 − 6 = −5
olacaktır. Üçüncü toplamın eşitini bulmak içinde x3 = x2 − 2 eşitliğin de
eşitliğin her iki tarafını da x ile çarparsak, x4 = x3 − 2x ise
a4 +b4 +c4 = a3 −2a+b3 −2b+c3 −2c = a3 +b3 +c3 −2 (a + b + c) = −5−2 (1) = −7
154 BÖLÜM 2. KONULAR

olarak sonuç bulunur.

Örnek. Aşağıda verilen denklem sisteminin reel veya karmaşık tüm çö-
züm üçlülerini bulunuz.

x + y + z = 3,
x + y 2 + z 2 = 3,
2

x3 + y 3 + z 3 = 3.

Çözüm. Varsayalım x, y, z değerleri bir P (t) polinomunun kökü olsun.


Buna göre,

p (t) = (t − x) (t − y) (t − z) = t3 − (x + y + z) t2 + (xy + yz + zx) t − xyz

olacaktır. Burada
¡ 2 ¢
(x + y + z)2 x + y2 + z2 9 3
xy + yz + zx = − = − =3
2 2 2 2

eşitliğini elde ederiz. Buradan


¡ ¢
x3 + y 3 + z 3 − 3xyz = (x + y + z) x2 + y 2 + z 2 − xy − yz − zx

eşitliğini kullanarak xyz = 1 olacaktır. Dolayısıyla

p (t) = t3 − 3t2 + 3t − 1 = (t − 1)3

olacaktır. Buna göre soruda verilen sistemin tek çözümü x = y = z = 1


olacaktır.
Bu konuda öğrencilerin belkide bilmesi en önemli başlıklardan birisi de
Lagrange Interpolasyon Tekniği’dir. Şimdi bu tekniği ve bu teknik ile çözüle-
bilecek soruları inceleyelim.
2.17. VİETE TEOREMI 155

2.17.2 Lagrange İnterpolasyon Teknı̇ğı̇

Örnek. Kökleri 1, 2, 3 olan ve p(4) = 666 eşitliğini sağlayan kübik poli-


nomu bulunuz.

Çözüm. Soruda istenen polinom üçüncü dereceden olduğuna göre kesin-


likle
p (x) = a (x − 1) (x − 2) (x − 3) , a ∈ R
formunda olmalıdır. Buna göre,
666 = p (4) = a (4 − 1) (4 − 2) (4 − 3) = 6a, a = 111
bulunacağından polinom p (x) = 111 (x − 1) (x − 2) (x − 3) olacaktır.

Örnek. p (1) = 1, p (2) = 2, p (3) = 3, p (4) = 5 eşitliklerini sağlayan p (x)


polinomunu bulunuz.

Çözüm. Önceki örnekte çözüme elementer yöntemlerle ulaşabileceğimiz


için Lagrange Interpolasyon Tekniği’ni uygulamamıştık. Ancak bu soruda bu
tekniği hem uygulamasını öğreneceğiz hem de soruyu çözeceğiz. Lagrange tek-
niğine göre,
p (x) = a (x) + 2b (x) + 3c (x) + 5d (x) ,
polinomun da sırasıyla a (x) , b (x) , c (x) , d (x) birer kübik polinom olsun. Buna
göre, a (1) = 1 ve a (x) polinomunun kökleri x = 2, 3, 4; b (2) = 1 ve b (x)
polinomunun kökleri x = 1, 3, 4; c (3) = 1 ve c (x) polinomunun kökleri x =
1, 2, 4; d (4) = 1 ve d (x) polinomunun kökleri x = 1, 2, 3 olacaktır.
Bir önceki örnekte uyguladığımız tekniği yeniden burada uygularsak,
(x − 2) (x − 3) (x − 4)
a (x) = −
6
(x − 1) (x − 3) (x − 4)
b (x) =
2
(x − 1) (x − 2) (x − 4)
c (x) = −
2
(x − 1) (x − 2) (x − 3)
d (x) =
6
156 BÖLÜM 2. KONULAR

polinomlarını elde ederiz. Dolayısıyla, soruda istenilen polinom


1 3 5
p (x) = − (x − 2) (x − 3) (x − 4)+(x − 1) (x − 3) (x − 4)− (x − 1) (x − 2) (x − 4)+ (x − 1) (x − 2) (x − 3)
6 2 6
olacaktır. isterseniz soruda verilen şartları sağlayıp sağlamadığını kontrol ede-
bilirsiniz.
Sırada ki örneğimize, Bilgi Üniversitesi Cahit Arf Matematik Günleri VII-
2008 sınavında sorulan bir polinom sorusu ile devam edelim.

Örnek. n > 1 bir tamsayı ve p (x) ise derecesi (n − 1) olan bir polinom
olsun. Eğer her k = 1, · · · , n için p (k) = 1/k ise p (n + 1) ifadesinin eşitini
bulunuz.

Çözüm. Varsayalım elimizde p (x) − x1 şeklinde bir polinom olsun. Ancak


1
x ifadesi burada ki varsayımımızı bozacağından g (x) = x·p (x)−1 polinomunu
seçelim. p (x) polinomunun derecesi (n − 1) ise g (x) polinomunun derecesi
n. dereceden olacaktır. Ayrıca polinomunun kökleride olacaktır. Buna göre
aslında g (x) polinomu
g (x) = c · (x − 1) · (x − 2) · · · · · (x − n)
olarak yazılabilir. Buradaki c katsayı sabitini bulabilmek için önce g (0) = −1
olduğunu buluruz. Buna göre, x = 0
g (0) = c · (0 − 1) · (0 − 2) · · · · · (0 − n)
= c · −1 · −2 · −3 · · · · · −n
= c · (−1)n · n!
eşitliğini kullanırsak
−1 (−1)1−n
c= n =
(−1) n! n!
olacaktır. Bizden istenen p(n + 1) ise
g (n + 1) = (n + 1) p (n + 1) − 1
ve
(−1)1−(n+1)
g (n + 1) = · (n + 1 − 1) · (n + 1 − 2) · · · · · (n + 1 − n)
(n + 1)!
2.17. VİETE TEOREMI 157

olacağından
(−1)−n
(n + 1) · p (n + 1) − 1 = · n!
(n + 1)!
olacağından
· ¸
(−1)n 1 (−1)n + (n + 1)
p (n + 1) = +1 · =
n+1 n+1 (n + 1)2

olacaktır.
Konu olarak Viete Teoremi ile çözülebilecek sorular, nispeten daha kolay
sorulardır. Bu sebepten konu anlatımı içerisindeki sorularıda çalışma sorula-
rına ekledik. Soruların tamamı AMC ve AIME sınavlarından derlenmiştir.

2.17.3 Çalışma Soruları


1. ax2 + bx + c = 0 polinomunun köklerinin tersleri toplamını bulunuz.
√ √
2. x1 ve x2 değerleri x2 − 3 2x − 3 4 = 0 polinomunun kökleri ise
x1 x2
+
x2 x1
toplamını bulunuz.

3. b ve c birer sabit sayı olmak üzere verilen

(x + 2) (x + b) = x2 + cx + 6

eşitliğini sağlayan c değerini bulunuz.

4. x1 6= x2 olmak üzere 3x21 − hx1 = b ve 3x22 − hx2 = b eşitlikleri veriliyor.


Buna göre x1 + x2 toplamını bulunuz.

5. P (x) = x3 +ax2 + bx +c polinomunun köklerinin aritmetik ortası, kökle-


rin çarpımına ve katsayılar toplamına eşittir. P (x) polinomunun grafiği
y eksenini 2 noktasında kestiğine göre, b değerini bulunuz.

6. x2 + mx + n = 0 denkleminin köklerinin küpleri x2 + px + q = 0 olduğuna


göre, aşağıda verilen eşitliklerin ifadelerin doğruluğunu kontrol ediniz.
158 BÖLÜM 2. KONULAR

• p = m3 + 3mn
• p = m3 − 3mn
• p = 3mn − m3
• p + q = m3
¡ ¢3 p
• m n =q

7. ax2 + bx + c = 0 polinomunun kökleri r ve s olduğuna göre, kökleri ar + b


ve as + b olan ikinci dereceden denklemi yazınız.

8.
¡ ¢
x2 − px + p2 − 1 /4 = 0
denkleminin kökler farkının mutlak değerini bulunuz.

9. x2 + kx + 6 = 0 polinomunun kökleri x1 ve x2 olarak verildiğinde x2 −


kx + 6 = 0 polinomunun kökleri sırasıyla x1 + 5 ve x2 + 5 olmaktadır.
Buna göre, k değerini bulunuz.

10. x2 + mx + n = 0denkleminin kökleri x2 + px + m = 0 denkleminin kök-


lerinin iki katı olduğuna göre, n/p değerini bulunuz.(n ve p değerlerinin
her biri sıfırdan farklıdır.)

11. a ve b birer reel sayı olmak üzere,

8x3 + 4ax2 + 2bx + a = 0

denkleminin birbirinden farklı üç pozitif kökü vardır. Köklerin log2 ta-
banındaki toplamları 5 olduğuna göre, a değerini bulunuz.

12. x2 − 63x + k = 0 denkleminin iki kökü de birer asal sayıdır. Buna göre
k değerinin alabileceği kaç farklı değer vardır.

13.
√ 12
4
x= √
7− 4x
denklemini sağlayan değerler toplamını bulunuz.
2.17. VİETE TEOREMI 159

14. p
x2 + 18x + 30 = 2 x2 + 18x + 45

denklemini sağlayan değerler çarpımını bulunuz.

15. p, q ve r değerleri x3 − x2 + x − 2 = 0 denklemini birer kökü olduğuna


göre,
p3 + r 3 + q 3

toplamının eşitini bulunuz.

16. x1000 − 10x + 10 = 0 denkleminin kökleri r1 , r2 , r3 , · · · , r1000 olduğuna


göre,
r11000 + r21000 + r31000 + · · · + r1000
1000

değerini bulunuz.

17. 
 a + 2b + 4c = 12

ab + 4bc + 2ac = 22


abc = 6

denklem sistemini sağlayan (a, b, c) üçlülerini bulunuz.

2.17.4 Çözümler
1. ax2 + bx + c = 0 denkleminin köklerine ve diyelim. Buna göre soruda
istenilen toplam
1 1
+
x1 x2
ifadesinin eşiti olacağına göre,

1 1 x1 + x2 −b/a b
+ = = =−
x1 x2 x1 · x2 c/a c

olarak istenilen toplam bulunur.


160 BÖLÜM 2. KONULAR

2. Önceki sorudaki benzer yöntemleri kullanarak çözüme gidelim. x1 ve x2


denklemim birer kökü olduğuna göre,

x1 x2 x2 + x22 (x1 + x2 )2 − 2x1 · x2


+ = 1 = = −3
x2 x1 x1 · x2 x1 · x2
olarak bulunacaktır.
3. Denklemin kökleri −2 ve −b olduğuna göre kökler çarpımı (−2 · −b) = 6
ve kökler toplamı −b − 2 = c olacağından, soruda istenilen değerler
kolaylıkla bulunur.
4. Soruda verilen iki denklem birbirine eşit olduğuna göre,

3x21 − 3x22 = hx1 − hx2

olacaktır Burada x1 6= x2 olduğuna göre, soruda istenilen ifade x1 + x2 =


h/3 olarak bulunacaktır.
5. Varsayalım denklem kökleri x1 , x2 , x3 olsun. Buna göre,
x1 + x2 + x3
= x1 · x2 · x3 = a + b + c + 1
3
olacaktır. Soruda ayrıca P (0) = 2 olarak verildiğine göre, c = 2 olacaktır.
Buna göre yeni eşitliğimizi sağlayan değerler a = 12 ve b = −17 olarak
bulunur.
6. Varsayalım denklemlerimizin köklerini x31 , x32 olarak alalım. Buna göre,
¡ ¢ ³ ´ ¡ ¢
x31 +x32 = (x1 + x2 ) x21 + x22 − x1 x2 = −m (x1 + x2 )2 − 3x1 x2 = −m m2 − 3mn

olacaktır. Buna göre,

x31 + x32 = −p = −m3 + 3mn

olacağından eşitliklerden

m3 − 3mn = p

olanı doğrudur.
2.17. VİETE TEOREMI 161

7. Bu sorunun çözümü okuyucuya bırakılmıştır.

8. Denklemin kökler toplamı ve kökler çarpımını kullanmaya çalışalım. Buna


göre,
p2 − 1
x1 + x2 = p, ve x1 · x2 =
4
olacaktır. Diskriminantı kontrol edersek de
¡ ¢1
∆ = p2 − 4 p2 − 1 = p2 − p2 + 1 = 1
4
ise denklemin kökleri,

p+1 p−1
x1 = ve x2 =
2 2
olacağına göre, soruda istenilen cevap,

|x1 − x2 | = 2

olacaktır.

9. Bu sorunun çözümü okuyucuya bırakılmıştır.

10. Bu sorunun çözümü okuyucuya bırakılmıştır.

11. Bu sorunun çözümü okuyucuya bırakılmıştır.

12. Bu sorunun çözümü okuyucuya bırakılmıştır.



13. Soruda verilen denklemde 4
x = a olarak alınırsa, yeni denklem

a2 − 7a + 12 = 0

olacaktır. Buradan a1 = 4 ve a2 = 3 olacağından istenen x değerleri


x = 256 ve x = 81 olarak bulunur.

14. Bu sorunun çözümü okuyucuya bırakılmıştır.


162 BÖLÜM 2. KONULAR

15. p, q, r denklemin kökleri olduğuna göre,


p3 + r3 + q 3 = p2 + q 2 + r2 − p − q − r + 6
= (p + q + r)2 − 2 (pq + pr + qr) − (p + q + r) + 6
= 1−2·1+1+6=6
olarak cevap bulunur.
16. Her bir ri değeri denklemi sağladığına göre
r1100 − 10r1 + 10 = 0
olacaktır. Eğer her bir r1 , r2 , · · · , r100 değerini yerine koyalım buna göre,

r1100 = 10r1 − 10
r1100 = 10r2 − 10
··· = ···
100
r100 = 10r100 − 10
eşitliklerini elde ederiz. Bu eşitlikleri alt alta toplarsak

= 10 (r1 + r2 + · · · + r100 ) − 10 · 1000


= 10 · 10 − 10 · 1000
= 10 · (−990)
= −9900
cevabı elde edilir.
17. Eğer a, 2b ve 4c değerlerini bir denklemin kökleri olarak alalım. Buna
göre denklemimiz
0 = (x − a) (x − 2b) (x − 4c)
¡ ¢
= x2 − 2bx − ax + 2ab (x − 4c)
¡ ¢
= x2 − x (2b + a) + 2ab (x − 4c)
= x3 − x2 (a + 2b + 4c) + x (2ab + 4ac + 8bc) − 8abc
= x3 − 12x2 + 22x − 48
2.18. BAĞINTI SAYILARI 163

olduğuna göre gerçektende a, 2b ve 4c değerleri 3. dereceden bir denkle-


min kökleridir. Buna göre toplam da 3 · 3 · 2 = 6 tane üçlü bulunabilir.

2.18 Bağıntı Sayıları


A kümesi boş kümeden farklı bir küme olmak üzere A × A kartezyen çar-
pım kümesinin tüm alt kümelerine birer bağıntı denir. Varsayalım kümemiz
A = {a, b, c} olsun. Önce kartezyen çarpımı ve bağıntıyı tablo ile gösterelim.
Tablo çizmekteki amacımız hem kartezyen çarpımı hemde seçebileceğimiz her-
hangi bir bağıntıyı tablo üzerinde daha kolay açıklayabilmemiz olacaktır.

a b c
a ∗
b
c ∗ ∗
Yukarıdaki tabloda verilen bağıntının ikilileri, eğer bu bağıntıya β dersek,
β = {(a, b), (c, a), (c, c)}
olacaktır. Yıldızları tablodaki karelere rastgele koyarak, yada hiçbir kareye
koymayarak, bağıntılar elde edebiliriz. Bu noktadan sonra karşılaşabileceğimiz
soruladan biriside, bu şekilde kaç bağıntının yazılabileceği olacaktır. Yukarıda
çizdiğimiz tablo üzerinden düşünürek başlayalım. Eğer tablodaki her bir hüc-
reye, birer yıldız koyarsak toplamda 3 × 3 = 9 yıldız olacaktır. Bağıntılarımız
kartezyen çarpımlarımızın birer alt kümesi olduğuna göre, 29 tane bağıntımız
olacaktır.
Bu durumu genelleştirelim. Eğer s(A) = n olarak alırsak, bağıntı sayımız
2
2n×n = 2n
olacaktır. Şimdi, bağıntı çeşitlerini ve sayılarını inceleyelim.

Yansıyan Bağıntı. x ∈ A olmak üzere, ∀x ∈ A için (x, x) ikilisi bağıntı-


nın bir elemanı ise bağıntımız yansıyan olacaktır. Eğer A = {a, b, c} ise,
164 BÖLÜM 2. KONULAR

a b c
a ∗
b ∗
c ∗
yukarıdaki tabloda da görüldüğü üzere, en küçük yansıyan bağıntımız
β = {(a, a), (b, b), (c, c)}
olacaktır. Öyleyse, bu bağıntıya ekleyeceğimiz her ikili için bağıntı hala yansı-
yan bağıntı olarak kalacaktır. İşaretli olmayan 6 kutu için, kartezyen çarpımın
26 tane alt kümesi olacağından 64 tane yansıyan bağıntı yazılabilir. Eğer A
kümesinin n tane elemanı olduğunu varsayarsak, A × A kümesinin n2 tane
elemanı olacaktır. Oluşturulacak tablonun köşegeni üzerindeki n elemanı çı-
karırsak kalan n2 − n tane elemanın alt kümelerini almamız yeterlidir. Yani
yansıyan bağıntı sayımız
2
2n −n = 2n·(n−1)
olacaktır.
İngilizce kaynaklarda Irreflexive Relation olarak geçen, bağıntıya biz Türkçe
olarak Yansımaz Bağıntı diyelim. Buna göre, eğer yazılan bağıntının içinde
x ∈ A için (x, x) ∈/ β durumu varsa, bağıntımız bir yansımaz bağıntı olacaktır.
Daha basit bir anlatımla çizilecek tablonun köşegeni üzerinde bağıntılar yazı-
lırken hiç yıldız bulunmayacak. Demek ki, yansıyan bağıntı sayısı ile yansımaz
bağıntı sayısı aynıdır. Ayrıca Yansımaz Bağıntı ile Yansıyan Olmayan Bağıntı
birbirlerinden farklıdır.

Simetrik Bağıntı. Bir bağıntının simetrik olması için, (x, y) elemanı ba-
ğıntının içindeyse, (y, x) bağıntısınında bağıntının içinde olması şartı aranır.
Eğer tablo üzerinden düşünürsek, tablonun kşegene göre simetrik olması gere-
kir.

a b c
a ∗
b ∗ ∗
c ∗ ∗
2.18. BAĞINTI SAYILARI 165

Örneğin yukarıda verilen tabloda verilen bağıntı simetriktir. Bağıntının ele-


manları
{(a, b), (b, a), (b, c), (c, b), (a, a)}
olacaktır. Şimdi durumu genelleştirelim. Eğer s(A) = n olarak alırsak, simetrik
bağıntı yazacağımız için köşegen üstündeki ve üzerindeki noktaları seçip bu
noktaların oluşturduğu kümenin alt kümelerini almamız yeterlidir. Durumu
genelleştirelim. Tüm ikililerin sayısı n2 , köşegen üzerindeki noktaların sayısı n
ise,
n2 − n n · (n + 1)
+n=
2 2
istediğimiz ikililerin sayısını alt kümelerin sayısı, yani Simetrik Bağıntı Sayısı
n·(n+1)
2 2

olacaktır.

Anti-Simetrik Bağıntı. x ve y birbirinden farklı elemanlar olmak üzere


(x, y) ve (y, x) aynı anda bağıntının elemanı değilse bağıntı antisimetrik bir
bağıntı olur. Örneğin aşağıda tablosu verilen bağıntı bir antisimetrik bağıntı-
dır.

a b c
a ∗ ∗
b
c ∗ ∗
Bağıntının elemanları,
(a, a), (a, b), (c, a), (c, c)
olacaktır. Bağıntıda (a, b) varken (b, a) ve (c, a) varken (a, c) yoktur. Ama kö-
şegenler üzerindeki elemanlar istenildiği gibi seçilebilir. Peki, kaç tane simetrik
bağıntı vardır? Bu sayıyı bulmak için köşegen üstünde olma ve olmama du-
rumlarını ayrı ayrı değerlendirelim. Eğer s(A) = n olarak alırsak, köşegen
üzerindeki, elemanlar için 2n tane durum vardır. Köşegen üzerinde olmayan
elemanlar için ise üç durum vardır. Bunlar,
166 BÖLÜM 2. KONULAR

(x,y) (y,x)
içinde dışında
dışında içinde
dışında içinde
şeklinde olacaktır. (x, y) ve (y, x) ikilisinin ikiside zaten bağıntının içinde ola-
maz. Kümemizde n tane eleman olduğunu varsayarsak,
µ ¶
n n · (n − 1)
C =
2 2
farklı durum vardır. Her ikili için üç farklı durum olduğuna göre,
3n·(n−1)/2
farklı durum oluşacaktır. Buna göre toplam antisimetrik bağıntı sayısı
2n · 3n·(n−1)/2
olacaktır.
Ayrıca antisimetrik bağıntı ile simetrik olmayan bağıntılar birbirinden farklı
durumlardır. Örneğin,
{(a, b), (a, c), (c, a), (c, c)}
bağıntısı simetrik değildir. Çünkü, (a, b) bağıntının elemanı iken (b, a) bağıntı-
nın elemanı değildir. Benzer biçimde bağıntı antisimetrik değildir çünkü hem
(a, c) hemde (c, a) bağıntının birer elemanıdır.

Anti-Simetrik Bağıntı ve Yansıyan Bağıntı. Eğer bu durumu antisi-


metrik bağıntı ile karşılaştırırsak, aradaki tek fark, bu durumda her iki köşegen
üzerindeki tüm karelerde birer yıldız olacaktır. Ancak hala,
n · (n − 1)
2
eleman için 3 farklı durum vardır. Bu sebepten de, yansıyan ve antisimetrik
bağıntı sayısı
3n·(n−1)/2
olacaktır.
2.18. BAĞINTI SAYILARI 167

2.18.1 Çalışma Soruları


Aşağıdaki çalışma soruları Tübitak XVI. Bilgisayar Olimpiyatları sına-
vında sorulmuştur. Soruların çözümlerini önce kendiniz yapmaya çalışınız.
Daha sonra çözümleri okuyunuz.

A = {0, 1, {1}, {1, {1}}} küme olarak tanımlanıyor.

1. A kümesi üzerinde kaç farklı ikili bağıntı tanımlanabilir?

2. A kümesi üzerinde kaç farklı simetrik olan ikili bağıntı tanımlanabilir?

3. A kümesi üzerinde kaç farklı simetrik ve yansıma özeliğine sahip ikili


bağıntı tanımlanabilir?

2.18.2 Çözümler
1. A kümesinin sorudada görüldüğü üzere 4 tane elemanı vardır. Buna göre
A kümesi üzerinde tanımlanabilecek bağıntı sayısı için önce kartezyen
çarpımın elemanlarını bulalım. s(A × A) = 4 · 4 = 16 olacaktır. Her
bir bağıntı kartezyen kümenin alt kümesi olduğuna göre, alt kümeleri
bulmamız yeterli olacaktır. Buna göre istenen cevap

24×4 = 216

olacaktır.
n·(n+1)
2. Simetrik bağıntı sayısı 2 2 olduğuna göre, n = 4 için istenen cevap
210 olacaktır.

3. Bağıntımız hem yansıyan hem simetrik olacağına göre, bağıntının içinde


mutlaka
{(a, a), (b, b), (c, c), (d, d)}
ikilileri mutlaka bulunacaktır. Buna göre bu bağıntının içine eleman ola-
rak ekleyeceğimiz her ikili bir yansıyan bağıntı olacaktır. Bağıntımız eğer
aynı anda simetrik bağıntı özeliğinide taşıyacağına göre, bağıntının içinde
168 BÖLÜM 2. KONULAR

(a, b) eleman olarak bulunuyorsa, (b, a) ikilisi de eleman olarak bağın-


tının içinde bulunacaktır. Geriye kalan 12 eleman içindeki her bir ikili
simetrikliği sağlamak için yanında bir ikiliyi daha taşıyacağından aslında
bizim 6 elemanlı bir kümeden alt kümeler seçmemi gerekir. Buna göre,
istenen sayı 26 = 64 olacaktır.

2.19 Lı̇neer Denklemlerı̇n Tamsayı Çözümlerı̇


Genel olarak tekrarlı permütasyon konusu altında verilen bu konuda asıl
amacımız
x1 + x2 + · · · + xr = n, n ∈ Z+ ∨ n ∈ Z+ ∪ {0}
formunda verilen bir lineer denklemin pozitif tamsayı veya negatif olmayan
tamsayılardaki çözüm sayılarını bulmak olacaktır. Önce teoremleri ve çözümlü
örnekleri dikkatli bir şekilde çalışarak konu sonunda verilen çalışma sorular ile
konuyu daha iyi kavramaya çalışınız. Önce bir teoremle başlayalım.

Teorem (De Moivre) . n pozitif bir tamsayı olmak üzere verilen

x1 + x2 + x3 + · · · + xr = n

denkleminin pozitif tamsayı çözümlerinin sayısı


µ ¶
n−1
r−1

olacaktır.

Kanıt . n sayısını n = 1 + 1 + · · · + 1 + 1 olarak yazalım. Bu toplamda


n tane 1 ve (n − 1) tane + işaretinin olduğu açıktır. Buna göre, n toplamını
r tane parçaya bölmek için bizim (r − 1) tane + işaretini seçmemiz yeterli
olacaktır. Buna göre seçimimiz
µ ¶
n−1
r−1

olacağından ispat tamamlanır. ¥


2.19. LİNEER DENKLEMLERİN TAMSAYI ÇÖZÜMLERİ 169

Örnek. 9 rakamını üç pozitif tamsayının toplamı olarak kaç farklı biçimde
yazabiliriz? Mesela, 1 + 1 + 7 ve 7 + 1 + 1 birbirinden farklı iki toplamdır.

Çözüm. Soruyu eğer denklem biçiminde yazarsak, aslında sorulan soru

a + b + c = 9 a > 0, b > 0, c > 0

denkleminin çözüm sayısı olacaktır. Öyleyse istenen cevap


µ ¶ µ ¶
9−1 8
= = 28
3−1 2

olacaktır.

Sonuç . n pozitif bir tamsayı olmak üzere verilen

y1 + y2 + · · · + yr = n

denkleminin negatif olmayan tamsayılardaki çözüm sayısı


µ ¶
n+r−1
r−1

olacaktır.

Kanıt . Denklemde yr = xr − 1, xr ≥ 1 değişken değiştirmesini yaparsak

x1 − 1 + x2 − 1 + · · · + xr − 1 = n

olacağından
x1 + x2 + · · · + xr = n + r
olacaktır. Kanıtın bundan sonrası De Moivre teoreminin bir uygulamasına
dönüştüğüne göre µ ¶
n+r−1
r−1
çözüm sayısı olacaktır. ¥
170 BÖLÜM 2. KONULAR

Örnek.

a + b + c + d = 100, a ≥ 30, b ≥ 21, c ≥ 1, d ≥ 1

durumlarını sağlayan kaç farklı (a, b, c, d) tamsayı dörtlüsü seçilebilir?

Çözüm. Önce uygun değişken değiştermeleri yapalım. a = a0 + 29, b =


b0 + 20 olarak alırsak yeni denklemimiz

a0 + b0 + c + d = 50

olacaktır. Bu denkleminde pozitif tamsayı çözümleride


µ ¶
49
= 18424
3
olacaktır.

Örnek. 8 katlı bir binanın asansörüne binen 5 kişi asansörden katlara


kaç farklı biçimde dağılabilirler?

Çözüm. Aslında soruda bulunması istenen sayı

x1 + x2 + · · · + x8 = 5

denkleminin negatif olmayan çözüm sayısıdır. Buna göre cevap


µ ¶ µ ¶
8+5−1 12
= = 792
8−1 7
olacaktır.

Örnek. a + b + c + d ≤ 2009 eşitsizliğini sağlayan kaç farklı negatif


olmayan tamsayı (a, b, c, d) dörtlüsü vardır?

Çözüm. a+b+c+d ≤ 2009 denkleminin negatif olmayan tamsayı çözüm


dörtlülerinin sayısı

a + b + c + d + f = 2009, f ≥ 0
2.19. LİNEER DENKLEMLERİN TAMSAYI ÇÖZÜMLERİ 171

denkleminin çözüm dörtlülerinin sayısına eşittir. Benzer biçimde son yazdığı-


mız denklemin çözümlerinin sayısıda

a1 − 1 + b1 − 1 + c1 − 1 + d1 − 1 + f1 − 1 = 2009

denkleminin çözüm sayısı ile aynı olacağına göre istenen cevap


µ ¶
2013
4
olacaktır.
Buraya kadar yaptığımız örneklerde, değişkenlere dair yaptığımız sınırla-
malar hep tek yönlüydü. Sıradaki örneğimizde durum biraz daha farklı.

Örnek.

a + b + c + d = 100, 1 ≤ a ≤ 10, b ≥ 0, c ≥ 2, 20 ≤ d ≤ 30

olamak üzere verilen denklemin tüm tamsayı çözüm dörtlülerinin sayısını bu-
lunuz.

Çözüm. Çözümü durum, durum inceleyerek sürdürelim. Eğer, a ≥ 1, b ≥


0, c ≥ 2, d ≥ 20 olarak alırsak denklemimizin
µ ¶
80
= 82160
3
farklı çözümü olacaktır. Eğer a ≥ 11, b ≥ 0, c ≥ 2, d ≥ 20 olarak alırsak çözüm
sayımız µ ¶
70
3
kadar olacaktır. Eğer a ≥ 1, b ≥ 0, c ≥ 2, d ≥ 31 olarak alırsak çözüm sayımız
µ ¶
69
3
kadar olacaktır. Bu iki durumun kesişimi ise
µ ¶
59
3
172 BÖLÜM 2. KONULAR

kadar olacağından birleşim kümesinin eleman sayısı


µ ¶ µ ¶ µ ¶
70 69 59
+ − = 74625
3 3 3
olacaktır. Buna göre istenen çözüm sayısı
µ ¶ µ ¶ µ ¶ µ ¶
80 70 69 59
− − + = 7335
3 3 3 3
olacaktır.
Genel olarak karşınıza çıkabilecek lineer denklemler ve pozitif tamsayı çö-
zümleri örneklerle açıklandı. Şimdi bir çoğu Tübitak Bilgisayar Olimpiyatları
birinci aşama sınavında çıkmış çalışma sorularını yapmaya çalışarak konuyu
daha iyi kavramaya çalışınız.

2.19.1 Çalışma Soruları


1. 1 ile 1000 arasındaki sayılardan kaç tanesinin rakamlarının toplamı 7
yapar?
2. (5a + 8b + 2c)15 açıldığında kaç terim elde edilir?
3. 0 ≤ x1 , x2 , x3 , x4 ≤ 7 olduğunda x1 + x2 + x3 + x4 = 18 denkleminin kaç
farklı tamsayı çözümü vardır?
4. 20 adet boş kartın her birinin üzerine kare, daire ve üçgen resmi çizi-
lebilmektedir. Buna göre kaç farklı şekilde 20 karttan oluşan bir demet
oluşturulabilir?
5. 100 tane birbirinin aynısı top ve 5 adet birbirinden farklı kutu bulun-
maktadır. Her bir kutuda en az 6 adet top bulunacak şekilde topları kaç
farklı şekilde kutulara dağıtabiliri?
6. 100 tane birbirinin aynısı top ve 5 adet birbirinden farklı kutu bulun-
maktadır. Her bir kutuda en fazla 40 adet top bulunacak biçimde kaç
farklı şekilde kutulara dağıtabiliriz?
7. Kırmızı, beyaz ve mavi zarların üçü birden atıldığında, kaç farlı durumda
gelen sayıların toplamı 10 yapar?
2.20. FONKSİYONEL DENKLEMLER 173

8. a + b + c + d = 98 eşitliğini sağlayan kaç (a, b, c, d) pozitif tek tamsayı


dörtlüsü vardır? (AIME 1998)

2.19.2 Çözümler
Konu anlatımı içerisinde verilen çözümler, örnekleri çözmeniz için yeterli-
dir. Biraz uğraşark çözümlere ulaşabilirsiniz.

2.20 Fonksı̇yonel Denklemler


Bilinmeyenlerinin birer fonksiyon olduğu denklemlere genel olarak fonk-
siyonel denklem denilir. Benzer biçimde bilinmeyenlerin polinomlardan oluş-
tuğu denklem sorularınıda bu başlık altında alabiliriz. Ancak bu tür soruların
ne yazıkki genel bir çözüm yöntemi yoktur. Belkide bu sebepten matematik
yarışmalarında sıklıkla sorulan sorular haline gelmişlerdir. Bu başlık altında
genel olarak tek değişkenli fonksiyonel denklemlerin genel çözüm teknikleri ve
çok değişkenli fonksiyonel denklemler konuları ele alınmıştır.

2.20.1 Tek Değişkenliler - Temel Teknikler


Tek değişkenlilerin çözümlerine uygun dönüşümler yapılarak ulaşılır. Ör-
nekleri inceleyerek devap edelim.

Örnek. f (x + 1) = x2 − 3x + 2 fonksiyonel denklemini çözünüz.

Çözüm. t = x+1 ise x = t−1 olacağından f (t) = (t−1)2 −3(t−1)+2 =


t2 − 5t + 6 eşitliğinden f (x) = x2 − 5x + 6 olarak bulunur.

Örnek.
x+1 x2 + 1 1
f( )= +
x x2 x
fonksiyonel denklemini çözünüz.

Çözüm. t = (x + 1)/x olarak alırsak x = 1/(t − 1) olacağından


1 2
( t−1 ) +1 1
f (t) = 1 2 + 1 = t2 − t + 1
( t−1 ) ( t−1 )
174 BÖLÜM 2. KONULAR

bulunur. Buna göre f (x) = x2 − x + 1 olarak bulunur.


Genel olarak çözümlerde uyguladığımız teknik, f [ϕ(x)] = g(x) eşitliğini f
için çözmektir. Eğer ϕ fonksiyonunun tersi varsa, t = ϕ(x) olarak alabiliriz.
Dolayısıyla da
f (x) = g[ϕ−1 (x)]
olur.

Örnek. f (ex ) = x3 + sin x fonksiyonel denklemini çözünüz.

Çözüm. t = ex değişken değiştirmesini yaparsak, x = ln t olacağından

f (x) = (ln |x|)3 + sin(ln |x|)

olacaktır.

Örnek. a 6= ±1 olmak üzere verilen


x
f( ) = af (x) + ϕ(x)
x−1
fonksiyonel denklemini çözünüz.

Çözüm. t = x/(x − 1) değişken değiştirmesini yaparsak, x = t/(t − 1)


olacaktır. Buna göre,
t t t
f (t) = af ( ) + ϕ( ) = a(af (t) + ϕ(t)) + ϕ( )
t−1 t−1 t−1
olacağından, istenilen fonksiyon
x
aϕ(x) + ϕ( x−1 )
f (x) =
1 − a2
olacaktır.
Bir fonksiyonel denklem değişkenlere bağlı cebirsel ifadeler ihtiva edebilir.
Bu tür sorularda uygulanacak temel teknik eşzamanlı fonksiyonlar oluştur-
maktır. Şimdi aşağıdaki örnekleri inceleyelim.
2.20. FONKSİYONEL DENKLEMLER 175

Örnek.
3f (x) + 2f (1/x) = 4x
fonksiyonel denklemini çözünüz.

Çözüm. x değişkeni yerine 1/x alınırsa,

3f (1/x) + 2f (x) = 4/x

olacaktır. Soruda verilen fonksiyon −3/2 ile çarpılıp son bulunan denklem ile
toplanırsa, istenilen denklem

12x2 − 8
f (x) =
5x
olarak bulunacaktır.

Örnek. x 6= 0 olmak üzere verilen


1 1
· f (−x) + f ( ) = x
x x
fonksiyonel denklemini çözünüz.

Çözüm. Soruda verilen denklemde x yerine −x yazarsak,


1 1
− f (x) + f (− ) = −x
x x
1
bulunur. Eğer x yerine x alınırsa

1 1
xf (− ) + f (x) =
x x
denklemi elde edilecektir. Bu iki fonksiyonel denklemden, istenilen fonksiyon
1 1
f (x) = (x2 + )
2 x
olarak elde edilir.
176 BÖLÜM 2. KONULAR

Fonksiyonel denklemlerin çözümü yapılırken eğer fonksiyonun özelikleride


göz önüne alınırsa çözüme ulaşılması daha kolay olacaktır. Öyle ki fonksi-
yonun sürekliliği, monotonluğu, sınırlılığı, türevlenebilir olması gibi bilgilerin
bilinmesi çözümlerde faydalı olacaktır.

Örnek.
f (x + 1) + f (x − 1) = 2x2 − 4x
fonksiyonel denklemini çözünüz.

Çözüm. İki fonksiyonun toplamı ikinci dereceden olduğuna göre f (x)


fonksiyonuda ikinci dereceden olacaktır. Buna göre, f (x) = ax2 + bx + c ise

2ax2 + 2bx + 2(a + c) = 2x2 − 4x

eşitliğinden f (x) fonksiyonu f (x) = x2 − 2x − 1 olarak bulunur.

2.20.2 Çok Değişkenliler


Bazı çok değişkenli fonksiyonel denklemlerin çözümlerinde simetriden fay-
dalanılarak denklemi tek değişkenliye indirgemek çözümü oldukça kolaylaştırır.
Aşağıdaki örnekler ve çözümler bu durumla alakalıdır.

Örnek.

(x − y)f (x + y) − (x + y)f (x − y) = 4xy(x2 − y 2 )

fonksiyonel denklemini çözünüz.

Çözüm. Eğer soruda verilen eşitliği düzenlersek

f (x + y) f (x − y)
− = 4xy
x+y x−y

olacaktır. Buna göre


f (x)
g(x) =
x
2.20. FONKSİYONEL DENKLEMLER 177

fonksiyonunu alalım. Eğer

g(x + y) − g(x − y) = 4xy

ise
g(x + y) − (x + y)2 = g(x − y) − (x − y)2
olacağından g(x) − x2 = k, k ∈ R ve g(x) = x2 + k, f (x) = x3 + kx olarak
bulunur.
Değişken sayısını düşürmenin bir diğer yoluda uygun değerleri değişkenlere
atamaktır. Aşağıdaki örnek bu metodla çözülmüştür.

Örnek. f (x) · f (y) − f (xy) = x + y fonksiyonel denklemini f (x) için


çözünüz.

Çözüm. Varsayalım y = 0 olsun. Buna göre, f (0)[f (x)−1] = x olacaktır.


f (0) 6= 0 olacağına göre,
x
f (x) = +1
f (0)
eşitliği elde edilir. Benzer biçimde x = y = 0 alınırsa f (0)[f (0) − 1] = 0 ise
f (0) = 1 olacağından f (x) = x + 1 olarak bulunur. Gerçektende bulduğumuz
fonksiyon soruda verilen denklemi sağlamaktadır.

Örnek. f fonksiyonu

f (x) + f (y) = f (x + y) − xy − 1

eşitliğini sağlamaktadır. Buna göre, eğer f (1) = 1 ise f (n) = n eşitliğini


sağlayan negatif tamsayı değerlerini bulunuz.

Çözüm. x = 1 olarak alırsak, f (y + 1) − f (y) = y + 2 ve y = 0 alırsak


f (0) = −1 olacaktır. n ≥ için

n−1
X n−1
X (n + 1)(n + 2)
f (n+1)+1 = f (n)−f (0) = (f (y+1)−f (y)) = (y+2) = −1
2
y=0 y=0
178 BÖLÜM 2. KONULAR

olacağından
+ 3n+2
n2 n2 + 3n − 2
f (n) = −2=
2 2
olacaktır. Eğer x = n ve y = −n alırsak, f (n) + f (−n) = n2 − n ve

(−n)2 + 3(−n) − 2
f (−n) = n2 − 2 − f (n) =
2
olacaktır. Demekki,
n2 + 3n − 2
f (n) =
2
eşitliği negatif n değerleri içinde geçerlidir. Sorudaki f (n) = n durumu için

n2 + 3n − 2
=n
2
ise (n − 1)(n + 2) = 0 olacağından n = 1 veya n = −2 olacaktır.
Fonksiyonel denklemlerin çözümleri yapılırken bazı kullanışlı sonuçların bi-
linmesinde fayda vardır. Örneğin aşağıda verilenler bunlardan sadece birkaçı-
dır. Buna göre, f sürekli bir fonksiyon olmak üzere

i. f (x + y) = f (x) + f (y) ise f (x) = c,

ii. f (x + y) = f (x) · f (y) ise f (x) = cx

iii. f (xy) = f (x) + f (y) ise f (x) = c ln x

iv. f (xy) = f (x) · f (y) ise f (x) = xc

olacaktır. Sıradaki örneğimizin çözümünde bu sonuçları kullanabiliriz.

Örnek. (Jensen Fonksiyonel Denklemi) x, y ∈ R olmak üzere verilen


µ ¶
x+y f (x) + f (y)
f =
2 2

eşitliğini sağlayan tüm sürekli f fonksiyonlarını bulunuz.


2.20. FONKSİYONEL DENKLEMLER 179

Çözüm. Eğer y = 0 olarak alınırsa,


³ x ´ f (x) + f (0)
f =
2 2
olacaktır. Buna göre,
µ ¶
f (x) + f (y) x+y f (x + y) + f (0)
=f =
2 2 2
olacaktır. Yani, f (x + y) = f (x) + f (y) − f (0) olarak bulunur. Varsayalım
h(x) = f (x) − f (0) ise h(x + y) = h(x) + h(y) olacağından h(x) = cx ve
f (x) = cx + f (0) olarak bulunur.

Örnek. f (x+y) = g(x)+h(y) eşitliğini sağlayan tüm sürekli fonksiyonları


bulunuz.

Çözüm. Eğer y = 0 olarak alırsak h(0) = b ise f (x) = g(x) + b ve eğer


x = 0 olarak alırsak benzer biçimde f (y) = a + h(y) olacaktır. Buna göre,
f (x + y) = [f (x) − b] + [f (y) − a] = f (x) + f (y) − (a + b)
olacağından F (x) = f (x) − a − b ve F (x + y) = F (x) + F (y) olacaktır. F
fonksiyonu toplamsal sürekli fonksiyon olduğundan F (x) = kx olacaktır. Sonuç
olarak f (x) = kx + a + b, g(x) = kx + a ve h(x) = kx + b olacaktır.
Olimpiyat sınavlarında özellikle TST ve kamp sınavlarında sorulan soru-
ların çözümleri oldukça zordur. Belli teknikler kullanarak çözümeye gitmeye
çalışmak her ne kadar gerekli olsada çoğu zaman yereli değildir. Ancak bazı
yardımcı özelikleri araştırmak sanırız çözüm yolundaki en büyük yardımcılar-
dan olacaktır. Bu sebepten elimizdeki fonksiyonun
i. Birebir veya örtenliği,
ii. Periyodikliği veya tek, çift fonksiyon olma durumu,
iii. Artan yada azalan olma durumu,
iv. Simetrikliği
gibi özelliklerinin olup olmadığını bilmek, çoğu zaman çözümü ciddi biçimde
kolaylaştıracaktır.
180 BÖLÜM 2. KONULAR

2.20.3 Çalışma Soruları


1. f : R → R olmak üzere verilen

f (f (x + y)) = f (x + y) + f (x)f (y) − xy x, y ∈ R

eşitliğini sağlayan tüm f fonksiyonlarını bulunuz.

2. f : Z → R ve x + y toplamı 3’ün tam katı olduğuna göre,


µ ¶
x+y f (x) + f (y)
f =
3 2
eşitliğini sağlayan f − fonksiyonlarını bulunuz.

3. f : R → R olmak üzere verilen

f (xf (y) + x) = xy + f (x)

eşitliğini sağlayan tüm f fonksiyonlarını bulunuz.

4. f : R → R olmak üzere verilen

f (f (x) + y) = f (x2 − y) + 4f (x)y

eşitliğini sağlayan f fonksiyonlarını bulunuz.

2.20.4 Çözümler
1. Eğer y = 0 alırsak,
f (f (x)) = [1 + f (0)]f (x)
eşitliği elde edilir. Eğer x yerine x + y alırsak

[1 + f (0)]f (x + y) = f (f (x + y)) = f (x + y) + f (x)f (y) − xy

eşitliğinden
f (0)f (x + y) = f (x)f (y) − xy
elde edilir. Eğer y = 1 alınırsa,

f (0)f (x + 1) = f (x)f (1) − x


2.20. FONKSİYONEL DENKLEMLER 181

eşitliği elde edilir. Eğer y = −1 ve x yerine x + 1 alınırsa


f (0)f (x) = f (x + 1)f (−1) = f (x + 1)f (−1) + x + 1
eşitliği elde edilir. Eğer son iki denklemden f (x + 1)’i çekersek
[f 2 (0) − f (1)f (−1)]f (x) = [f (0) − f (−1)]x + f (0)
eşitliği elde edilir. Eğer f 2 (0) − f (1)f (−1) 6= 0 ise f − fonksiyonu lineer-
dir. Eğer f 2 (0)−f (1)f (−1) = 0 ise son denklemde x = 0 alarak f (0) = 0
olarak bulunur. Bu durumda fonksiyon f (x)f (y) = xy olacaktır. y = 1
alırsak f (x)f (1) = x olacaktır. Dolayısıyla f (1) 6= 0 ve f (x) lineerdir.
Sonuç olarak, f (x) = ax + b olarak soruda verilen denklemde yerine
koyarsak a = 1 ve b = 0 bulunur. Demek ki, fonksiyon f (x) = x’dir.
2. Bütün n tamsayıları için
f (0) + f (3n) = 2f (n) = f (n) + f (2n)
olacağından
f (3n) + f (3n)
f (n) = f (2n) = = f (3n)
2
olacaktır. Demek ki f (n) = f (0) eşitliği vardır. Dolayısıyla tüm tüm
sabit fonksiyonlar birer çözüm olacaktır.
3. Eğer x = 1, y = −1 − f (1) ve a = f (y) + 1 alırsak eşitliğimiz
f (a) = f (f (y) + 1) = y + f (1) = −1
olacaktır. Eğer y = a ve b = f (a) olarak alırsak
b = f (xf (a) + x) = ax + f (x)
ve
f (x) = −ax + b
olacaktır. Eğer sorudaki denklemde yerine koyarsak,
ax2 y − abx − ax + b = xy − ax + b
eşitliği bulunur. Polinom eşitliğinden a = ±1 ve b = 0 dolayısıyla f (x) =
x veya f (x) = −x bulunur.
182 BÖLÜM 2. KONULAR

4. y = x2 olarak alırsak

f (f (x) + x2 ) = f (0) + 4x2 f (x)

bulunur. y = −f (x) alınırsa

f (0) = f (f (x) + x2 ) + 4f (x)2

eşitsizliği elde edilir. Bulduğumuz bu iki eşitliği karşılaştırırsak, her bir


x değeri için f (x) = 0 veya f (x) = x2 olmalıdır. Varsayalım f (a) = 0
olsun. x = a olarak alırsak

f (y) = f (a2 − y)

bulunur. y 6= a2 /2 için
y 2 6= (a2 − y)2
ise f (y) = 0 olacaktır. Son olarak x = 2a ve y = a2 /2 alırsak

f (a2 /2) = f (7a2 /2) = 0

olacaktır. Demek ki, f (x) = 0 veya f (x) = x2 olmalıdır.

You might also like